Anda di halaman 1dari 309

MERP Medical Immunology

Course:
Instructor:
Textbook:

Microbiology and Immunology


Inna Lindner, Ph.D. ilindner@devrymedical.org
Recommended: 1) The Immune System 4rd Edition, by Peter Parham ISBN: 978-0-8153-4466-7
2) Medical Microbiology 7th Edition by Patrick Murray, Ken Rosenthal, Michael Pfaller ISBN:
987-0-323-05470-6
LECTURE LEARNING OBJECTIVES

Unit I: Immunology
Lecture 1: Introduction to Immunology

Describe the difference between innate and adaptive immune response.


List the cells of the immune system and their origin
Describe innate barriers of the immune system
Describe the meanings of the word inflammation
Describe the differences between antigen recognition of cells in the adaptive response and cells
in the innate response
Describe the components of and timing of adaptive immunity in both primary and secondary
response
Describe the basic differences between B cells and T cells
List the advantages and disadvantages of passive versus active immunity
Describe the function of the primary and secondary lymphoid organs

Describe and diagram the trafficking of lymphocytes and antigens in the body.
List the major structures of the lymph node
Describe the basics behind how B cells and T cells recognize antigen

Lectures 2 and 3: Generation of B cell diversity and B cell development

Describe the steps involved in the differentiation of B cells.


Draw out the mechanism behind somatic recombination
Define the terms and know the functions of RAG, TdT, mu chain, allelic exclusion
Describe the mechanism behind B cell development from stem cell to mature B cell
Explain what determines the isotype and which isoype is expressed at which point of B cell
development
Explain the mechanism behind the events involved in somatic hypermutation
Explain mechanism behind the events involved in isotypes switching
Describe the function of different isotypes
List the causes of SCID

Detail the structure of antibodies.


Describe structure and function of Idiotype
Describe structure and function of Isotype
1

MERP Medical Immunology

Describe structure and function of Fc region


Describe structure and function of Fab region
Be familiar with the role of antibody genetics (heavy and light chain genes, variable and
constant regions, number of alleles for each gene, the possible combinations of different genes)
in generation of antibody

Lecture 4: Antigen Processing and MHC Presentation


MHC
Describe the structure and function of MHC proteins
List the locations of various types of antigens
Detail the structure of MHC I vs MHC II
Define the terms polymorphic and polygenic
List the genes encoding MHC I and MHC II
List the cells that express MHC I and MHC II
Describe the mechanism behind the binding of antigen to MHC I vs MHC II
Explain the basics of disease mechanism in Bare lymphocyte syndrome
T cell Development
Describe the steps in the differentiation of T cells.
Describe the basic structure of the thymus (cortex, corticomedullary junction, medulla, TEC,
DC, thymocytes, macrophages)
List the steps in T cell development from stem cell to mature single positive T cell
Detail the structure of TCR
Define the role of RAG, TdT, double negative cells, double positive cells in T cell development
Describe the principle behind positive and negative selection and the consequence of lacking these
processes
Lectures 5 and 6: Activation of Immune Response
1. Describe the cells/proteins involved in the activation of CD8 T cells, CD4 T cells and B cells. Be
familiar with the sequence of events that takes place during the activation of immune response.
Describe diapedesis
Describe the differences between the three types of antigen presenting cells
Detail the steps in CD8 response (MHC I, TCR, B7, CD28, CD8, IL2, perforin, granzymes, cell
death)
Detail the steps in Th1 response (CD4, MHC II, B7, TCR, CD28, IL2, IL12, CD40LCD40R,
Interferon gamma, macrophage activation, intravesicular pathogen)
Detail the steps in Th2/Tfh response (CD4, MHC II, B7, TCR, CD28. IL2. IL4, IL4, IL10, B cells,
somatic hypermutation, isotype switching, plasma cells, memory cells, IgM vs. other isotypes)
Detail the steps in inactivation of immune response
Describe the process of peripheral anergy
2. Describe the mechanism behind T cell independent B cell responses.
List the properties of antigens that stimulate these responses
3. Describe the role of haptens in immunity
Lecture 7: Functions of antibodies, Natural Killer Cells, Blood Types, Antibody Profile in Infants
1. Describe the different roles of antibodies in fighting different infections, and know how the difference in
isotypes relates to antibody functions (neutralization, opsonization, complement activation).
2

MERP Medical Immunology

2. Describe the role of complement in immune response.


Differentiate Classical vs Alternative cascade
Describe the result of deficiency in MAC
Describe the result of deficiency in C3
Draw out the basics of complement cascade.
3. Describe the mechanism of Natural Killer Cell induced cell death.
Describe NK mechanism of killing through ADCC
Describe NK mechanism of killing through lack of MHC I
4. Describe the importance of blood types in transfusion reactions. Detail the principle behind the
Rh factor and hemolytic disease of the newborn.

List the various blood types


Describe the relationship between blood types and preformed antibodies
Describe the conditions in which one would see agglutination reactions
Be able to interpret the results of testing for blood type
Describe the characteristics of universal donor vs. universal recipient

5. List the antibodies produced by newborns and the course of antibody development.
Determine when children are protected against infections and why
Determine when children are most susceptible to infections and why
Lecture 8: Hypersensitivity Reactions
Differentiate between the mechanisms of the four hypersensitivity reactions.
Describe the role of sensitization in hypersensitivity
List the steps in IgE production in Type I hypersensitivity
Differentiate early vs. late phase in Type I hypersensitivity
Describe chronic desensitization
Describe the role of ADCC in Type II hypersensitivity
List examples of foreign antigens versus selfantigens in Type II hypersensitivity
Draw the mechanism of druginduced hypersensitivity
List examples of Type II hypersensitivity (know disease mechanism of blood transfusion
reaction, hemolytic disease of the newborn, hyperacute graft rejection)
List examples of foreign antigen verses self-antigen induced Type III hypersensitivity
Explain the mechanism of disease in Arthurs reaction, serum sickness, rheumatoid arthritis,
SLE, Farmers lungs
Describe Type IV mechanism in detail
List examples of foreign antigen verses self-antigen induced Type IV hypersensitivity
Explain the mechanism of disease in contact dermatitis, poison ivy, PPD test, diabetes Type I, graftvs-host and
hostvs.graft, celiac disease, multiple sclerosis
Unit 2: Virology
Lecture 9: Introduction to Virology I
1. Describe the viral life cycle.
Define viruses
Explain differences between receptor mediated endocytosis and receptor mediated
fusion
3

MERP Medical Immunology

Differentiate between lysis, budding, exocytosis


Differentiate between lytic, persistent, latent and chronic infections
1. Describe the main differences between nonenveloped and enveloped viruses.
Difference in stability
Difference in entry
Difference in exit
Difference in transmission
Difference in immune response required
Difference in structure
Lecture 10: Introduction to Virology II
1. Describe the differences between viruses that have RNA versus DNA as their genome.
Describe the difference in stability within a cell
Describe the difference latency vs. no latency
Be able to answer why the replication of a particular virus is in the nucleus vs. the cytoplasm
Describe the mechanism and importance of temporal regulation of transcription
2. Describe in detail the steps in the replication of the RNA genome, stressing the differences between
positive and negative RNA.
3. Describe in detail the steps in the replication of the DNA genome.
4. For each viral family list the steps necessary to produce viral RNA and viral proteins.
Lecture 11: DNA Viruses: Poxviridae and Parvoviridae
1. Describe clinically and biochemically Poxviridae family.
Describe the structure of Pox
Describe the unique features of Pox
Describe the reasons for eradication of smallpox
Describe the importance of vaccinia
Describe the spread, clinical presentation, population at risk, treatment for Molluscum
contagiosum
2. Describe clinically and biochemically Parvovirus B19.
Describe the cell target of Parvovirus
Describe the diseases progression of Parvovirus
List human diseases caused by Parvovirus
Describe the role of immune response in Parvovirus infections
Lecture 12: DNA Viruses: Papillomaviridae, Polyomaviridae and Adenoviridae
1. Describe clinical and biochemical features of Papilloma and Polyomaviridae families
Describe the various cells that Papilloma virus targets and the replication cycle in the cells
Describe the diseases that various types of HPVs can cause
Describe the mechanism behind cervical carcinoma
Describe how the viruses belonging to this group can immortalize cells
Describe the composition of HPV vaccine
Describe the population and diseases that Polyoma viruses cause
4

MERP Medical Immunology

2. Describe clinical and structural features of Adenoviridae family


Describe how the viruses belonging to this group can immortalize cells
Lecture 13: DNA Viruses: Herpesviridae
1. Describe clinically and structurally the family of Herpesviridae. List the features that are common to all
Herpesviridae. Differentiate clinically among the different disease caused by Herpesviridae.
Describe HSV disease/clinical presentation
Describe where HSV causes latency/recurrence
Describe the effect of HSV on the central nervous system
Describe the role of HSV in neonatal disease
Describe Tzanck smear and inclusion bodies
Describe VZV disease/clinical presentation (specifics about the rash and how it differs from
other rashes)
Describe where VZV causes latency/recurrence (chicken pox versus shingles)
Describe VZV diagnosis and prevention
Describe the clinical features of diseases caused by EBV
Know where EBV causes latency and recurrence
List the target cells for EBV
Explain the importance of T cells in control of all Herpes (including EBV)
Describe the link between EBV and Burketts lymphoma
Describe EBV diagnosis (detail the importance and time course of heterophile antibodies,
atypical lymphocytes, EBV specific antibodies)
Describe CMV disease mechanism and list populations at risk
Describe the mechanism of CMV spread
Be able to describe the characteristic histology of CMV
HHV6 (roseola) describe the rash and differentiate between this rash and all the other
childhood rashes
Lecture 14: DNA Viruses: Hepadnaviridae
1. Be able to describe clinically and structurally the causative agent of Hepatitis B.
Describe the unique features of this virus including its replication cycle (role of
RTase).
Explain the significance of HBsAg, HBcAg, HBeAg, HBsAb, HBcAb, HBeAb
Interpret the serological charts for acute and chronic Hepatitis B infected individuals
Describe Hepatitis B as a cause of PHC
Describe the vaccine composition and its administration
Explain the role of passive immunization in Hepatitis B
Lecture 15: RNA Viruses: Piconaviridae and Orthomyxoviridae
1. Describe biochemical and structural features of Picornaviridae.
Relate the structure of these viruses to their stability in the environment
Describe the relationship between RNA genome and viral stability inside the host cell
Describe the immune responses important for clearing these viruses
List the mode of transmission of Picornaviridae
Detail the advantages and disadvantages of live and killed polio vaccines
Identify the diseases caused by this family based on description of symptoms
5

MERP Medical Immunology

Differentiate between Hepatitis A and B

2. Describe biochemical and structural features of Orthomyxoviridae.


Describe the mechanism and consequences of genetic drift versus shift
Draw out the replication cycle of Orthomyxoviridae
List the symptoms and complications of flu
Lecture 16: RNA Viruses: Paramyxoviridae, Togaviridae and Flaviviridae
1. Describe biochemical and structural features of Paramyxoviridae.
List the differences between the systemic and local infections that belong to this family of
viruses
Describe the properties of the family including envelope structure, fusin protein, receptor
mediated fusion, syncytia formation, RNA replication
Describe disease mechanism for measles transmission, spread within the body, description of
the rash, complications, and prevention
Describe disease mechanism for mumps transmission, spread within the body, infection of the
organs, complications, prevention
Describe disease mechanism for RSV transmission, population at risk, complications
Describe disease mechanism for Parainfluenza virus transmission, population at risk,
complications
2. Describe biochemical and structural features of Togaviridae and Flaviviridae.
Describe vectortransmitted versus humantohuman transmitted infections
Describe clinical features of Dengue fever (explain why second time is worse), yellow fever,
WNE
Identify how rubella is different from other childhood rashes
List the differences between Hepatitis A, B and C
Lecture 17: RNA Viruses: Rhabdoviridae
Describe biochemical and clinical features of Rhabdoviridae.
Describe the spread of rabies
Describe the importance of long incubation period
Characterize rabies infection
Explain the serological status of patient during all stages of rabies
Be able to identify advantages and disadvantages of passive and active immunization for rabies
Lecture 18: Retroviridae
Describe in detail the replication cycle of HIV.
Detail the entry, reverse transcription, integration, mRNA synthesis, replication, protein
synthesis, exit, and the role of protease in HIV
Describe the disease mechanism and stages of HIV
Be able to read and Describe HIV serology as it relates to various stages of HIV
Describe briefly the stages of HIV replication targeted by drugs
Unit 3 Bacteriology
Lecture 19: Introduction to Bacteriology
6

MERP Medical Immunology

1. Describe the differences between the gram-positive and the gram-negative bacteria.
Describe various bacterial structures and know what function these structures serve
Describe the differences in transcription mechanism of eukaryotes vs. prokaryotes (Describe lac
operon)
Describe the differences between exotoxins and endotoxins
2. Describe the difference between aerobic and anaerobic bacteria in terms of the enzymes they encode
and growth conditions they prefer.
Detail the role of oxygen in the process of glycolysis, anaerobic fermentation, aerobic
fermentation
Describe the mechanisms by which bacteria are able to disarm oxygen radicals
Identify organisms as strict anaerobes, strict aerobes, facultative anaerobes or airtolerant
3. Describe the various stages of the growth curve of bacteria in liquid media.
Calculate the number of bacteria, generation times etc. based on information given
Lectures 20 and 21: Skin Infections: Staphylococcal and Streptococcal diseases
1. Describe the structures, virulence factors, and clinical presentation of Streptococci and Staphylococci.
Define the functions of Protein A, alpha toxin, exfoliative toxin, enterotoxin, and superantigen
Be able to identify pictorially or based on a description common skin infections (impetigo, boils,
SSSS), GI poisoning and systemic infections that Staphylococci cause
Differentiate which Staphylococcal infections are mediated by bacterial replication vs. secreted
exotoxin
Describe the function of superantigen
Describe the media that selects for Staphylococcus aureus
Describe blood agar and the differences between alpha, beta and gamma hemolysis
Differentiate between the three Streptococcal species (S. pyogenis, S. agalactaiae, and S.
pneumoniae) based on biochemical properties and clinically
List the virulence factors for GAS (capsule, M protein, sAg and pyogenic exotoxin)
Describe Streptococcal infections including strep throat, scarlet fever, rheumatic fever, skin
infections, and systemic invasive infections
Describe the basis of GAS rapid antigen test
Describe the disease mechanism of GBS and Describe the CAMP test
Identify diseases caused by and the virulence factors for S. pneumonaeie
Describe the vaccines for S. pneumonaeie
Lectures 22 and 23: Food Infections vs. Food Poisoning
1. Differentiate clinically between general symptoms of food poisonings (Staphylococcus aureus, Bacillus
cereus, Clostridium spp.) and food infections (E. coli, Salmonella, Shigella, Cholera, Listeria).
Describe structurally and biochemically bacteria that cause food poisonings and food infections
For E. coli, differentiate the strains that cause inflammatory versus non-inflammatory diarrhea
and know the functions of toxins for EHEC and ETEC
For Salmonella discuss the differences between strictly enteric infections and S. typhi
Describe the various forms of disease caused by Shigella and the infection site
Describe the function of Shigella toxin
Describe the function of toxin in cholera, relate its function to disease and Describe the use of
oral rehydration
7

MERP Medical Immunology

Describe the use of, and identify growth on of McConkey and Hektoen media
Describe the composition of the vaccine where one exists
For Bacillus, describe the clinical symptoms and virulence factors for anthrax
Be able to differentiate biochemically B. cereus from B. anthraces
For Clostridium, describe skin disease caused by C. perfringens and neurological disease
symptoms for C. botulinum, as well as disease mechanism for C. tetani
For E. coli describe UTI and neonatal meningitis as well as septicemia
Biochemically distinguish the three causes of neonatal meningitis: E coli, GBS and Listeria

Lecture 24: Bacterial Meningitis and Respiratory Infections


1. Describe and differentiate structurally/biochemically the common causes of non-neonatal meningitis
(Streptococcus pneumoniae, Neisseria meningitides, Haemophilus influenzae). Describe the pathogens
that cause neonatal versus non-neonatal meningitis.
Describe clinical presentation of meningitis caused by Neisseria and the symptoms of
meningococcemia
Describe the composition of N. meningitides vaccine
Differentiate biochemically between N. meningitides and N. gonorrhoeae
Describe the clinical presentation of gonorrhea in men versus women
List the features of gonococcemia and ophthalmia neonatorum
Describe ways to diagnose gonorrhea (media used, biochemical tests, gram-stain, etc.)
2. Describe the structure and clinical symptoms of respiratory pathogens: Corynebacterium diphtheria,
Bordetella pertussis, Haemophilus influenza
Identify respiratory versus systemic diphtheria symptoms
Describe the function of diphtheria toxin and its role in myocarditis
Explain the mechanism behind the Elek test and the media used for C. diphtheria
List the clinical and structural differences between H. influenzae type b and non-typable H.
influenzae
List the clinical features of epiglottitis, cellulitis and meningitis
List the growth requirements for H. influenza
Describe the clinical symptom of B. pertussis whooping cough
Describe the composition of the vaccines for these pathogens
Describe the basics about the media used to grow Bordetella
Lecture 25: Mycobacteria
1. Describe the structure and pathogenesis of Mycobacteria tuberculosis, and detail how these bacteria
are structurally different from all the others discussed.
Describe acidfast stain
Describe reasons for the environmental resistance of TB
Describe the various presentations and stages of TB (Asymptomatic TB, Primary TB, Reactivated
TB, military TB)
Describe the histology and function of granulomas
Describe what happens in caseating necrosis
Detail the mechanism behind PPD test
List the population at risk

MERP Medical Immunology

IMMUNOLOGY MODULE
LECTURE 1: INTRODUCTION TO IMMUNOLOGY
Learning Objectives:

Describe the difference between innate and adaptive immune response.


List the cells of the immune system and their origin
Describe innate barriers of the immune system
Describe the meanings of the word inflammation
Describe the differences between antigen recognition of cells in the adaptive response and cells
in the innate response
Describe the components of and timing of adaptive immunity in both primary and secondary
response
Describe the basic differences between B cells and T cells
List the advantages and disadvantages of passive versus active immunity
Describe the function of the primary and secondary lymphoid organs

Describe and diagram the trafficking of lymphocytes and antigens in the body.
List the major structures of the lymph node
Describe the basics behind how B cells and T cells recognize antigen

All cellular elements of blood


(and of innate and adaptive
immune system) arise from
hematopoietic (multipotent)
stem cells in the bone
marrow. These multipotent
cells divide to produce more
specialized progenitor cells
called; Lymphoid and Myeloid
Stem cells. In the
hematopoietic system,
'leukocyte' is the term given
to white blood cells. There
are many different types of
leukocytes in the body; for
example lymphocytes are
specialized leukocytes that
are derived from lymphoid
stem cells. The two most
important lymphocytes in the
adaptive immune system are
T cells and B cells.

HSC

MERP Medical Immunology

Immunity Protects Us from Infectious Disease

10

MERP Medical Immunology

Antigen (Ag) is the name for any molecule that stimulates an immune response. Most
antigens are pathogen proteins or carbohydrates that are "foreign" or "non-self" to the host.
Innate pathogen barriers-first defense:
1. Physical Barriers to pathogen entry include skin and mucous membranes with tight junctions
between cells.
2. Mechanical responses include the movement of cilia during blinking, the process of
sneezing, coughing, vomiting all working to remove antigen.
3. Chemical agents include fatty acids on the skin, HCl in the stomach, lysozyme and other
digestive enzymes in tears and mucus.
Innate immune response-second defense:
Even simple multi-celled animals like sea squirts and starfish, as well as many plants, have
defense systems that can recognize a generic "danger" signal and respond to antigens by engulfing
or walling off foreign organisms.
Humans have a corresponding innate immune response that begins immediately in response
to tissue damage. This innate response is mainly mediated by phagocytes. Phagocytes (such as
macrophages and neutrophils) are attracted to the site of infection or tissue damage and engulf and
destroy pathogens, as well as our own dead or damaged cells. In some instances, innate immune
responses alone are effective at eliminating antigen from the body.
1. Phagocytes of the innate system: neutrophils, and monocyte/macrophages in blood and in
many body tissues

2. Inflammatory responses: attract white blood cells (leukocytes) to the infection site
Inflammation is part of the complex biological response of vascular tissues to harmful stimuli.
This response is mediated mainly by small proteins called cytokines. There are several roles
for cytokines:
i.

ii.

Cytokines signal the blood vessel endothelial cells to express more adhesion
molecules, so that leukocytes can stick and move between the endothelial cells to enter
the tissues.
Cytokines attract leukocytes to the site of infection (these have a special name:
chemokines). As an example, chemokines can attract neutrophils and monocytes to
11

MERP Medical Immunology

iii.

iv.

the site of infection, a process known as chemotaxis. Chemokines and other cytokines
are "locally" produced in tissues throughout your body.
Cytokines increase the amount of fluid that can leave the circulation, so that
antibacterial molecules enter the infection site. They do so by dilating smooth muscle
around the vasculature which increased blood flow and causing contraction of
endothelial cells.
Cytokines signal the bone marrow to produce more leukocytes.

Below is an example of inflammatory response mediated by cytokines

3. Chemical agents Some chemical agents participate in the innate immune response.
These include antibacterial molecules such as defensins and complement which promote
phagocytosis and pathogen destruction. Also antiviral proteins called interferons block virus
replication in host cells.
Adaptive Immunity
Although not antigen-specific, inflammation (via phagocyte recruitment and flushing of tissue
with increased flow) is often enough to eliminate small numbers of bacterial pathogens from the body.
However, if antigen is not eliminated, adaptive immunity, which includes lymphocytes and secreted
molecules called antibodies, can also participate in antigen destruction. There are several
characteristics of adaptive immunity that set it apart from innate immunity.
Adaptive immunity:
1. is antigen-specific
2. is found only in vertebrates
3. responds more quickly and efficiently to a repeat infection (immune memory)
often so efficient that no symptoms develop
4. can be long-lasting but is usually not permanent.
Adaptive immune responses is mediated by lymphocytes and includes:
1. Synthesis of antibody to bind antigen and promote its elimination by B cells
2. T cell-killing of virus-infected cells
12

MERP Medical Immunology

3. T cell-mediated activation of macrophages to destroy phagocytosed pathogens that are


resistant to phagocytosis.
Characteristics of Lymphocytes:
Remember that lymphocytes are specialized types of
leukocytes that are derived from lymphoid stem cells.
Lymphocytes are small, round cells with a large nucleus.
Below is a description of lymphocytes.
1. They are antigen-specific leukocytes.
2. Lymphocytes of adaptive immunity are called B cells
and T cells.
a. Each lymphocyte recognizes one specific
antigen with its receptor.
b. The antigen receptor on B lymphocytes is called an immunoglobulin (antibody) or
BCR (B Cell Receptor).
c. The antigen receptor on T lymphocytes is called the T Cell Receptor (TCR).
d. Each lymphocyte has about 100,000 identical copies of its antigen receptors on its
membrane.
Differences between antibodies of B cells and T cell receptors of T cells

Antibodies bind (are specific for) both protein and non-protein antigens. Antibodies bind
antigen in its native conformation. Antibodies can be secreted, T cell receptors cannot.
TCRs cannot bind antigen directly. Antigen (Ag) must first be processed (cut into peptides)
and presented on major histocompatibility complex (MHC) molecules on antigen
presenting cells (APC). TCRs only recognize peptides.
Professional APCs are: Macrophages, Dendritic cells, B cells.

Similarities between antibodies and T cell receptors

Both T cells and B cells undergo a process called recombination in order to have each cell
express a unique receptor that distinguishes it from another cell.
o Recombination occurs randomly, so the millions of lymphocytes produced each day
collectively have millions of different antigen specificities.
o The large number of different specificities that are produced is called the immune
repertoire.
13

MERP Medical Immunology

Innate and adaptive immune responses collaborate to fight off infection.


Innate immune mechanisms including inflammation and phagocytosis are essential for the efficient
functioning of the adaptive immune system. As the diagram below shows, lacking innate immunity
results in a rapid growth of microorganisms. Lacking adaptive immunity also results in prolonged
infection that usually cannot be cleared, however, the organisms are at least partially controlled by
innate immunity. When operating together, innate and adaptive immunity can clear the organism.
Figure (right): If an individual lacks adaptive immunity, the innate
immunity has some limited capacity to fight pathogens. However, without
innate immunity, an individual would succumb to infection, because
innate immunity is necessary for first line of defense and for recruitment
of adaptive response.

Organs of the Immune System

There are two sets of organs that play a role in the development of immune cells and the activation of
these cells.
1. Primary lymphoid organs are sites where white blood cells develop and mature. Examples include:
a. Bone-marrow (where B cells develop and hematopoiesis takes place)
b. Thymus (where T cells finish development)
An antigen is NOT required for the development of mature antigen-specific T and B
lymphocytes.

14

MERP Medical Immunology

2. Secondary (peripheral) lymphoid organs:


Secondary lymphoid organs function
to bring together leukocytes and
antigens. They line the mucous
membranes of the respiratory,
digestive, and urogenital systems
where contact with pathogens is
highest. They are also found deeper
within the body.
Mature nave lymphocytes get
activated in secondary lymphoid
organs. These sites include:
a. Lymph nodes (distributed all over
the body, collect Ag from tissue)
b. Spleen (where blood-borne antigens, especially bacteria, encounter the immune system)
c. MALT (Mucous Associated Lymphoid Tissue): tonsils (lymphoid tissue in the respiratory tract),
appendix, peyers patches (lymphoid tissue of the gut).
The trafficking of immune cells
The fluid, called lymph (which is similar in composition to plasma), drains tissue, collects in the
lymphatic vessels and passes through the lymph nodes on its way back to the blood circulation.
Lymph circulates in only one direction - from tissues, to lymph nodes, from lymph nodes to the
thoracic duct, back to the bloodstream, back to the
tissues.
Below are the steps that take place if the tissues
are infected.
1b

1. 1a. Antigen is carried to the nearby (draining)


lymph nodes via afferent lymphatic vessels. It is
carried either by a cell called a dendritic cell, or
travels alone.
1b. Nave B cells and T cells leave the primary
lymphoid organs to populate the blood pool.
1c. High endothelial venules (specialized
anatomical structures in the vasculature) allow
lymphocytes to leave blood and enter the lymph
nodes.

1c
2
a

2. In the lymph node, antigen comes in contact


with lymphocytes to initiate an adaptive immune
response.

15

1c

MERP Medical Immunology

3. Efferent lymphatic vessels transport lymph and activated lymphocytes from the lymph nodes
back into lymphatic circulation, and then into blood circulation.
4. From circulation, activated lymphocytes go to the tissues
5. At any given time many leukocytes recirculate throughout the body.
Function of adaptive immune system:
Mature nave T and B cells (those that have not yet bound foreign antigen) leave the primary
lymphoid organs and recirculate via blood through the secondary lymphoid organs.
If a lymphocyte binds antigen, it proliferates
into a clone of lymphocytes that differentiate
into antigen-eliminating effector cells.
In a primary response to antigen, the
time it takes to select and activate a specific
T cell and B cell is 1-4 weeks long. At this
time, not only effector cells that fight the
current antigen but also memory lymphocytes
specific for the same antigen are produced.
On repeat exposure to the same antigen
(secondary response), the adaptive
response is faster, stronger and longerlasting. This is due to the expansion of
antigen-specific memory lymphocytes formed
during the previous primary response.
It is important to realize that there are many instances when the activation of the adaptive immune
system in response to an antigen does not result in disease because the organism levels are kept
low. Only when multiplication of the organism extends beyond threshold level will the disease and
clinical presentation become apparent. We secrete antibodies in response to many pathogens that
we encounter however, only a small number of these organisms have actually made us feel sick.

Adaptive immunity can be acquired in two different ways:


1. Actively
16

MERP Medical Immunology

Actively acquired adaptive immunity requires 1-4 weeks to become established and may be
very long-lasting, from years to a lifetime. Active immunity is acquired by:
o Natural infection with the organism
o Vaccination with killed or weakened (attenuated) pathogen or inactivated toxin
(toxoid).

2. Passively
Passive immunity protects as soon as the antibodies are transferred. Although it is specific
for the antigen, it lasts only weeks as the transferred antibodies are removed from the
circulation in a natural process called "turnover" and there are no B cells producing these
antibodies in the recipients body. Passive immunity is acquired from:
o Immune person by the transfer of antibodies
Practice questions:
1.The main advantage of passive immunization over active immunization is that:
a. it can be administered orally.
b. it provides antibody more rapidly.
c. antibody persists for a longer period.
d. it contains primarily IgM.
e. it provides more specific antibody.
2. A lymphoid progenitor cell differentiates into ______ .
a. T cells
b. Macrophages
c. Erythrocytes
d. B cells
e. both a and d
3. The innate immune system is mainly mediated by _____.
a. T cells and B cells
b. Macrophages and neutrophils
c. Erythrocytes
d. Megakaryocytes
4. Cytokine release _____ vascular permeability.
a. increases
b. decreases
5. T cells finish development in the _____.
a. Thyroid
b. Bone marrow
c. Spleen
d. Thymus
e. MALT
17

MERP Medical Immunology

6. T cell receptors can bind to _____.


a. Peptides only
b. Non-peptides only
c. Both peptides and non-peptides
LECTURES 2 AND 3: GENERATION OF B CELL DIVERSITY AND B CELL DEVELOPMENT
Learning objectives:
1. Detail the structure of antibodies.
Describe structure and function of Idiotype
Describe structure and function of Isotype
Describe structure and function of Fc region
Describe structure and function of Fab region
Be familiar with the role of antibody genetics (heavy and light chain genes, variable and
constant regions, number of alleles for each gene, the possible combinations of different genes)
in generation of antibody
2. Describe the steps involved in the differentiation of B cells.
Draw out the mechanism behind somatic recombination
Define the terms and know the functions of RAG, TdT, mu chain, allelic exclusion
Describe the mechanism behind B cell development from stem cell to mature B cell
Explain what determines the isotype and which isoype is expressed at which point of B cell
development
Explain the mechanism behind the events involved in somatic hypermutation
Explain mechanism behind the events involved in isotypes switching
Describe the function of different isotypes
List the causes of SCID
To reach the lymph nodes, B cells and T cells travel
from bone marrow to the blood stream and then enter the
lymph nodes via high endothelial venule. From the lymph
nodes, the lymphocytes exit via efferent lymphatic vessels
and enter tissues through signals received from
inflammation. The vasculature in the inflamed tissues has
a higher blood flow and endothelial cells that have
contracted and are therefore more permeable.

18

MERP Medical Immunology

B cells job is predominantly to secrete antibodies that fight against extracellular antigens.
During B cell development in the bone marrow, many different B cells are generated from identical
stem cells, yet all the B cells differ from each other in their specificity. This differentiation into antigenspecific B cells is antigen independent. However, when a particular B cell encounters an antigen in
the periphery, it gets stimulated by that antigen and by additional signals in secondary lymphoid
organs. Activation then causes proliferation of that particular B cell, production of plasma cells that
secrete the antibody encoded by that B cell,
and generation of long-term memory cells
that help fight the antigen in the future after
soluble antibodies have been degraded.
Immunoglobulin structure
Antibodies are tetramers of two identical
"light" chains (pink) and "heavy" chains
(blue).

The variable region (idiotype) includes the N-terminal


end of the heavy and light chains, and is responsible
for antigen recognition. More specifically and antibody
binds to a specific epitope, also known as antigenic
determinant which usually constitutes a part of a larger
antigen (see Figure on the right).

The constant region (isotype) is usually derived from


one of the five major classes of the heavy chain (mu, delta, gamma, epsilon and alpha). When
a heavy chain with a particular isotype combines with a light chain an antibody is produced.
This antibody could be IgM, IgD, IgG, IgE, or IgA.

19

MERP Medical Immunology

Genomic organization of immunoglobulin gene families


An antibody is encoded by two genes: Heavy chain and Light chain. Each person inherits:
a) 2 alleles of the Heavy chain gene (one allele from mom, one from dad). Within each gene
locus, there are multiple variable (V) regions, diversity (D) and joining (J) regions as well as
several constant region segments (isotypes).

b) 2 alleles of the Kappa light chain gene (one allele from mom, one from dad) each having
multiple V and J regions as well as one C region.

c) 2 alleles of the Lambda light chain gene (one allele from mom, one from dad) each
having multiple V and J regions as well as a C region.

20

MERP Medical Immunology

B cell development and the generation of antibody diversity


Important Definitions:
Allotypes the sequences of inherited immunoglobulin genes, which vary by individuals. They vary
because the genes that code for the L and H chains are polymorphic (there are multiple alleles of
these genes in a population), and individuals inherit two different alleles (or allotypes) of each gene.
The allotypic differences in heavy and light chain genes are clustered in the constant regions of the
genes.
Isotype refers to the constant region of an antibody. Even though there is more to an antibody than
the constant region, an antibody as a whole carries the name of its isotype. For example, IgM and
IgG are different isotypes; the constant region of their heavy chains (mu and gamma respectively) are
different antigenically. This nomenclature does not say anything about the antibodys specificity. The
five immunoglobulin isotypes (or classes) are the following - IgG, IgM, IgA, IgD, and IgE have heavy
chains that are different. The IgG isotype is further subdivided into four subtypes (sub-isotypes),
IgG1, IgG2, IgG3, and IgG4, similarly IgA1 and IgA2 are different sub-isotypes. Kappa and lambda
chain genes are not isotypes of each other; they are distinct genes that have been duplicated in time.
They are considered different isoforms of the light chain.
Idiotypes are the antigenic
determinants formed by the specific
amino acids in the variable region. Each
idiotype is unique for the
immunoglobulin produced by a specific
clone of antibody-producing cells.
There are several factors that contribute
to diversity of idiotypes among B cells (so that as a result each antibody will bind to a different epitope
of an antigen):

1. Somatic recombination of immunoglobulin genes leads to:


Combinatorial diversity which results from mixing and matching of various VDJ segments (due
to RAG)
Junctional diversity which causes imprecise joining of VDJ segments (due to TdT, discussed
later)
2. Pairing of different light and heavy chains to form a functional antibody
21

MERP Medical Immunology

3. Somatic hypermutation (post antigen encounter).

Factors 1 - 2 occur in the bone marrow.


Immunoglobulin gene rearrangement
In the figure to the right,
the recombination of the

a.

heavy chain is shown. It


involves the enzymes RAG
and TdT. The genomic DNA in
(Figure a) is not rearranged
and contains all of the
segments. The dotted lines
represent the V, D, and J

b.

regions in between the


adjacent segments.
Specialized sequences called

c.

recombination signal
sequences (RSS) mark the 3 end of all the V regions, the 5 end of all the J regions and both the 5
and the 3 ends of the D segments. During recombination, RAG first recombines a 3 RSS of a
particular D segment with a 5 RSS of a J segment creating a DJ joint. After a D and J are joined,
RAG recombines a 3 RSS of a particular V region with the 5 RSS of the D segment previously used
by RAG for recombination (in b and c of the figure above). In figure above we can see that D2
joins with J4 to form D2J4 and V23 joins with D2J4. Note that in panel b of the figure above the 5
Vs (V1 through V23) and the 3 Js (J4 through J6) remain on the genomic DNA (b). Panel b of the
figure represents recombined DNA.
c. In Figure c above, the transcription of the heavy chain gene begins with the promoter of
the V region that participated in recombination, in this case V23, and proceeds all the way through the
constant region mu portion, where transcription ends. This generates a pre-mRNA (not shown) that
contains V23-D2-J4-J5-J6-mu sequence. Note that while the pre-mRNA contains only the V and the
D segments that were used in recombination, it still contains the J segments downstream of
recombination. RNA processing then splices out the downstream Js (in this case J5 and J6), so that
the final transcript contains only the recombined portions of the variable region and the mu portion of
the heavy chain c. In the figure c above, the final mRNA transcript would read V23D2J4mu.
22

MERP Medical Immunology

Note that the


isotypes (gene
regions
encoded by the
constant portion
of the heavy
chain) are not
altered during B
cell
differentiation in
the bone
marrow (figure above). Initial mRNA, however, reflects transcription through the mu region only. The
figure above is also omitting the upstream V regions (V1 V23) and the downstream J regions (J5
and J6) which should still be on the recombined DNA.
Overview of B-cell development:
The recombination of the heavy and light chain occurs during specific stages of B cell development.

Stem cell (gives rise to common lymphoid progenitor CLP)

Pro B cell

Pre B cell

Immature B cell

Mature B cel

1. In the Pro-B
cell VDJ
gene
segments

1. Division
2. Rearranging
light chain
(k or lambda)

of the

alternative RNA
splicing

heavy chain
are

1. VDJ
rearranged
2. Expression of
m protein on cell
surface
3. Allelic
exclusion of m
gene completed

Heavy chain VDJ rearrangement

rearranging
via RAG 1
and RAG 2

Steps completed

1.
2.
3.
4.

VDJ rearranged
VJ rearranged
IgM expressed
Allelic
exclusion of
light chain
completed

IgM and IgD expressed

Steps completed

enzymes
which utilize the recombination sequences (RSS). RSSs are composed of seven conserved
nucleotides (a heptamer) that reside next to the gene encoding sequence followed by a spacer
23

MERP Medical Immunology

(containing either 12 or 23 unconserved nucleotides) followed by a conserved nonamer (9


base pairs). Only a pair of dissimilar spacer RSSs are efficiently recombined (i.e. one with a
spacer of 12 nucleotides will be recombined with one that has a spacer containing 23
nucleotides). After the RAG enzyme cuts two RSS sequences, the TdT fills in the ends of
broken DNA.
2. In the pre-B cell the heavy chain called the mu chain is expressed as a transmembrane protein
on the surface of the cell. This is the product of either moms or dads heavy chain allele
recombination. The mu chain expression signals the other heavy chain allele to be shut down,
in a process called allelic exclusion.
The pre-B cell undergoes division and subsequent light chain rearrangement in the
daughter cells. This division step creates more diversity of B cell specificities because
all daughter cells of a particular pre-B cell will have identical heavy chains but not the
same light chains. Due to the randomness inherent to the process of recombination the
light chains recombination will yield different VJ joints in the different daughter cells.
Light chain recombination occurs in an orderly fashion such that k (kappa) light chain
gene rearrangement occurs first (on either moms or dads allele), and only when both
alleles are exhausted (i.e. kappa rearrangement was unsuccessful), then lambda light
chain gene is rearranged if necessary. However, if a successful recombination event
occurred, the recombination of the light chain seizes, causing the expression of
recombined gene only. This ensures the allelic exclusion of the light chain.
3. Immature B cell- Once the light chain is rearranged, it combines with the heavy chain in the
ER, and a functional IgM is expressed on the surface.
As a result of the above processes, only one antibody specificity is produced per B cell.
Monospecificity of B cells is ensured by allelic exclusion: as soon as the heavy chain gene
segments are rearranged, heavy chain protein is expressed on the cell surface. The surfaceexpressed protein generates a signal that switches off gene rearrangements on the second
chromosome and at the same time turns on light chain rearrangement. After the light chain is
rearranged and is synthesized, it will pair with the heavy chain to give rise to a complete membrane
immunoglobulin (IgM). This generates a signal that turns off further light chain rearrangement.

24

MERP Medical Immunology

At this point an immature B cell samples


the environment in the bone marrow
(see Figure on the right). Only those B
cells that do not recognize antigen in the
bone marrow (and are therefore not selfreactive) will proceed to the next step.
4. Mature B cell - In the mature
stage, the B cell begins to
express both IgM and IgD
isotypes (bottom). This is due to
the mRNA processing that allows
a longer mRNA transcript to be
produced at this stage
(containing both the mu and delta
region). This transcript is alternatively spliced to yield variable (VDJ) region mRNA adjacent to
the mu OR the delta constant region. Despite alternative splicing, in the mature B cell stage,
the constant region of the heavy chain DNA is unaltered, but the chromatin structure changes
so that the delta region of the gene is available for transcription. The remaining isotype regions
are still there on the gene, but are not accessible for transcription until after the antigen in
encountered. The mature B cell leaves the bone marrow and populates the blood stream.

25

Alternative RNA splicing

MERP Medical Immunology

A look ahead: after B cell encounters the specific antigen


After it encounters antigen, and provided it receives additional stimulatory signals, the B cell
gets activated. This causes the B cell to divide and for some of the daughter cells to release IgM as a
pentamer in a secreted form. Other daughter cells undergo two additional processes that result in a
better affinity of the antibody for its antigen through a process of somatic hypermutation and a
change in the heavy chains constant region through a process called isotype switching.
Somatic hypermutation
The somatic hypermutation mechanism involves specific mutations in the
variable region of the antibody gene in dividing B cells. For some daughter B
cells, these mutations result in an antibody that is more specific to the initial
antigen than was the original antibody molecule; these B cells are selected to
differentiate into plasma cells in a process known as affinity maturation. Some
of the daughter cells with high affinity antibody do not secrete their antibody and
are kept as memory cells with membrane bound immunoglobulin. Plasma cells
and memory cells that are generated as a result of somatic hypermutation
accumulate random mutations in their DNA. Selection by antibody binding
ensures that the antibody with the highest affinity for its antigen will be selected.
Isotype switching (Figure right)
Unlike simultaneous expression of IgM and IgD seen in the mature B cell, isotype switching
causes a DNA loss in the
constant region of the heavy
chain gene. This involves
recombination between the
switch region contained before
the mu portion and the switch
region located before one of the
other isotypes (other than delta).
All intervening DNA between the
two switch regions (including the
mu, the delta, and any other
isotypic region up to the second
switch region) is removed. The B cell now expresses a different isotype encoded by a nucleotide
sequence downstream of the mu and delta regions.
26

MERP Medical Immunology

B cell activation

Isotype switching results in a functionally different Ab

The summary of various processes that effect antibody variable and constant regions

Practice questions:
1. Two distinct IgM-producing, nave B cells are likely to have:
a.
b.
c.
d.
e.

Same idiotypes, different isotypes


Different idiotypes, same isotypes
Different idiotypes, different isotypes
Same idiotypes, same isotypes
Antibodies that cannot recognize any antigen

2. The lack of which of the following enzymes would result in SCID?


a.
b.
c.
d.

Ligase
DNA dependent DNA polymerase
RAG
DNA dependent RNA polymerase
27

MERP Medical Immunology

e. None of the above


3. Diversity is an important feature of the immune system. Which one of the following statements
about it is INCORRECT?
a. Humans can make antibodies with many different VH X VL combinations
b. A single cell can synthesize IgM antibody, then switch to IgA antibody
c. The hematopoietic stem cell carries the genetic potential to create more than 104 different B cells
d. A single B lymphocyte can produce antibodies of many different specificities, but a plasma cell is
monospecific
e. Heavy chain has more diversity than light chain
4. Your patient became ill 10 days ago with a viral disease. Laboratory examination reveals that the
patient's antibodies against this virus have a high ratio of IgM to IgG. What is your conclusion?
a. It is unlikely that the patient has encountered this organism previously
b. The patient is predisposed to IgG-mediated hypersensitivity reactions
c. The information given is irrelevant to previous antigen exposure
d. It is likely that the patient has an autoimmune disease
e. The patient has recovered from his/her infection
5. You find a patient that has a rare disease that arrests their B-cell development in the immature
stage. You analyze B cells in this patient. What do you expect to see? (How would the answer
change if I asked about what you would find in the serum of the patient?)
a. IgM only
b. IgG only
c. IgA only
d. IgM and IgD
e. None of the above
6. You see a patient during rounds that has been exposed to heavy amounts of radiation in an
industrial accident. The patients bone marrow has been particularly damaged, and is functionally
useless at this time. What is the only source of idiotypic diversity in this patients remaining B cell
population?
a. Combinatorial variability
b. Junctional variability
c. Heavy/Light chain pairing variability
d. Somatic hypermutation
7. Which of the following is the immunoglobulin that is initially seen on the primary immune response?
It is present as a monomer on B cell surfaces but as a pentamer in serum.
a. IgG
b. IgM
c. IgE
d. IgA
e. IgD

28

MERP Medical Immunology

LECTURE 4: ANTIGEN PROCESSING, MHC PRESENTATION, T CELL DEVELOPMENT


Learning Objectives:
MHC
1. Describe the structure and function of MHC proteins
List the locations of various types of antigens
Detail the structure of MHC I vs MHC II
Define the terms polymorphic and polygenic
List the genes encoding MHC I and MHC II
List the cells that express MHC I and MHC II
Describe the mechanism behind the binding of antigen to MHC I vs MHC II
Explain the basics of disease mechanism in Bare lymphocyte syndrome
T cell Development
Describe the steps in the differentiation of T cells.
Describe the basic structure of the thymus (cortex, corticomedullary junction, medulla, TEC, DC,
thymocytes, macrophages)
List the steps in T cell development from stem cell to mature single positive T cell
Detail the structure of TCR
Define the role of RAG, TdT, double negative cells, double positive cells in T cell development
Describe the principle behind positive and negative selection and the consequence of lacking these
processes
Why do we need MHC presentation?
While B cells recognize
unprocessed antigen found in
the extracellular space, T cells
only recognize proteins that
have been processed into
peptides and are presented on
a host encoded protein called
major histocompatibility complex (MHC) (right bottom figure).
There are also some structural differences
between T and B cell receptors. An
immunoglobulin which consists of two copies
of the heavy and light chain proteins, the T
cell receptor is composed of a single beta
chain and a single alpha chain. While TCR
contains a transmembrane domain unlike
immunoglobulin, TCR is never secreted.

29

MERP Medical Immunology

There are also similarities between an immunoglobulin and a TCR. Both of these molecules
are composed of two distinct proteins and each chain with these receptors contains variable and
constant regions, such that the variable region differ between receptors on different cells.

The biological role of MHC proteins is to bind small peptides and to "present" these foreign or
non-foreign antigens at the cell surface for the inspection by T cell antigen receptors. Below are
examples of types of pathogens that are presented on MHC protein.
1. Endogenous pathogens are those that reproduce in host cell cytosol. They include
viruses, intracellular bacteria and protozoan parasites. Since antibody cannot enter infected cells,
cytotoxic T cells (CD8 T cells) must be activated to recognize and kill the infected cells. To be
activated by endogenous antigen, CD8 T cells use their TCR to bind endogenous antigen peptides
presented on membrane MHC I proteins of target cells.
2. Exogenous pathogens include bacteria, their soluble toxins, extracellular parasites and
fungi. They can often be phagocytosed and destroyed by lysosomal enzymes or oxidative burst in
neutrophils and macrophages, especially with the opsonizing activities of complement and antibody.
However, some pathogens persist unharmed and protected in phagocytic vesicles of
phagocytes and therefore need to be dealt with. Regardless of whether the pathogen is truly
extracellular (i.e replicates outside a cell but is phagocytosed by macrophage), or is intravesicular
(replicates inside a vesicle in a cell), a peptide form such pathogens is presented on MHC II proteins.
To recognize and respond to exogenous antigens, CD4 T cells (called helper T cells) must use their
TCR to bind antigen peptides presented on MHC II proteins. These helper CD4 T cells can then
differentiate into Th1 cells that help phagocytes destroy the bacteria within them or into Th2 and Tfh
cells that activate B cells.

30

MERP Medical Immunology

Below is a schematic of various scenarios where T cells need to recognize antigen and respond to it.
In each case, MHC is necessary for the T cells to do their job.

MHC Structure
MHC I
Class I MHC is a heterodimer of membranebound alpha chain and non-covalently associated beta
2-microglobulin. The genes for Class I alpha are highly
polymorphic (there are multiple alternative versions of
each gene in a population). The alpha chain has three
domains named a1, a2, and a3 and a region adjoining
alpha3 that anchors it in the plasma membrane. Beta 2microglobulin is encoded by a gene on another
chromosome. Beta 2-microglobulin molecules are noncovalently associated with Class I alpha chain and are
not polymorphic. Class I MHC proteins are expressed on
all nucleated cells. Leukocytes express the highest level
of MHC I and neural cells the least.
Antigen binds between MHC I alpha 1 and alpha
2 domains. The amino acid sequence of these antigen31

MERP Medical Immunology

binding domains varies from allele to allele. Variability is maximized in the amino acids which contact
antigen. MHC I can hold an 8-10 amino acid peptide. The cleft is closed at the ends, limiting the size
of the peptide which can bind in the peptide binding groove. The alpha 1 and alpha 2 domains also
bind TCR. CD8 protein binds MHC I alpha 3 domain, which has a species-specific constant amino
acid sequence. There are three MHC I genes that encode MHC I proteins: these genes are called
HLA-A, HLA-B, HLA-C. Remember that an individual inherits two alleles of each of these genes.
For a particular MHC I to hold a peptide, this peptide needs to have certain anchor residues
that fit into the pockets on MHC I. Anchor residues are amino acids in the peptide that interact with
residues in the MHC binding cleft. For a given MHC allele, these residues must be related - for
example, all hydrophobic or all acidic amino acids. The MHC I peptide is anchored by its carboxy and
amino terminals and two other anchor residues in the middle of the peptide. The amino acid residues
lining the peptide binding sites of MHCs vary among different allelic variants and determine the
identity of the anchor residues that can bind to a particular MHC. For example, dads and moms
allelic variants of HLA-A in an individual will bind peptides with different anchor residues. Keep in
mind that while T cells and B cells each bind a unique epitope of an antigen and are mono-specific,
individual MHC I proteins can present many different peptides (one at a time) provided that these
peptides have the anchor residues with affinity for the peptide binding groove. MHC II proteins also
bind peptides via anchor residues.
Schematic of TCR and MHC binding

Examples of anchor residues

32

MERP Medical Immunology

MHC II
MHC Class II is a non-covalently bound heterodimer
of alpha and beta chains. Peptides bind Class II alpha 1 and
beta 1 domains. These domains vary from one MHC II allele
to the next. In addition to binding antigen, alpha 1 and beta
1 domains also contact the variable region of TCR. CD4
binds to the non-polymprophic region of MHC II. MHC II
peptide-binding site binds longer (13-50 residues) peptides
than MHC I. Peptides which bind to Class II are not
anchored by their amino and carboxyl termini. Class IIbinding peptides contain 2 anchor residues that must be of
certain types to bind a particular allele. MHC II proteins are
found on: B cells, dendritic cells, macrophages and thymic epithelial cells. Three gene loci
encode MHC II : HLA-DP, HLA-DQ, HLA-DR. Since MHC II protein is a heterodimer composed
of two proteins (alpha and beta chains) each of the MHC II loci codes for alpha and beta chain.
The MHC II proteins are coded for by HLA-DP alpha and HLA-DP beta genes, HLA-DQ alpha and
HLA-DQ beta genes and HLA-DR alpha and HLA-DR beta genes. Remember that each individual
inherits two allels of each gene.

MHC Function
MHC I processing (right)
5

1. Newly synthesized,
partly folded MHC I
alpha chain (light blue)
4

binds the chaperone


calnexin (dark blue) in
the lumen of the ER.

3b

Chaperone binding
prevents mis-folding

that otherwise would


occur in the absence of
3a

antigen peptide.

2. When beta 2-microglobulin binds to the MHC I alpha chain, calnexin dissociates.
33

MERP Medical Immunology

3. MHCI/beta 2-microglobulin form a complex with other chaperones such as calreticulin (3a) and
tapasin (3b), and the TAP transporter.
4. When MHC I binds peptide, it is released from theTAP transporter and from other chaperones.
Unbound peptides from the ER are thought to be transported back into the cytoplasm for
reprocessing and retransport.
5. The MHC I -peptide complex is transported through the Golgi (5) to the plasma membrane (6).
MHC I lacking a peptide returns to the cell to be reloaded, it does not remain empty on the
membrane. Althoug MHC pathway has evolved to help us fight infection, in uninfected cells,
membrane MHC I presents self peptides.
Viruses have evolved mechanisms which interfere with TAP function or with transport of Class I MHC
to the plasma membrane which allows them to evade destruction by cytotoxic T cells.

MHC II processing (right)


Exogenous antigens are processed in the endosomal processing pathway. Bacteria, soluble
protein antigens,
phagocytosed antibodycoated viruses and viral
5

pieces released from


infected cells are
4

examples of exogenous
antigens. Additionally,

some bacteria and


2

parasites have actually


adapted to survive and
multiple in the
endosomal vesicles;
these are also

considered exogenous antigens. The endosomes become increasingly acidic as they move away
from the plasma membrane farther into the cytoplasm. Increased acidity activates proteases that cut
the antigen into peptides.
1. During translation MHC II alpha and beta chains are transported into the ER lumen, where they
assemble with another polypeptide, called the Invariant Chain (Ii). CLIP (Class II-associated
invariant chain peptide is) is a short portion of the invariant chain which occupies the peptide
binding site.
34

MERP Medical Immunology

Invariant chain blocks association of MHC II with endogenous peptides


present in the ER lumen.
2. The invariant chain also directs the transfer of Class II MHC through the
trans-Golgi to a specialized vesicular compartment called phagolysosome
(also referred to as MHC class II compartment or MIIC), where the
invariant chain is degraded by the low pH.
3. A chaperone molecule (DM) is involved in loading the MHC class II
molecule with a peptide.
4. In the presence of antigen, MHC IIs groove (which opens up in the
lysosomal compartment due to the low pH and the help of DM) will bind
pieces of bacterial proteins that also got processed in the lysosome. The
low pH displaces CLIP in favor of higher affinity peptides.
5. The complex is moved to the cell surface to be presented to a T cell.
In the absence of infection, APC present MHC II containing self-peptides.

MHC and diversity of antigens being presented


There are two main reasons for why collectively MHC proteins offer diversity for antigen
binding.
1) MHC genes are highly polygenic (multiple gene families encode MHC I and MHC II). MHC I
family contains: 3 genes called HLA-A, B, and C. MHC II family contains 3 gene loci called DP, DQ,
and DR that are arranged in pairs encoding alpha and beta chains of MHC II proteins.
2) MHC genes are highly polymorphic (each gene has alternative alleles in a population). HLAA contains almost 1000 alleles, HLA- B contains ~1500 alleles, HLA-C contains ~500 alleles. There is
also great variation in the number of alpha and beta alleles encoding HLA-DP, HLA-DQ, and HLADR. The existence of three versions of MHC I and MHC II, and the fact that it is likely that an
individual inherits different alleles of each HLA gene from their parents allows for presentation of a
vast number of peptides.

Organization of
HLA locus

35

MERP Medical Immunology

Clinical correlation:
Bare lymphocyte syndrome is a partial or complete deficiency in MHC I or MHC II proteins.
People with bare lymphocyte syndrome have an increased susceptibility to viral and opportunistic
infections. Symptoms range from none to severe combined immune deficiency, depending on the
number of MHC loci that are expressed.
Summary of T cell
and MHC
interactions

Practice questions:
1. Processing and presentation of antigen for presentation on class I MHC differs from the
corresponding process involving class II MHC in that only in processing for class II:
a.
b.
c.
d.
e.

Is the MHC peptide synthesized by the ER.


Does the MHC acquire antigen after it has passed through the ER.
Does microglobulin associate with the MHC in the ER.
Are cytoplasmic proteins passed into phagolysosomes for hydrolysis.
Is tapasin necessary for proper loading of antigen.

2. What is the earliest recombination event in B cell development?


a.
b.
c.
d.
e.

Rearrangement of the kappa chain


Allelic exclusion of the lambda chain
Rearrangement of the heavy chain
Allelic exclusion of the heavy chain
Expression of IgM on the surface of the cell

3. Cytotoxic T cells induced by infection with virus A will kill target cells
a. from the same host infected with any virus.
b. infected by virus A and identical at class I MHC loci of the cytotoxic T cells.
c. infected by virus A and identical at class II MHC loci of the cytotoxic T cells.
d. infected with a different virus and identical at class I MHC loci of the cytotoxic cells.
e. infected with a different virus and identical at class II MHC loci of the cytotoxic cells.

36

MERP Medical Immunology

4. Each of the following statements concerning class II MHC proteins is correct EXCEPT:
a. They are found on the surface of both B and T cells.
b. They have a high degree of polymorphism.
c. They are involved in the presentation of antigen by macrophages.
d. They have a binding site for CD4 proteins.

37

MERP Medical Immunology

T CELL DIFFERENTIATION
Cellular organization of the thymus
The thymus is an
essential organ for the
development of T cells. In
mice and humans with
DiGeorge's syndrome, the
thymus fails to form fully which
results in a lack of T cells and
a severe immunodeficiency.
The figures on the right
demonstrate structural
organization of the thymus
(top) and the stages of T cell development (bottom). Top figure: bone marrow precursors enter at the
outer, sub-capsular region and migrate through the cortex toward the center of the lobe (medulla).
The cortex is composed of a dense reticular network of a highly specialized thymic cortical
epithelial cells (TECs), which appear to be the only cell type capable of mediating positive
selection of thymocytes. In addition to the cortical epithelial cells, there are a few other cells found in
the thymic medulla: a distinct type of medullary epithelial cells, bone marrow-derived dendritic cells
and macrophages. Dendritic cells and
macrophages appear to be the most efficient
at triggering negative selection.
Overview of T cell development
Bottom right figure: T cells normally undergo
development in the thymus. Cells arrive from
the bone marrow via the blood with no
obvious T cell characteristics. They enter
into the lobes and subcapsular area. They
begin to divide and migrate down toward
medulla. The most immature progenitors are
called double negative (DN) because they do not express CD4 or CD8. CD4-CD8- DN T cells also
lack the signaling proteins collectively called CD3. These cells expand in number mainly under the
influence of IL-7 produced in the thymus. They then undergo rearrangement of beta-chain, followed
38

MERP Medical Immunology

by rearrangement of the alpha-chain, and upregulation of CD3, CD4, CD8. These cells are called
double positive (DP) thymocytes and these are the cells that will undergo positive and negative
selection. The DP cells then differentiate into either CD4 or CD8 single positive (SP) T cells. As a
result of these selective events >95% of thymocytes die within the thymus.

Specifics of T-Cell Development: Rearrangement of T Cell Receptor Genes

a.

b.

d.

c.

e.

Stem cell to immature double-positive T cell differentiation (stages a d above)


TCR gene segments rearrange during intrathymic T-cell development. Similar to B cell
development, this is a somatic recombination process that involves the RSS recognition elements
(this follows the 12-23 rule to ensure proper VDJ recombination between heptamer/nonamer regions).
The enzymes that are used in T cell recombination are the same as the ones used in B cell gene
rearrangement (e.g. RAG-1 and RAG-2, TdT). However, the process of T lymphocyte maturation
also has some unique features, primarily related to the specificity of different subsets of T cells for
peptides displayed by different classes of MHC molecules.
TCR gene recombination occurs in a pro-T cell (b in the figure above). V, D and J gene
segments rearrange with the help of RAG and TdT. Transcription and splicing of VDJ exons to
the exon coding for the constant region of the beta chain (C) generates the mRNA for a T cell chain protein. If VDJ recombination is successful in one of the two inherited loci, a TCR -chain
protein is synthesized. It is expressed on the surface in association with an invariant protein called
pre-T (not shown), to form the pre-TCR complex of pre-T cells. The surface expression of the beta
chain marks the pre-T cell stage (c in the figure above). If a complete TCR chain is not produced
in a pro-T cell, that cell dies.
39

MERP Medical Immunology

The pre-TCR complex delivers intracellular signals to promote survival, proliferation, and TCR
gene recombination, and to inhibit VDJ recombination at the second TCR -chain locus (allelic
exclusion).The alpha chain begins to rearrange in the dividing pre-T cell (between stages c and d
in the figure above). The V gene segment rearranges to a J segment to create a functional exon;
then transcription and splicing of this VJ exon to C generates the mRNA for a functional T cell chain protein.
At this point, when the alpha chain is rearranging, the dividing pre-T cells also upregulates
CD4, CD8 and CD3 proteins. The resulting cell, called the double positive T cell thus expresses a
T cell receptor, CD4, CD8 and CD3 (d in the figure on p.40). After rearrangement, each T cell bares
about 30,000 T cell receptors (made of alpha and beta chain) on its surface.
Because the T cell receptor cannot by itself
transmit a signal (TCR has no intracellular domain)
the signaling is performed by a complex of
transmembrane proteins called the CD3 complex
(see Figure on the right). The CD3 complex is a
complex of 3 proteins that are stably associated with
the TCR on the cell surface. All of the CD3
molecules associate with protein tyrosine kinases
following receptor activation and thus signal the cell
interior that the cell has encountered its antigen
(see below).

Immature double-positive to mature singlepositive cell differentiation (stages d e,


Figure p. 39)

The CD4/CD8 decision (lineage


commitment) is determined when T cells are
positively selected by self-MHC. T cells that
have "low affinity" for self MHC presenting selfpeptide are allowed to survive. T cells need to
be able to recognize MHC I or II because in the
periphery this will be the main molecule that the
40

MERP Medical Immunology

TCR will interact with on the target cell. If a T cell does not recognize any of the hosts MHCs it will
not be able to function in the future. So T cells that have rearranged their receptors in such a way that
they do not recognize either MHC I nor MHC II die by apoptosis, a process also referred to as "death
by neglect". This preservation of self MHC-restricted (i.e., useful) T cells is the process of positive
selection. Next, the cells lose either CD4 or CD8 depending on whether their TCR bound to MHC I or
II. During positive selection, the T cells also become functionally segregated: the CD8+ T cells are
capable of becoming CTLs on activation, and the CD4+ cells are helper cells.
T cells that survived during positive
Summary of Positive and Negative
selection

selection will then encounter a wide range of


self-peptides presented by self MHC molecules
expressed on dendritic cells and macrophages
further in the medulla. Those T cells with high
affinity receptors for MHC + self-peptide
undergo clonal deletion (also called negative
selection) through induction of apoptosis. Any
disturbance in this process can lead to the
escape of auto-reactive T-cells that can trigger
autoimmune disease. These selection
pressures ensure that the mature T-cell
population which leaves the thymus can react
with foreign antigen (presented by self MHC I
and self MHC II), but does not react with selfantigen. The summary of positive and negative
selection is illustrated in the figure on the right.

TCR diversity
Overall, the amount of diversity in the TCR is comparable with that of the immunoglobulin genes. The
TCR has a different requirement for diversity than the immunoglobulin. Immunoglobulins have to bind
to a wide variety of ligands. The ligand for a T-cell receptor is always in an MHC molecule. TCRs
have a relatively invariant shape and most of the diversity is focused on the antigenic peptide
occupying the center of the surface in contact with the receptor. Since the TCR does not undergo
somatic hypermutation, most of the diversity in TCR is generated during rearrangement. The limited
number of V region gene segments may generate the diversity needed for MHC recognition, whereas
41

MERP Medical Immunology

the enormous diversity generated at the junctional regions by TdT facilitates recognition of antigen.
One reason that T cells do not undergo somatic hypermutation is that it could be disastrous if a T cell
receptor was to arise in the periphery that reacted with a self-protein. Remember that T cells spend a
long time undergoing thymic education just to avoid these self-reactive problems. Additionally,
somatic hypermutation might prevent the T cell from recognizing self-MHC.

Practice questions:
1.
a.
b.
c.
d.
e.

Which of the following is not expressed by double positive immature T cell in the thymus?
CD3
CD4
CD8
HLA-DP
Beta chain

2. Which of the following molecules is expressed first on a surface of a developing T cell?


a. CD3
b. CD40L
c. CD4
d. CD8
e. Beta chain
3. Which of the following molecules is expressed by a pre-T cell and by an immature double positive
T cell.
a. T cell receptor
b. CD4
c. CD8
d. Beta chain
e. Alpha chain

42

MERP Medical Immunology

LECTURES 5 AND 6: ACTIVATION OF IMMUNE RESPONSE


Learning Objectives:
1. Describe the cells/proteins involved in the activation of CD8 T cells, CD4 T cells and B cells. Be
familiar with the sequence of events that takes place during the activation of immune response.
Describe diapedesis
Describe the differences between the three types of antigen presenting cells
Detail the steps in CD8 response (MHC I, TCR, B7, CD28, CD8, IL2, perforin, granzymes, cell
death)
Detail the steps in Th1 response (CD4, MHC II, B7, TCR, CD28, IL2, IL12, CD40LCD40R,
Interferon gamma, macrophage activation, intravesicular pathogen)
Detail the steps in Th2/Tfh response (CD4, MHC II, B7, TCR, CD28. IL2. IL4, IL4, IL10, B cells,
somatic hypermutation, isotype switching, plasma cells, memory cells, IgM vs. other isotypes)
Detail the steps in inactivation of immune response
Describe the process of peripheral anergy
2. Describe the mechanism behind T cell independent B cell responses.
List the properties of antigens that stimulate these responses
3. Describe the role of haptens in immunity
T cell activation
T cell activation is MHC dependent as illustrated below:
MHC I
Expressed by
Present
Acquires peptide in
Associated with
Encoded by

All nucleated cells

APCs

Intracellular peptides (self and


viral peptides)

Extracellular peptides (engulfed


pathogens)

Rough ER

Vesicles, after fusion with acidic


endosomes

Beta 2 Microglobulin
chain: HLA-A, -B, -C genes
macroglobulin chain:
chromosome 15 (2)

Binds to

MHC II

TCR, and CD8 receptor

HLA-DR, -DP, and DQ genes

TCR, and CD4 receptor

43

MERP Medical Immunology


Summary of T cell differentiation and activation

Immune response activation occurs in secondary lymphoid organs. T cells and B cells enter
secondary lymphoid organs
from the blood stream,
specifically from a region
called the high endothelial
venule (HEV) in a process
called diapedesis (see Figure
right). Diapedesis is mediated
by chemoattraction, rolling adhesion,
tight adhesion and transmigration. The HEVs are located in the T cell zones of lymphoid tissues and
are lined by specialized endothelial cells, which express carbohydrate ligands that bind to L-selectin.
HEVs also display chemokines that are made only in the lymphoid tissues (such as CCR7) that bind
CCR7R. Naive T cells in the blood express L-selectin and engage in L-selectin mediated rolling
interactions with the HEV. This slows T cells down in the bloodstream. CCR7 binds to the CCR7R
and transduces intracellular signals that activate the integrin leukocyte function-associated antigen 1
(LFA-1) on the naive T cell to bind tightly to its ligand, intercellular adhesion molecule 1 (ICAM-1) on
the HEV, resulting in further arrest of the rolling T cells.
Once in the lymph node, nave T cells are not activated until they receive stimulation from an
APC. There are several cells that can function as professional APCs and activate nave T cells.

44

MERP Medical Immunology

These cells include; dendritic cells, macrophages and B cells. The table (below) compares these
three populations of cells.
Antigen is usually encountered on a professional antigen-presenting cell (APC) in a secondary
lymphoid organ. If the TCR recognizes antigen displayed on MHC molecules and also receives a
second costimulatory
signal, the T cell
becomes activated. The
activated cells then
proliferate and undergo
clonal expansion and
differentiation into
effector cells, most of
which are short-lived.
The effector T cells can
move into peripheral
tissues and other organs
to handle pathogen
infection directly, or they
can migrate to germinal
centers to help activate B
cells with specificity for
the same antigen to
secrete antibody. The
CD8+ and CD4+ T-cell
subsets can develop with
different effector functions. By virtue of the different specificities of CD4+ and CD8+ effector T cells,
the immune response can monitor extracellular pathogens (e.g., bacteria and parasites) and
intracellular pathogens respectively.

45

MERP Medical Immunology

Simplified diagram of the types of cell-mediated immune reactions designed to eliminate different
types of microbes. CD4+ helper T cells secrete cytokines that recruit and activate other leukocytes to
phagocytose (ingest) and destroy microbes. CD4 T cells can also recruit B cells to secrete
antibodies. CD8+ cytotoxic T lymphocytes (CTLs) kill any infected cell containing microbial proteins
in the cytosol or nucleus, eliminating cellular reservoirs of infection.
Microbial infections may occur anywhere in the body, and some infectious pathogens are able to infect and
live within host cells. Pathogenic microbes that infect and survive inside host cells include all viruses that
work by infecting phagocytic and nonphagocytic cells and living and replicating in the cytoplasm of these
cells. In addition, many bacteria, fungi, and some protozoa that are ingested by phagocytes resist the killing
mechanisms of these phagocytes and thus survive in the vesicles or in the cytoplasm.
The different classes of T cells differ in the cellular locations of microbes they recognize and in the nature of
the reactions they elicit. In general, CD4+ T cells recognize antigens of microbes in phagocytic vesicles and
secrete cytokines that recruit and activate other leukocytes that kill the microbes, whereas CD8 + cells
recognize antigens of microbes that are present in the cytosol and directly destroy the infected cells.

46

MERP Medical Immunology

CD8 T cell activation


Phases of activation: T cell
responses follow several phases of
activation. As shown in the diagram on the
right, CD8 T cells with particular antigen
specificity are very rare in the periphery.
However when the antigen specific
activation of such T cells occurs, they
expand (II) and their levels increase to a
much higher level than originally found.
However, not all effector cells remain alive
and many activated T cells undergo a
contraction phase (III), where they die by apoptosis. Only a limited number of memory T cells
remain in the body to fight this antigen in the future. Note that the number of antigen specific memory
T cells is still higher than the original number of such T cells (IV).
CD8 T cell activation:
Dendritic cells can be exposed to cytoplasmic endogenous antigen in the tissue either by direct
infection with these antigens, or by phagocytosis and cross-presentation of the antigen on MHC I.
TCR of a nave T cell specific for that antigen binds a peptide from the antigen presented on the MHC
I molecule of the dendritic cell. This interaction provide a primary signal for T cell activation. Part of
this primary signal is the binding of CD8 co-receptor to the non-polymorphic region of MHC I.
A secondary signal (costimulation) is also required for T-cell activation: the binding of the B7
costimulatory molecule on the APC to CD28 on the T cell provides costimulation. The name
costimulator derives from the fact that these molecules provide stimuli to T cells that function
together with stimulation of TCR by MHC/antigen complex. The best-defined costimulators for T cells
are two related proteins called B7-1 (CD80) and B7-2 (CD86), that will collectively be referred as B7
in this book. These proteins are expressed on APCs and their expression is increased when the
APCs encounter microbes. CD28-mediated signaling is essential for the responses of naive T cells; in
the absence of CD28-B7 interactions, antigen recognition by the TCR is insufficient for T cell
activation, and in fact has an opposite effect deactivating the T cell.
After receiving the two signals, the nave CD8 T cells then become activated cytotoxic T cells
(CTLs). During activation the activated T cells begin to secrete IL-2 which they bind in an autocrine
manner and which causes them to proliferate. CTLs begin to synthesize perforin and granzyme. They
then travel from the lymph nodes to the blood via the thoracic duct and exit the blood stream in the
47

MERP Medical Immunology

infected tissue. In the tissue they find the infected cells by recognizing MHC I and the specific peptide
presented on these cells. The secretion of perforin and granzyme directly into the target cells induces
apoptosis in these cells. However, since non-infected cells do not present pieces of the pathogen,
they are not affected by the CTLs.

A.

CD8 response

B.

D.

C.

Induction of CD8 T cell response: (A) Dendritic cells (DC) pick up antigen and present a peptide on MHC
I. These DC travel to the lymph node. (B) Naive CD8+ T cells recognize peptides presented by DC in
peripheral lymphoid organs. The T lymphocytes are stimulated to proliferate and differentiate into effector
cells by 1) TCR/CD8 signaling and 2) CD28 signaling. The T cells respond by producing cytokines, such as
interleukin-2 (IL-2), and expressing receptors for these cytokines, leading to an autocrine pathway of cell
proliferation. Some of the progeny differentiate into effector cells, which serve various functions in (cell
mediated immunity) CMI, and memory cells, which survive for long periods. (C) Effector T cells and other
leukocytes migrate from efferent lymphatic vessels to blood vessels. (D) The T cells exit in peripheral
tissues by binding to endothelial cells that have been activated by cytokines produced in response to
infection in these tissues. (E) CD8+ cytotoxic T lymphocytes (CTLs) kill infected cells.

T cell inactivation (for both CD8 and CD4 effector cells):


Inactivation involved a molecule CTLA-4. In contrast to CD28, CTLA-4 is transiently expressed
only following T cell activation. The signal delivered via CTLA-4 down-regulates T cell function and
inhibits excessive expansion of activated T cells. CTLA-4 knockout mice exhibit massive
lymphoproliferative disorder and spontaneous autoimmune disease that is uniformly fatal by 4-5
weeks. Therefore, CTLA-4 is a negative regulator of T cell function. In numerous animal transplant
48

MERP Medical Immunology

models, treatment with CTLA-4 Ig (soluble CTLA-4 which competes with hosts CD28 for B7 binding),
prolonged graft survival, reduced the lethality of graft-vs.-host disease, and in some cases resulted in
donor-specific tolerance.
In addition to CTLA-4, activated T cells express FasR (CD95). The binding of the inducible
CD95 ligand (CD95L) to CD95 on activated T lymphocytes results in apoptotic cell death. This
activation-induced cell death is implicated in the control of immune cell homeostasis and immune
response termination.
CD4 cell activation
CD4+ T-cells
have been
subdivided
into many
different
subsets on
the basis of
their cytokine
production
and their
functions (Figure right).

Th1
o

These participate in both cell-mediated immunity and antibody-mediated immunity.


They are essential for controlling intravesicular pathogens such as certain bacteria,
e.i., Listeria and Mycobacterium tuberculosis (the bacillus that causes TB). They also
provide cytokine-mediated "help" to cytotoxic T lymphocytes perhaps the body's most
potent weapon against intracellular pathogens. They are the most potent IL-2
producers.

Th2
o

These provide help for B cells and are essential for the production of IgE antibodies and
assist in the production of some subclasses of IgG as well. Antibodies are needed to
control extracellular pathogens which, unlike intracellular parasites, are exposed to
antibodies in blood and other body fluids.

49

MERP Medical Immunology

Tfh
o

These also provide help to B cells enabling them to develop into antibodysecreting plasma cells. This occurs in nests of lymphoid cells called follicles in
the lymph nodes. The most abundant helper T cells there are B-cell helpers
called follicular helper T (Tfh) cells.

Th17
o

These protect surfaces (i.e., skin and lining of the intestine) against extracellular
bacteria. These are characterized by production of IL-17 and recruitment of neutrophils.
Th17 cells may play a role in a range of inflammatory conditions including uveitis,
multiple sclerosis, experimental autoimmune encephalomyelitis, psoriasis, and
rheumatoid arthritis.

In addition, there is another related subset of T cells that dampens rather than promotes immune
responses. These cells are designated 'regulatory T cells or (Treg) and their function is to inhibit
autoimmunity and protect against tissue injury.
Upon activation, CD4 T cells start as a TH0 cell that can develop into Th1, Th2, Th17, Tfh and
other T cells with different functions, these functions are determined by the initial DC and cytokine
interactions. The different types of Th cells are defined by the cytokines they secrete and thus the
responses that they induce are determined by cytokines. Dendritic cells and other cells in the vicinity
of T cell activation initiate and determine the type of CD4 T-cell responses. In general, T helper (Th)
cells are CD4+ cells that help other immune cells perform their functions. Similar to CD8 T cells,
CD4 T cells also need peptide presentation on MHC II and B7 co-timulation in order to be activated.
Generation of Th1 cells (see diagram below)

A. When a dendritic cell encounters an intravesicular antigen it expresses a peptide from that
antigen on MHC II.

B. The DC travels to the lymph node and secrete interleukin-12 (IL-12). Nave CD4 T cells
present in the lymph node need two signals to get activated (1. Triggering of TCR and CD4 by
MHC II, and 2. Triggering of CD28 by B7). DC provide the MHC II/peptide and the B7 needed to
activate nave CD4 T cells in the lymph node.

C. Activated CD4 T cells differentiate into Th1 cells in the presence of IL-12.
Th1 cells return to the tissue and are responsible for the activation of macrophages that express
the antigen recognized by the Th1 cell.

50

MERP Medical Immunology

Macrophages also need two signals to get activated in order to destroy the intravesicular bacteria
that parasitized them. These intravesicular bacteria often have evolved mechanisms to disarm the
vesicles formed by the macrophages.

D. In the tissue, Th1 lymphocytes express CD40L, which engages CD40 on the macrophages,
and the T cells secrete interferon- (IFN-), which binds to IFN- receptors on the macrophages.
This combination of signals activates the macrophages to produce microbicidal substances that
kill the ingested microbes.
o Activated macrophages produce toxic substances such as nitric oxide (NO) and oxygen
radicals which either destroy or help control the bacteria. Th1 cells do not kill the
macrophage.
o Activated macrophages also secrete cytokines that induce inflammation. Tumor
necrosis factor (TNF) is a potent vasodilator, interleukin-1 (IL-1) increases the
expression of adhesion factors on endothelial cells to enable transmigration and
induces fever, IL-6 raises temperature, CXCL8 recruits neutrophils, and IL-12 which in
addition to directing Th1 cell differentiation also recruits and activates NK cells that in
turn strengthen macrophage.

A.

Th1 response (in


the presence of
IL-12)

B.
D.

C.
51

MERP Medical Immunology

o Macrophage activation also leads to the induction of their professional APC properties.

In addition to macrophage activation, cytotoxic T cells also get stimulated by Th1, via IL-2 and
IFN- that Th1 cells secrete.

In addition to helping macrophages fight intra-vesicular cacteraTh1 cells can also modulate some
isotype switching in B cells to isotypes that will help bacteria opsonization (discussed later).

Th2 and Tfh cells


T helper type 2 (Th2) and T follicular helper (Tfh) cells are a distinct lineage of CD4+ effector T cell
that secretes IL-4, IL-5, IL-6, IL-9, IL-10, IL-13, IL-21 as well as other cytokines. These cells are
required for humoral immunity and play an important role in coordinating the immune response to
extracellular pathogens as well as bacterial toxins. Th2 differentiation occurs in the presence of IL-4,
whereas Tfh cells differentiate in the presence of other cytokines. Various functional classes of CD4
helper T cells are formed after they receive two activation signals from the APC and differentiate in
different cytokine environment. Unlike Th1 cells which travel to the tissue to find infected
macrophages, Tfh and Th2 cells remain in the lymph node and travel to the germinal centers.

Th2 (and Tfh) cell activation and B cell stimulation:


In order to get optimally stimulated, each B cell needs to bind antigen via its antibody, receive
cytokine signals from Th cells and interact with CD40L on the helper T cell (via CD40R on the B cell).
Optimal B cell activation results in three population of cells: 1) IgM-producing short-lived plasma cells
2) somatically hypermutated and isotype switched memory cells and 3) somatically hypermutated and
isotype switched long-lived plasma cells. Steps 2 and 3 require germinal center formation. The
antigen which stimulates Th2 (and Tfh) response is usually soluble and extracellular, so that it can
travel to the lymph node and binds to a B cell that expresses antigen specific antibody.

52

MERP Medical Immunology

Steps in B cell activation

A. DC can
phagocytose

Th2 response (in the presence of IL-4)

extracellular antigen
in the tissue, it will

A.

then carry the

B.

antigen from the


tissue to the lymph
node, presenting the

E.

peptide on MHC II.

B. Antigen specific B
C.

cell also
phagocytose the
extracellular antigen
when it travels to the
lymph node, process

D.-Th2

it and present a
peptide on MHC II.

C. The DC can then


activate a CD4 nave T cells using the two signals needed for activation (MHC II/antigen and B7).

D. In the presence of IL-4 (or IL-6, IL-21 and TGF-beta for Tfh) in the lymph node, the CD4 T cell
will differentiate into a Th2 (or Tfh) cell.

E. Activated T helper CD4 (Th2 or Tfh) cells can then bind to the antigen-specific B cell presenting
a piece of antigen on MHC II. Th cells will then activate the B cell which they recognized.

In addition to activating B cells, Th2 cells also stimulate and recruit specialized subsets of
immune cells, such as eosinophils and basophils, to the site of infection or in response to
allergens or toxin leading to tissue eosinophilia and mast cell hyperplasia. This induces
mucus production, goblet cell metaplasia, and airway hyper-responsiveness. Because of
their influence on the production of IgE antibodies and allergic responses, over activation of
Th2 cells appears to be responsible for the exacerbation of allergies.

53

MERP Medical Immunology

B cell differentiation and activation

Isotype switching (right)


Remember that all isotype
information is carried on the
constant portion of the heavy
chain gene. Isotype switching
requires CD40L/CD40R
signaling to induce an enzyme
AID to open up the
heterochromatin of the heavy
chain. Subsequently, the
different cytokines secreted by
the Th2 and Tfh cells will
dictate which isotype needs to
undergo switching. Th2 cells secrete IL-4 to induce IgE production by B cells. Tfh cells secrete IL-6,
IL10, IL-21 to induce switching to isotypes other than IgE. Remember that isotype switching causes
recombination and a permanent DNA loss between the switch
region of the mu region and a switch region of another isotype.

Somatic hypermutation (right)


During isotype switching, the B cells also undergo another
process called somatic hypermutation. In this process
deaminating mutations are introduced into the variable
portions of heavy and light chain genes. As a result, new
amono acids are synthesized in the variable regions.
The daughter B cells with the highest affinity for antigen
will survive and become plasma cells or long-lived
54

MERP Medical Immunology

memory cells. B cells are selected for highest affinity via a process of affinity maturation during
which the selection and survival of B cells with the highest antigen-binding affinity takes place as
antigen levels drop due to antigen clearance. Somatic hypermutation also requires AID enzyme.
Since isotype switching occurs at the same time as somatic hypermutation, resulting B cells will
not only have a high affinity antibody expressed on their membrane but also the antibody will have an
isotype other than IgM. When secreted, these antibodies will have characteristics such as long lifespan, ability to activate macrophages and also to activate complement cascade as well as cross
various tissue barriers. Daughter cells that secrete high affinity, isotype switched antibody serve as
long-lived plasma cells (as opposed to IgM secreting plasma cells which are short-lived). Remember,
that after activation, each B cell will only have antibody with one specificity and one isotype
expressed.
Note that while isotype switching and somatic hypermutation require CD40R engagement on the
B cell, antigen binding and cytokines (without CD40) alone may be enough for IgM secretion. An
CD40R deficiency syndrome, called hyper-IgM syndrome, is characterized by a lack of isotype
switched antibodies but abundance of IgM.
Most of the activated T cells (whether CD8 or CD4) die by apoptosis after they have fought
against the infection. This restores homeostasis to the T-cell pool. A few effector cells mature into
memory T cells, which have the ability to respond faster and more effectively on re-encountering
antigen.
How do dendritic cells know whether to secrete IL-4, IL-12 or other cytokines necessary for the
regulation of Th2, Th1, and Tfh responses, respectively? DC and macrophages express Toll-like
receptors (TLRs) which
recognize structurally
conserved molecules derived
from microbes. These
microbe-derived molecules
are called pathogenassociated molecular patterns
(PAMPs). PAMPs are broadly
shared by pathogens but
distinguishable from host
molecules. Depending on the
TLR triggered during
infection, APCs will secrete a
certain profile of cytokines.
These cytokines then
influence immune response.

55

MERP Medical Immunology

Peripheral tolerance
It may happen that self-reactive T cells make their way out of the thymus and into the periphery.
Therefore and additional mechanism of tolerance induction occurs in the periphery. This is in addition
to central tolerance in the thymus.
There are several ways that peripheral tolerance can be induced:
1) Clonal deletion: a T cell that binds repeatedly (because of high concentration of self-antigen) will
undergo programmed cell death
2) Anergy: anergic T cells recognize self-antigen but remain inactive owing to a lack of the
constimulatory molecules required for activation of the T cells.
3) Active suppression: self-reactive T cells are kept nonfunctional when self antigen is presented at
low levels. The cells reacting at low levels differentiate into regulatory cells, that secrete regulatory
cytokines in order to prevent other cells from reacting to that antigen. This is the most common
mechanism for controlling peripheral T cells.
Anergy induction

T cell independent T cell responses


B cells need at least two signals to initiate activation. For maximal antibody production most B
cells requires T cell help. Such responses are thus called T-cell-dependent responses, and the
antigens that can activate B cells is this manner are called T- cell- dependent antigens. With a Tdependent antigen, the first signal comes from antigen cross linking BCR and the second from the
Th2 or Tfh cell. T-dependent-antigens usually contain protein so that peptides can be presented on B
cells MHC II to Th2 or Tfh cells. These effect T cells then help trigger B cell proliferation and
differentiation into plasma cells and memory cells. As discussed in the previous chapter, when the B
cell presents a peptide on MHC II, the T cell recognizing this peptide will activate the B cell via CD40L
and cytokines. Isotype switching to IgG, IgA, and IgE and memory cell generation occur in response
to such T-dependent antigens. T-dependent responses require that a B cells antibody and theTh2 or
Tfh cell this B cell interacts with recognize epitopes on the same antigen. The T and B cell epitopes
dont necessarily need to be identical; for example, a T cell can respond to an internal viral protein

56

MERP Medical Immunology

presented on MHC II of a B cell, while the B cell presenting to the T cell expresses antibody specific
to viral coat protein.
Some antigens are T-cell-independent, meaning antigen alone delivers both the antigen and
T cell dependent

the second signal to the B cell.

T cell independent

Mice without a thymus (nude or


athymic mice) can respond to
T-independent antigens. Many
bacteria have repeating
carbohydrate epitopes that
stimulate B cells to respond
with IgM synthesis (in the
absence of T cell help, even at
low antigen concentrations).
Most pathogens, however, do
not have many repetitive
epitopes and therefore are not able to cross-link several antibodies on the membrane of a single B
cell.
Unlike T-cell-dependent-B cell activation, T-cell-independent-B-cell activation pathway does
not result in production of memory cells nor appreciable class-switching. The ability of B cells to be
activated without T cell help depends mainly on the nature of the pathogen.

57

MERP Medical Immunology

Haptens:
Haptens are substance that are non-immunogenic but are foreign (or self, in autoimmune
reactions) and can react with an antibody on a B cell. Haptens are small molecules that normally do
not induce an immune response when administered by themselves (they are usually carbohydrates
that cant be
Figure:

presented in

Top. Injection of hapten alone yields


no antibody response in the rabbit.
Collection of rabbit serum and
performing ELISA results in no
fluorescence.

MHC), but can

Bottom. Conjugating the hapten to


a carrier protein, prior to injection,
stimulates an immune response in
the rabbit. This is evidenced by
serum fluorescence seen in ELISA.

coupled to a

induce an
immune
response when

carrier protein.

Concept of hapten - Right bottom figure: Here an antigen is made up of hapten (orange, blue,
green) which is a carbohydrate lacking repetitive epitopes and protein (pink). If the hapten alone is
administered it would not elicit an immune response. Attaching the protein (pink) to the hapten can
convert an otherwise non-immunogenic carbohydrate hapten into a T-dependent antigen. In the figure
above, the carbohydrate-specific B cell binds the antigen via hapten (1) but presents a peptide (not
the hapten) to Tfh cells (2). Tfh cell
then activates the B cell through CD40
and cytokines (3 and 4). B cell then
produces and secretes antibodies
2.
specific for the carbohydrate (5 and 6).
Peptide
1. Binding the
(not
hapten
Cooperation between B cells and T
hapten)
cells allows them to recognize the
same antigen through various epitopes
of that antigen. So while an epitope
recognized by a Tfh or Th2 cell is
constrained to be a peptide, the epitope
recognized by the B cell can be protein,
carbohydrate or lipid. These epitopes
are derived from the same antigen.
This linked recognition of antigen by
T and B cells is important in
manipulating haptens in order to make them immunogenic.
A well-known example of a hapten is urushiol, a toxin found in poison ivy. When absorbed
through the skin from a poison ivy plant, urushiol reacts with skin proteins to form hapten conjugates.
Usually, the first exposure causes only sensitization, in which there is a proliferation of effector Tcells. After a second exposure the effector T cells can generate a strong immune reaction, producing
the typical blisters of poison ivy exposure.
58

MERP Medical Immunology

LECTURE 7:
FUNCTIONS OF ANTIBODIES, NATURAL KILLER CELLS, BLOOD TYPES, ANTIBODY PROFILE
IN INFANTS
Learning objectives:
1. Describe the different roles of antibodies in fighting different infections, and know how the difference in
isotypes relates to antibody functions (neutralization, opsonization, complement activation).
2. Describe the role of complement in immune response.
Differentiate Classical vs Alternative cascade
Describe the result of deficiency in MAC
Describe the result of deficiency in C3
Draw out the basics of complement cascade.
3. Describe the mechanism of Natural Killer Cell induced cell death.
Describe NK mechanism of killing through ADCC
Describe NK mechanism of killing through lack of MHC I
4. Describe the importance of blood types in transfusion reactions. Detail the principle behind the
Rh factor and hemolytic disease of the newborn.
List the various blood types
Describe the relationship between blood types and preformed antibodies
Describe the conditions in which one would see agglutination reactions
Be able to interpret the results of testing for blood type
Describe the characteristics of universal donor vs. universal recipient
5. List the antibodies produced by newborns and the course of antibody development.
Determine when children are protected against infections and why
Determine when children are most susceptible to infections and why
Antibody functions: So far we have discussed in detail what antigen binding is like, however it is
also important to understand what an antibody can do to the antigen it bound.
1. Neutralization: Antibodies can
neutralize viral infectivity in a
number of ways. They may
interfere with virion binding to
receptors, block uptake into
cells and/or prevent uncoating
of the genomes in endosomes.

Neutralization

59

MERP Medical Immunology

2. Opsonization: process by which a pathogen is marked for ingestion and is destroyed by a


phagocyte.
Opsonization
Opsonization
involves the
binding of an
opsonin, (i.e.,
antibody), to a
receptor on the
pathogen's cell
membrane.
After opsonin
binds to the
membrane,
Opsonization
phagocytes are attracted to the pathogen.
3. Activation of complement:
a. Inflammation
b. Opsonization
c. Lysis of bacteria

Complement
There are two major pathways that can initiate the complement cascade. These pathways are
called the Classical pathway and the
Alternative pathway. Although the Alternative
pathway most likely began earlier during
evolution, it was discovered after the
Classical pathway and therefore received the
name Alternative. Both pathways lead to
inflammation, opsonization of the pathogen
and direct lysis of the pathogen.

60

MERP Medical Immunology

Classical pathway
The classical pathway of complement is triggered by binding of IgM or most IgG isotypes to a
pathogen (right figure below). Early events involve a series of proteolytic steps. A large precursor
protein is cleaved to yield one large active fragment which usually binds to the surface of the
pathogen and a small peptide fragment that does not bind the membrane and often mediates
inflammatory responses.

Steps in classical pathway


activation (Figure right)

The first component is


C1. It is activated when

antibodies (IgM or IgG)


+

are bound to a pathogen.


C1 has three members:
C1q, C1r and C1s.

3
2

Binding activates an
enzymatic activity in C1r,
which causes C1s to act as an active serine protease. (1)

The activated C1 now acts on the next 2 components of the pathway: C4 and C2. First, C1
cleaves C4 into C4b and C4a (2). C4b binds covalently to the surface of the pathogen.

This bound molecule of C4b now binds one molecule of C2, making it susceptible to cleavage by
C1. (3)

C2 is cleaved into C2b and to C2a. (4)

C4b and C2b remain on the surface of the pathogen and function as C3 convertase (aka C4b2b).

C3 convertase (C4b2b) cleaves C3 into C3b and C3a. (5)


C3 cleavage results in the deposition of many molecules of C3b on the pathogen surface, which
can be recognized by complement receptors on the phagocytes and cause engulfment.

Binding of C3b to C4b2b generates C5 convertase (aka C4b2b3b). (6)

C5 is now cleaved to generate C5b, which forms part of the membrane attack complex and C5a
which is a mediator of inflammation. (7)

C5b, C6, C7, C8 and several C9s together form the membrane attack complex (MAC) which
punctures a hole in the pathogen. (8) This occurs because C8 can change conformation to expose
hydrophobic domains that insert in the lipid bilayer and C9s can polymerize to form a pore in the
membrane.
61

MERP Medical Immunology

Peptides C4a, C3a and C5a are local mediators of inflammation.

Steps in the alternative pathway


activation (Figure right)

The alternative pathway (right


5

figure) can be activated in the


absence of antibody and is

considered to be part of the

innate response.

The first fragment bound on

3
1

the surface of the pathogen is


C3b (the first one bound in the
classical pathway is C4b).

C3 can be spontaneously cleaved to generate C3a and C3b. (1)


Normally, this complex is rapidly inactivated. However, some C3b can become covalently
attached to the surface of the pathogen and be stabilized.

Next, factor B binds to C3b. (2)

This binding renders factor B susceptible to cleavage by Factor D, (3) yielding Ba and Bb. Bb now
forms C3bBb on the surface of the pathogen and Ba is an inflammatory mediator.

The complex C3bBb is the C3 convertase of the alternative pathway and can cleave many
molecules of C3. (4) When C3b generated by C3 convertase binds to a pathogen it can initiate the
alternative pathway, amplify the classical pathway or function as an opsonin. C3 convertase also
generates more C3a.

C3b can also bind to pre-formed C3bBb to generate C3bBb3b. This is the C5 convertase and it
cleaves C5 into C5a and C5b. (5)

MAC is formed exactly the same as for the classical pathway. (6)

Note that the cleavage of complement component C3 is central to both pathways of complement
activation. From this point onward, these pathways converge and C3b is one of the major
opsonins, binding to complement receptors on phagocytes.

Natural killer cells


Natural killer cells provide early defenses against intracellular infection. NK cells kill virally
infected and tumor cells while sparing normal self-cells. There are two distinct ways in which NK cells
can kill their target.
62

MERP Medical Immunology

1) NK cells can destroy


virally infected cells with the aid

Antibody-dependent cytotoxicity

of antibodies in a process called


antibody dependent cellular
cytotoxicity (ADCC) (Figure
right).

In ADCC the target cell


expressing an immunogenic
molecule is bound by
antibodies of the IgG isotype.

NK cells have Fc receptors for IgG, so they can recognize antibody coated target.

Once the NK cell binds to the IgGs Fc portion with its Fc receptor, the NK cell is triggered to
secrete perforin and granzyme to kill the target cell.

NK cells can also cause apoptosis in the target cell by engaging the cells FasR with NKexpressed FasL.

Macrophages also have Fc gamma receptors and so they too can kill IgG coated targets.
However, unlike NK cells, macrophages kill the target by phagocytosis and degradation of the
target in the vesicles (not shown).
2) NK cells can cause death in target cells that express a lack of (or have a limited amount of)

MHC I expression (see Figure below, right).

63

MERP Medical Immunology

This ability is conferred upon the NK cell by its expression of killer cell inhibitory receptors (KIRs)
that interact with class I MHC molecules on normal cells.

When MHC I is present on the target cell, the


NK cell is inactivated and doesnt kill the
target (top panel).

A lack of MHC I on a cell shuts down the


negative feedback via KIR, resulting in a
target cell death.

One of the most important roles for these


cells is in the early phases of infection with
intracellular pathogens, such as some viruses
and bacteria.

Many viruses have the ability to downregulate


MHC I in the host cell. A benefit of
downregulating MHCI is that the infected cell
can no longer be recognized by CTL, and
viral replication can proceed unchecked.

However in this scenario an NK cell can destroy the target because the target cell lacks MHC I
expression and the NK cells function is to detect and get rid of such cells.

In addition to recognizing the lack of MHC I, NK cells may also be able to recognize changes in
cell surface glycoproteins induced by viral or bacterial infection.

NK cells also secrete type I IFNs which inhibit viral replication during the early phases of infection.
NK cell activity can is increased by IL-12, which is also produced by phagocytes upon contact with
pathogens.

64

MERP Medical Immunology

Blood types
The ABO blood group system is the most important blood type system (or blood group system) in
human blood transfusion. There are two antigens and two
Phenotype

Genotype

Genotype

Phenotype

antibodies that are mostly responsible for rejection in the


ABO system. The two different types of antigens (also
called agglutinogens and isoantigens) are called type "A"
and type "B". The A and B alleles code for enzymes that
produce the type A and B sugar antigens respectively. A
third version of this gene, the O allele, codes for a protein
that is not functional and does not produce antigenic
surface molecules. Each biological parent donates one of
their two ABO alleles to their child, and so a child inherits
two alleles of the ABO antigens, one from each parent
(see figure, left). The genes coding for ABO antigens are
expressed co-dominantly.

Although a blood test


can tell you which antigens
are expressed and thus your
blood type phenotype, it is
not possible to determine the
exact genotype from a blood
test unless the person is AB
or O. For example if a blood
test determines that an
individual is Type A, it will not
tell you whether his/her
genotype is AA or AO (both of which yield Type A blood. People with type A blood develop antibodies
to type B (called anti-B Ab isoantobodies). People with type B blood develop antibodies to A (anti-A
Ab isoantobodies). People with type O blood develop both anti-A and anti-B Abs and people with type
AB blood lack both types of Abs. These antibodies are preformed without a known sensitization
reaction and are of IgM isotype. People do not develop Abs to their own blood group due to negative
selection of self-reactive B cells. There are various theories for the unusual nature of these preformed
Abs but no one knows for certain when or how they are formed. The figure above illustrates the
antigen and antibodies that define the ABO system.
65

MERP Medical Immunology

Blood transfusion
When a transfusion of blood is required, it is necessary to make
sure that the recipient does not have antibodies that will attack the
donors red blood. Much of the routine work of a blood bank involves
testing blood from both donors and recipients to ensure that every
individual recipient is given blood that is compatible and is as safe as possible. This is done through
testing the recipients serum and the donors RBC. If the blood is compatible, no agglutination
occurs (see a, figure above). If the blood is incompatible, a visible agglutination forms (b, figure
above).
Incompatible blood between donor and recipient may lead to acute hemolytic reaction, RBC
destruction, lack of blood supply to the kidney, renal failure and shock. Antibodies that attack RBCs of
the donor are of IgM isotype and bind components of the complement system possibly leading to
massive hemolysis. Therefore a Type A individual can only donate to Type A and AB, Type B can
donate to B and AB, AB can only donate to AB, and Type O can donate to any type blood individual.
RBC compatibility can be determined by a series of agglutination reactions between the RBCs of a
potential donor and the serum of the recipient. A postive aggutination reaction (which would lead to a
physiological rejection) occurs when the donor and the recipient are incompantible, as for example a
donor of Type A and recipient of Type B (below left). Based on these reactions a compatibilty table of
donor/recipient pairs can be made (below right).
Recipient
Group A

Group B Group AB

Recipient
Group O

Group A

66

Group B

Group AB Group O

MERP Medical Immunology

Rh incompatibility Similar to the ABO group system, another inherited factor, called the Rhesus
factor (Rh) can give rise to blood group incompatibility between individuals. Unlike A and B antigens
of ABO group, Rh factor is a protein, not a sugar, expressed by red blood cells. Therefore Rh factor is
directly encoded by a gene. Also, unlike antibodies to the ABO groups, Rh antibodies are not
preformed, but are only produced after an exposure to the Rh factor.
dad

mom

Anti_Rh Ab

A.

B.

C.

D.

Figure above:

Rh incompatibility is a condition that develops when an Rh-negative woman conceives an Rhpositive baby (A first baby). This can only occur when the woman is Rh negative but the father is
Rh positive. Incompatibility will occur in this situation only if the Rh negative mom conceives an Rh
positive fetus. An Rh negative fetus will not pose a problem.

During pregnancy, and more commonly during delivery (or miscarriage), red blood cells from the
unborn baby can cross into the mother's bloodstream through the placenta (B).

If the mother is Rh-negative, her immune system treats Rh-positive fetal cells as if they were a
foreign substance and makes antibodies against the fetal blood cells (C).

These anti-Rh antibodies are of IgG isotype and have the potential to cross the placenta. This is a
problem for a second Rh- positive fetus, rather than the first because by the time the IgG against
the Rh is made, the first baby is delivered. In the second or subsequent pregnancies, the IgG from
the mother that crossed the placenta to the fetus can cause red blood cells to be broken down (D).
Mating combinations between an Rh+ father and Rh- mother

Figure right: The inheritance of Rh trait can be predicted by a simple

67

Rh +

Rh -

Rh +

Rh -

conceptual model in which there are two alleles for this trait, D and d.
Individuals who are homozygous dominant (DD) or heterozygous (Dd)
are both phenotypically Rh+. Those who are homozygous recessive
(dd) are phenotypically Rh-. Since an Rh+ father can have either a DD
or Dd genotype, there are 2 mating combinations possible with
differing risks as shown below. Although there is a chance that an Rh+
father and Rh- mother conceive an Rh- child (mating combination 2), if
the father is Rh+ and the mother is Rh-, doctors assume that mating
combination 2 is possible and that there may be an incompatibility
problem and act accordingly.

Mating combination 1

Mating combination 2

MERP Medical Immunology

During hemolytic disease of the newborn (which can happen with Rh incompatibility), infants
RBCs are destroyed and they release bilirubin, which causes an infant to become yellow (jaundiced).
The level of bilirubin in the infant's bloodstream may range from mild to dangerously high. After birth,
the infant may have yellowing of the skin and whites of the eyes (jaundice), anemia and low muscle
tone (hypotonia) as well as lethargy. Occasionally Rh incompatibility can result in hydrops fetalis (also
called erythroblostis fetalis), an abnormal collection of fluid in at least two different fetal
compartments. This can lead to swelling, hemolytic abnormalities and miscarriage.
Currently the treatment that prevents the development of Rh antibodies in the mother is
administration of immune globulins called RhoGam. These immunoglobulins are against the Rh factor
and are IgG isotypes, but they are modified so that they themselves cant cross the placenta back to
the fetus. RhoGam is ready to neutralize the Rh positive RBCs of the baby if/when they cross to
maternal circulation and thus these fetal Rh+ RBCs cant sensitize maternal B cells specific for the Rh
factor.
Antibody profile in infants
A baby's immune
system is not fully
developed until he/she is
about six months-old. In the
meantime, pregnant
mothers pass
immunoglobulin antibodies
from their bloodstream,
through the placenta to the
fetus.
IgG is the only antibody that crosses the placenta to the fetus during pregnancy. IgG
antibodies are the smallest but most abundant making up 75-80% of all the antibodies in the body.
These antibodies help protect the fetus from developing an infection inside the womb. Immediately
after birth, the newborn has high levels of the mother's antibodies in the bloodstream. This is called
passive immunity because the mother is "passing" her antibodies to her child. This helps prevent the
baby from developing diseases and infections.
During the next several months, maternal IgGs are steadily degraded in the childs
bloodstream. When healthy babies are about two to three months old, the baby will start to gain the
ability to produce his/her own IgGs. Between 3 and 6 months, the baby will experience the body's
68

MERP Medical Immunology

natural low point of antibodies in the bloodstream. This is because the maternal antibodies have
decreased, and young children, who are making antibodies for the first time, produce them at a much
slower rate than adults.
The IgMs, however, can be produced by the baby even in utero. By the time the baby is born
his/her ability to secrete IgM is about 50% that of an adult. By 6 months the baby has the same
potential as an adult to produce IgMs. However, since these antibodes have low affinity, they do not
offer as much protection as IgGs.
Practice questions:
1. Which immune cell utilizes a pore-forming molecules to damage the target cell membrane?
a. NK cell
b. Th2 cell
c. Th1 cell
d. B cell
e. Red blood cell
2. Serum drawn from a three-day-old infant would contain which of the following?
a. Infant IgG2
b. Infant IgA
c. Maternal IgM
d. Maternal IgG1
e. Maternal IgE
3. Which pair of molecules are both opsonins?
a. IgG1 and C5a
b. IgA1 and IgG2
c. IgE and IgG4
d. IgG3 and IgM
e. IgG4 and C3b
4. A CD8+ T cell binds to an antigen-MHC1 complex on a dendritic cell. The CD28-B7 interaction
happens concomitantly, but no IL-2 is present yet. What is currently happening?
a. The CD8 T cell is activated only
b. Clonal expansion of the T cell only
c. Clonal expansion and activation of the T cell
d. Perforins and Granzymes are released into the lymph node
5. A patients immune response to a pathogen shifts from primarily IL-12 mediated to IL-4 mediated.
What can be said of the subsequent change in the immune response?
a. The TH1 response increases
b. Macrophage stimulation increases
c. CD8 effects are enhanced
69

MERP Medical Immunology

d. B cell stimulation increases


6. What step of the classical complement pathway will fail if the function of C3 convertase is blocked?
a. C1 will not be activated
b. C2 will not be split into C2a and C2b
c. C4 will not be split into C4a and C4b
d. Phagocytosis will be diminished
7. Which cell will be spared by a Natural Killer cell?
a. cell expressing MHCII only
b. cell expressing MHCI only
c. cell coated with IgG
d. cell expressing Fc gamma receptor and no MHC I
8. A worried mother of a 3 month-old baby comes to your clinic with concerns that her baby has had a
runny nose. You draw an antibody titer and find high levels of IgM and low levels of IgG. What can we
say about this childs immune system.
a. He has a congenital IgG deficiency
b. His immune system is normal
c. He has an over-abundance of IgM
d. He is unable to isotype switch from IgM to IgG
9. You are on NICU service in your intern year when you are presented with a severely jaundiced
neonate. Upon looking into the history, you find that the mother declined RhoGam therapy due to her
religious beliefs. This is her second baby. What can be said about the maternal and fetal blood types?
Mother
a. B+
b. O+
c. ABd. A-

Fetus
A+
BA+
AB-

10. The deletion of which of the following molecules would cause over-proliferation of T cells?
a. B7
b. TCR
c. MHC
d. Fas
e. CD40
11. Which of the following collectively describes the antibodies found in a neonate that is nursed?
a. Same idiotype, different isotypes
b. Same isotype, different idiotypes
c. Same idiotype, same isotype
d. Different idiotypes, different isotypes
e. Different allotypes, same idiotype
70

MERP Medical Immunology

12. Which of the following processes changes a B cells idiotype and occurs in the lymph node?
a. Allelic exclusion
b. Isotype switching
c. RNA splicing
d. Somatic hypermutation
e. Somatic recombination
13. Which of the following DNA sequences would most likely to occur in the heavy chain gene in an
immature B cell?
a. V1, V2Vn; D1, D2,Dn; J1, J2.J6; mu, delta, gamma3
b. V1, V2, D2, J5, J6; mu, delta, gamma3.
c. V1, D4, J2; gamma3, gamma1
d. V1, V2Vn; J1, J2.J6; kappa
e. V1, J3, lambda
14. You have a patient with a rare genetic disease that prevents their lysosomes from reaching their
normal, low pH. What will you find bound to their MHC II proteins?
a. tapasin
b. calcineurin
c. bacterial antigen
d. CLIP protein
15. You see a patient in the clinic with advanced AIDS. His T cell count is 40 (extremely low), but you
notice that the surviving population is almost all Th1-type cells. What immune function will be the
most preserved?
a. Infected cell apoptosis
b. Macrophage activation to kill pathogens
c. B cell antibody production

71

MERP Medical Immunology

LECTURE 8: HYPERSENSITIVITY REACTIONS


Learning Objectives:
Differentiate between the mechanisms of the four hypersensitivity reactions.
Describe the role of sensitization in hypersensitivity
List the steps in IgE production in Type I hypersensitivity
Differentiate early vs. late phase in Type I hypersensitivity
Describe chronic desensitization
Describe the role of ADCC in Type II hypersensitivity
List examples of foreign antigens versus selfantigens in Type II hypersensitivity
Draw the mechanism of druginduced hypersensitivity
List examples of Type II hypersensitivity (know disease mechanism of blood transfusion reaction,
hemolytic disease of the newborn, hyperacute graft rejection)
List examples of foreign antigen verses self-antigen induced Type III hypersensitivity
Explain the mechanism of disease in Arthus reaction, serum sickness, rheumatoid arthritis, SLE,
Farmers lungs
Describe Type IV mechanism in detail
List examples of foreign antigen verses self-antigen induced Type IV hypersensitivity
Explain the mechanism of disease in contact dermatitis, poison ivy, PPD test, diabetes Type I, graftvs-host and

hostvs.graft, celiac disease, multiple sclerosis


Hypersensitivity results from the damage done to the body by an immune response. Immune
responses can damage the body during antigen removal, causing swelling and pain from

72

MERP Medical Immunology

inflammation, or during lysis of virally-infected cells by cytotoxic T cells. Occasionally, when the
damage is too great, hypersensitivity can become life-threatening.
Hypersensitivities are classified into four types based on the mechanism of tissue damage
(Figure p. 69 above). Most of what we call "allergy" is Type I hypersensitivity, in which IgE is
produced in response to an antigen called an allergen. However IgG can also mediate tissue
damage, as seen in Type II and Type III hypersensitivities. Additionally, T cells are implicated in Type
IV hypersensitivity.

TYPE I HYPERSENSITIVITY
Type I (immediate) hypersensitivity is mediated by IgE and mast cells. Mast cells are
commonly found at sites in the body that are exposed to the external environment, such as the skin.
In these locations, they are found in close proximity to blood vessels, where they can regulate
vascular permeability and effector-cell recruitment. Although they do not have direct cellcell contact
with local populations of antigen-presenting cells (such as the Langerhans cells in the skin), mast
cells can modulate the behavior of these and other neighboring effector cells through the release of
mediators.

73

MERP Medical Immunology

IgE-mediated allergic
reactions/syndromes include: hay fever,
skin inflammation (urticaria), food
allergies, asthma, and systemic
anaphylaxis (Table, right). One in five
people in developed countries are
thought to have some form of Type I
hypersensitivity.
The risk of developing a Type I
hypersensitivity (also called atopy) is
linked to family history and IgE levels. A
child's risk of becoming atopic is
greater if s/he has parents who are
atopic. People with higher IgE levels
tend to be atopic more often than
people with lower IgE levels, although
the linkage is not absolute.
It is not clear what makes certain
molecules allergens for atopic people.
Allergens fall into groups of molecules
including: grasses, pollens, animal and
food products that usually reach mucosal surfaces at very low doses. They are usually proteins,
since only protein molecules can be presented to T cells and elicit T cell help which is necessary to
ensure IgE production by B cells.
Several of these allergens are
proteases. In addition to proteins,
haptens have also been
associated with Type I
hypersensitivity; this occurs when
the haptens conjugate a soluble
protein and thus become
immunogenic. The Figure on the
right illustrates the effects that
mast cells in Type I
hypersensitivity have on various
organs.
The main symptoms
associated with Type I
hypersensitivity are due to the
release of toxic mediators
synthesized by mast cells (Figure,
right). These mediators cause
inflammation and tissue damage the final effect depends on the tissue. Examples of the effects that
the release of toxic mediators have on various target organs is shown in the figure above.

74

MERP Medical Immunology

The mechanism of Type I Hypersensitivity


In this response, exposure to allergens generally occurs at mucosal membranes in the
respiratory and digestive tracts. Antigen dose is generally low, favoring activation of Th2 responses.
The allergen is usually low molecular weight and very soluble, so it can diffuse through the mucus
membrane.
Sensitization: Figure on the right:

Similar to the initiation of a Th2 response, the allergen in Type I hypersensitivity is first introduced
through mucosal or skin barrier into
the tissue (A).

Sensitization

A.

The DC uptake and present a peptide


portion of the allergen on MHC II.
This DC can then activate an
allergen-specific nave CD4 T cell
(B). The cytokine balance during

C.

activation favors Th2 cell

B.

differentiation (D).

E.

The soluble, extracellular allergen

F.

D.

also travels unprocessed to the


lymph nodes, where antigen-specific
B cells can bind to it via their antibody. The B cells then process and present epitopes from the
antigen on the MHC II (C).

The allergen-specific Th2 cell can then activate the allergen-specific B cell which is presenting the
peptide from the allergen to the Th2 cell (D). The same signals are necessary for the B cells to get
activated as discussed before. The prevalence of IL-4 favors not only Th2 differentiation but also
induces B cells to isotype switch to IgE. Note that the specific epitope recognized by the B cell
does not have to be the same as the epitope presented on MHC II of the B cell. For example, the
B cell could recognize a surface carbohydrate molecule on the antigen but present a peptide from
within the antigen.

The terminally differentiated plasma cells secrete IgE (E).

IgE then enters the bloodstream and binds to the Fc epsilon receptors (FcRI) expressed by mast
cells lining mucosal surfaces (F). The IgE bound to mast cells remains after the antigen has been
cleared from the body.

75

MERP Medical Immunology

Sensitization results in IgE binding to mast cells in smooth muscle, blood vessels, mucosal linings
and connective tissues around the respiratory and digestive tracts. Sensitization usually does not
result in symptoms.

Activation: After sensitization, a subsequent exposure to the same allergen cross-links the mast cellbound IgE molecules, activates the FcRIs and triggers the release of various pharmacologically
active substances. Mast cell degranulation is preceded by increased Ca++ influx.

Activation

Activation phase has two sub-phases: Immediate/Early and Late

Immediate/Early-phase (occurs within seconds)

Preformed mast cell mediators (such as histamine, heparin, and proteolytic enzymes) stored in
cytoplasmic granules are rapidly released.

Proteolytic enzymes break down tissue matrix proteins.

Histamine causes respiratory smooth muscle contraction, increased mucus release,


vasodilatation, and increased capillary permeability.

Increased blood flow and fluid release at mucus membranes and into tissues washes away the
antigen.

In the respiratory tract, mucus production, vasodilation and smooth muscle contraction
leads to runny nose, watery eyes, sneezing, coughing, sinus congestion and constricted
airways.
76

MERP Medical Immunology

In the gastrointestinal tract, smooth muscle contraction and fluid release cause
cramping, diarrhea, and vomiting.

In the skin, localized swelling causes hives (urticaria).

Late-phase response (follows hours after the initial reaction)

Leukocytes, such as eosinophils and neutrophils are attracted to the inflammation.

More cytokines are released

White blood cell production is stimulated in the marrow

Testing for Type I hypersensitivity


Skin testing for allergies is used to identify the substances that trigger allergy symptoms. It is
performed by applying a very small amount of an allergen to your skin dermis, and then evaluating
the skin's reaction. If an individual has previously been exposed to an allergen and has mounted a
Type I reaction, he will have a built-up of mast cell-bound IgEs in his skin. Upon exposure to antigen,
these IgEs will cause the mast cell to degranulate locally and produce a swelling as a result (see
figures below).

77

MERP Medical Immunology

Treatment/Management of Type I hypersensitivity:


Drugs: Antihistamines, Corticosteroids (anti-inflammatory), Cromolyn sodium (stabilizes mast cells),
Epinephrine (blood vessel constriction)
Type I desensitization
People with Type I allergy may receive chronic desensitization through injections of allergen
beginning with very low doses
that are increased over many
months. Desensitization is not
effective for every allergen or
for every individual, and its
mechanism of action is
unclear. It is thought that the
effectiveness of this
procedure results from the
induction of high levels of
IgG antibodies, which can
prevent allergic reactions by
competing for the allergen
and preventing it from reaching mast-cell bound IgE, or that the treatment induces suppressor T
cells (probably Th1 cells) which block B cell activation and IgE synthesis in response to the allergen.

78

MERP Medical Immunology

TYPE II HYPERSENSITIVITY
Type II reactions occur as a result of immune mediators responding to an antibody bound to
cell surface antigen. The cells with such antigens can be extrinsic (foreign) or intrinsic (hostderived). The antibody produced is usually IgM and/or IgG. The results of binding of these classes of
antibodies to the cell surface are shown in the figure below:
1. Complement activation
Classical complement activation
by IgG causes the release of

2.

inflammation-promoting
anaphylatoxins and may lead to

1.

formation of the membrane attack


complex (MAC) and lysis of the
antibody-coated cell.
2. Antibody-dependent
cell-mediated cytotoxicity
(ADCC)
ADCC is a mechanism of immune
defense whereby an effector cell

The effects of Type II hypersensitivity

of the immune system actively lyses a target cell, whose membrane-surface antigens have been
bound by specific antibodies.The most common cells participating in ADCC are natural killer (NK)
cells and macrophages. NK cells and macrophages express CD16, which is an Fcgamma receptor
(called FcgammaR1). This receptor recognizes, and binds to the Fc portion of an antibody, such as
IgG. The IgGs, in turn, are bound via their idiotypes to the surface of a pathogen-infected target cell.
When NK cells FcgammaR1 is triggered, the NK cell releases perforin and granzymes to kill the
target cell. Macrophages also express FcgammaR1 and can also destroy the antibody coated target
cell. However they mediate target cell death through phagocytosis.

Clinical examples of Type II hypersensitivities:


There are two categories of antigens (foreign and self) that can induce Type II
hypersensitivity:

79

MERP Medical Immunology

Examples of foreign antigen-induced Type II hypersensitivity:


1. Blood transfusion can also result in Type II Hypersensitivity to blood group antigens such as
A, B and Rh. Remember that in blood transfusion, if the blood types are not matched, the
recipients IgM may bind to donor red blood cells. This causes complement activation and
destruction of donor RBCs in the recipient.
2. Drugs like aspirin and penicillin, which often complex with erythrocyte membrane
proteins, may induce the host to synthesize IgG antibodies which then bind the drug on the
drug-coated erythrocytes and damage them.
3. Hyperacute graft rejection, a rejection in which preformed antibodies to blood group antigens
(ABO/Rh system) or transplantation antigens (MHC system) can cause immediate, severe, and
non-reversible damage to the graft. Since the destruction is mediated by antibodies bound to
the donor tissue, this reaction is considered Type II.
Host Ab

kidney

kidney

Examples of autoantigen-induced Type II hypersensitivity:


Antibodies are produced to self-membrane proteins.
1. Myasthenia gravis (MG) This disease is caused by the
production of antibodies to the nicotinic acetylcholine
receptors on muscle cells. This disease affects mainly
somatic nervous system which controls skeletal muscles.
For example, oculomotor nerve supplies both the muscles
that move the eyelid (e.g. levator palpebrae superioris) and
also carries parasympathetic innervation of pupil. If the
entire oculomotor nerve is damaged, the pupil dilates and
the eyelid droops. However, in MG, levator palpebrae
superioris cant respond to the stimulation by the alphamotor neuron, whereas the parasympathetic nerves function is unaffected. The result is: the
eyelid droops but the pupil constricts normally. Other symptoms of MG include difficulty talking,
difficulty climbing stairs and lifting objects, difficulty swallowing, and difficulty maintaining
steady gaze.

80

MERP Medical Immunology

2. Graves' disease a disease resulting from the production of antibodies to thyroid stimulating
hormone (TSH). TSH is secreted by the pituitary
gland in response to thyrotropin releasing hormone
(TRH). TSH receptors are located on the thyroid
follicular cells that produce the thyroid hormones T3
and T4. Normally, TSH acts on the TSH receptor to
stimulate T3 and T4 production by the thyroid gland.
Then, there is a negative feedback loop, where the
T3/T4 hormones act on pituitary gland to block TSH
production.
TSH antibodies are agonists, they act like
TSH hormone and result in overproduction of thyroid
hormone T3 and T4. T3 and T4 normally work on
the pituitary gland to inhibit the production of TSH;
however T3 and T4 cannot prevent the TSH
antibody from binding to the thyroid cells. This
causes the clinical symptoms of hyperthyroidism,
and the enlargement of the thyroid gland (goiter).
Other symptoms include anxiety, irritability, fatigue,
rapid or irregular heartbeat, increase in perspiration, sensitivity to heat, weight loss, and
bulging eyes (Graves' ophthalmopathy).
3. Autoimmune hemolytic anemia Hemolytic anemia (HA) occurs when the bone marrow is
unable to increase production of red blood cells (RBCs) to make up for the premature
destruction of RBCs. In the autoimmune type of HA, antibody against the erythrocyte
membrane protein is produced. Some of the symptoms include fatigue, shortness of breath,
and rapid heartbeat. Anemia also causes jaundice because unconjugated bilirubin is released
from the dying RBCs. Bilirubin, a breakdown product of hemoglobin, accumulates in the
circulation and in the tissues causing yellowing. Unconjugated bilirubin in the plasma is
conjugated by the liver and is returned back to the blood. Excess conjugated bilirubin present
in the urine can give urine an unusually dark color.

81

MERP Medical Immunology

TYPE III HYPERSENSITIVITY


Type III reaction occurs when an excess of immune complexes deposits in the tissues. Immune
complexes are usually formed when antibodies bind to soluble antigens. After immune complexes
are formed they are usually cleared by phagocytes but if the clearance is delayed the complement
cascade is activated and this may result in tissue damage.
Although phagocytes play an important role in immune complex removal, red blood cells are also
very important in carrying immune complexes to the spleen and liver for destruction. Often, numerous
immune complexes, too small to bind FcR on phagocytes, are removed from the circulation by
erythrocytes which bare complement receptor CR1 (its a receptor for C3b). CR1 expressed by the
red blood cell binds to C3b deposited on to the Fc portion of the antibody attached to an antigen.
These complexes are brought to the spleen by the red blood cells and are degraded before they can
cause any damage.
Under certain
conditions, such as when
the antigen persists in the
body for long periods or
when high levels of antigen
are encountered at one
time, immune complexes
reach such high levels that
they are no longer soluble
or easily removable. These
immune complexes can then deposit and stay in tissues. Common sites of complex deposition and
tissue damage are: blood vessel walls, kidneys, and joints. Immune completes often deposit in
organs where blood is filtered at high pressure to form other fluids, such as urine and synovial fluid.
Immune complex deposition can result in localized reactions or generalized (more systemic)
reactions. The antigen causing the antibodies to deposit can either be foreign or self (in which case
the antibodies are auto-antibodies).
Example of localized, foreign antigen-induced reaction:
1. Arthus reaction - In this condition, a preformed IgG (formed during the sensitization phase)
forms local immune complexes with the antigen. The antigen is usually something that a
person has been primed to before. For example, arthus reaction may result from a tetanus
booster shot that is re-administered after an accident. In this situation, an individual would
already have IgGs formed during prior immunizations. The tetanus toxoid booster shot
administered after accidents and these preformed IgGs will form complexes, locally, in the
82

MERP Medical Immunology

tissue where the shot was administered. These complexes activate complement, which in turn
releases mediators such as C5a, which creates a local inflammatory response. The lesion
contains primarily neutrophils, deposits of immune complexes and complement. Macrophages
infiltrating in later stages may be involved in the healing process.
Example of generalized, foreign antigen-induced reaction:
1. Serum sickness - generalized form of Type III hypersensitivity.
In the pre-antibiotic era, immune horse serum (containing antibodies specific for a
particular antigen) was often given to treat many infections. Antibodies from the horse serum
helped clear the infection in the human. However individuals receiving horses serum
sometimes developed their own immune response against the horse protein.
A similar situation can occur in a person receiving a therapeutic passive immunization
with horse neutralizing antibodies to rattlesnake venom. In both cases, a person receiving
horse antibodies for the first time makes a primary response to the foreign IgG protein 7-10
days after administration. After 7-10 days, enough human anti-horse IgG antibody has been
produced to form immune complexes in the circulation and deposit in small vessels. The
immune complex consists of human antibody binding to the Fc portion of the horse antibody.
This activates complement, neutrophils and macrophages systemically, since the passive
immunization is given as an IV.
Symptoms of serum sickness are systemic and include: fever, chills, rash, arthritis, and
sometimes kidney damage. Note that on second and subsequent exposure to horse
antibodies, the patient would begin experiencing serum sickness within a couple of days since
isotype switching to IgG had already occurred.
Examples of generalized, autoantigen-induced reactions:
1. Rheumatoid arthritis (RA). Rheumatoid arthritis is a
chronic inflammatory disorder that typically affects the
small joints in your hands and feet. Unlike the wear-andtear damage of osteoarthritis, rheumatoid arthritis affects
the cartilage lining of the joints, causing a painful
swelling.
In autoimmune RA, patients develop an auto-IgM
antibody against their own IgG which forms an immune
complex that contributes to arthritic joint inflammation.
RA usually affects joints on both sides of the body
equally. Wrists, fingers, knees, feet, and ankles are the
most commonly affected.
In RA, the synovial fluid is rich in
polymorphonuclear leukocytes that have encountered
and engulfed immune complexes. Joint symptoms may
include: morning stiffness, which lasts more than 1 hour,
warm joints, and joint pain often felt on the same joint on
both sides of the body. Once the inflammatory reaction is
established, the synovium thickens, the cartilage and the
underlying bone begins to disintegrate and evidence of joint destruction accumulates. Over
time, joints may lose their range of motion and may become deformed. In addition to causing
joint problems, rheumatoid arthritis sometimes can affect other organs of the body such as
the skin, eyes, lungs and blood vessels.
83

MERP Medical Immunology

2. Systemic Lupus Erythematosis (SLE)


Lupus is another chronic inflammatory disease that occurs when your body's immune system
attacks healthy tissue in your body. Lupus results from a combination of your genetics and
your environment. Many different body systems are affected by SLE including: joints, skin,
kidneys, liver, blood vessels, brain, nervous system, heart and lungs. Mechanistically, in SLE,
autoantibodies are produced against the patients own DNA and histones. Additionally
autoantibodies against platelets can be detected. The course of the disease is unpredictable,
with periods of illness alternating with remissions. Symptoms of lupus include severe fatigue,
painful or swollen joints, headaches, rash on cheeks and nose called butterfly rash, hair loss,
anemia, blood-clotting problems and Raynaud's syndrome (fingers turning white/blue and
tingle when cold).

84

MERP Medical Immunology


Events required triggering lupus

A schematic of Type III reactions:

Arthus rxn

Farmers
lung

Serum
sickness

SLE

RA

TYPE IV HYPERSENSITIVITY
Type IV reaction is called delayed-type hypersensitivity (DTH) because its effects are felt 4872 hours after antigen contact (rather than within minutes to hours). In contrast to Type I, II and II
reactions which are mediated by antibodies, Type IV response is mediated by antigen-specific Th1
cells, macrophages, or by CD8 T cells.
As in other hypersensitivities, the initial response is called sensitization, and may not result in
symptoms. The initial sensitization phase, i.e. the priming of immune cells, takes 1 2 weeks to
develop, the same amount of time as in Type I, II and III reactions. In sensitization, small non-protein
or protein antigens either penetrate the skin or are scratched into the dermis in response to itching.
They form complexes with skin self-proteins and alter them. The presentation of these modified selfproteins on MHCII or MHC I of a dendritic cell, in turn, causes activation of Th1 cells (if the antigen is
vesicular) or CD8 T cells (if the antigen is cytoplasmic) which results in the generation of memory T
cells. Upon second contact with the antigen, memory T cells will be recruited to the site of exposure,
recognize their MHC-antigen complex on skin cells and macrophages, secrete perforin and granzyme
(CTL memory) and cause the characteristic itchy rash.
In the case of DTH Th1 response, a Th1 cells may recognize macrophages presenting a
peptide within its MHC II. These Th1 cells then secrete chemokines and IFNgamma which activate
macrophages. Macrophage activation causes TNFalpha and TNFbeta to be secreted which in turn

85

MERP Medical Immunology

upregulates adhesion molecules on local blood vessels and causes vasodilation. IL-3 and GM-CSF
secretion leads to an increase in bone marrow output of monocytes.
Clinical examples of Type IV reactions include self and foreign antigens:
Foreign-antigen induced reactions:
1. Local skin reaction to poison ivy (right).
In the case of poison ivy, during sensitization phase
there is absorption of an allergen chemical (called
urushiol) into the skin which elicits an immune response
that is remembered on subsequent allergen exposures.
Urushiol is low-molecular-weight and can easily penetrate
the keratinocytes of the superficial epidermal layers.
Urushiol is also lipid-soluble and can therefore cross the
cell membrane and modify intracellular proteins. These
modified proteins generate modified peptides, within the
cytosol, which are translocated into the endoplasmic reticulum and are presented by MHC class I
molecules of the keratinocytes and local DCs. DCs take these MHC I bound peptides to the lymph
nodes to present to nave CD8 cells. During T cell activation, CD8 memory cells form. Urushiol is
such a strong sensitizer that it can produce intense inflammation in weak concentrations and can
cause sensitization in 10 to 14 days after only one exposure.
Within 24-72 hours after re-exposure to urushiol, the allergen penetrates the epidermis,
modifies intracellular proteins and recruits CD8 T memory cells. This causes an overt inflammatory
reaction to the allergen. This dermatitis can persist for 3 to 4 weeks even after the antigen is removed
and is characterized by erythema, edema, and vesiculation resulting from destruction of epidermal
cells and activation of the dermal vasculature.
The reason the re-exposure takes a few days to present (rather than a few hours, as in Type I,
II and III reactions) is because Type IV reaction is T cell mediated and the memory T cells have to be
recruited from the vasculature to the skin. These memory T cells are not present at high enough
numbers in the tissue. In contrast, antibody-mediated hypersensitivities take shorter time to cause an
effect upon re-exposure, since antibodies are found at high levels in tissues.
Other examples of DTH where CD8 cells are involved include reactions to proteins in insect
venom and contact sensitivities to metals such as nickel.
2. Local skin reaction to
Mycobacterial proteins used in
TB skin testing (PPD test). See
figure to the right.
Sensitization to TB occurs during
prior exposure to Mycobacterium
tuberculosis which in turns results in
production of memory Th1 cells to
Mycobacterial proteins. This
sensitization could be a result of
vaccination, an active TB infection or
latent/dormant TB.
86

MERP Medical Immunology

During the TB test, when purified tuberculin protein is injected into the skin, resident macrophages
phagocytose the particles and present them on MHC II. These macrophages recruit memory Th1
cells from the blood stream. The memory Th1 cells bind to tuberculin proteins presented by the
macrophages on MHC II and secrete more cytokines. The Th1- secreted cytokines activate
macrophages further and attract more macrophages and granulocytes from the blood. This reaction
causes induration and erythema as normally seen in a positive PPD test.
3. Chronic Allograft rejection
Most graft rejection reactions are T cell-mediated responses. Transplanted organs express
donor MHC molecules, resulting in 2 pathways of antigen recognition (allorecognition) by T cells:
direct and indirect.
Direct pathway (Figure below): Allorecognition is the term used to define immunological
recognition of HLA antigens between genetically disparate individuals within the same species.
Specifically, allorecognition
Direct pathway-After transplantation, dendritic cells (DCs) from the donor migrate
refers to direct recipient T cell
out of the graft into the recipients lymph nodes and present donor peptides on their
recognition of MHC molecules
own major histocompatibility complex (MHC) to the T-cell receptor (TCR) of the
on donor cells.
recipients T cells. This elicits an anti-donor T-cell allorecognition response that
destroys the graft.
In graft rejection, donor
APC may travel out of the graft
and into the hosts regional
lymph node (See panel a in
the figure to the right). When
these donors MHCs are
recognized by the host T cells
the T cells are activated (b).
c.
d.
b.
a.
These host T cells can then
migrate back to the graft,
recognize the MHC expressed
by the donor tissue and cause inflammation and rejection of the graft (c d). Both CD8 and CD4 T
cells are involved, but the destruction of the graft is mediated mainly by CTLs (CD8 cells).
The pathway behind the direct pathway is thought to involve the fact that there is no
mechanism for eliminating host T cells whose TCRs have a high affinity for allogeneic MHC
molecules. Since alloMHC is not present in a recipients thymus, these allo-reactive T cells are not

Direct pathway: Recognition of allogeneic major histocompatibility complex (MHC) molecules by T lymphocytes. Recognition
of allogeneic MHC molecules may be thought of as a cross-reaction in which a T cell specific for a self MHC molecule-foreign
peptide complex (A) also recognizes an allogeneic MHC molecule whose structure resembles that of a self MHC moleculeforeign peptide complex (B and C). Peptides derived from the graft (labeled self peptide) may not contribute to allorecognition
(B), or they may form part of the complex that the T cell recognizes (C). The type of T cell recognition depicted in B and C is
direct allorecognition.

87

MERP Medical Immunology

negatively selected and exit into the periphery. The process of negative selection eliminates host T
cells that strongly react to self MHC but leaves a small percentage of T cells to recognize foreign
MHC. Approximately 5% of T cells in an individuals peripheral pool are alloreactive. These T cells
have passed the requirements of positive and negative selections in the host, but have a TCR with
specificity for allogeneic MHC.
Indirect pathway: After transplantation, recipient DCs migrate into the graft, and take up and
process proteins from the donor cells. These donor peptides are then presented on recipient MHC to
recipient T cells in the lymph nodes, again eliciting an immune response. It is thought that the direct
pathway is responsible for acute rejection and that the indirect pathway is responsible for chronic
rejection.
Examples of autoantigen-induced reactions:
1. Type I diabetes (right)
Type I diabetes is a chronic condition in which the pancreas
produces little or no insulin, a hormone needed to allow glucose
to enter cells to produce energy. The far more common type 2
diabetes occurs when the body becomes resistant to the effects
of insulin or doesn't make enough insulin.
In Type I diabetes the destruction of beta cells in the
pancreas plays the most important mechanism. MHC class Idependent T cell responses are an essential component of both
the initiation and progression of pancreatic cell destruction,
ultimately leading to type 1 diabetes. It has been shown that an
autoimmune response towards beta cells is elicited, involving an
expansion of autoreactive CD8+ T cells. Later in the disease
CD4 T cells and insulin-specific autoantibody-producing B cells
also play a role.
2. Multiple sclerosis (MS) (right)
Multiple sclerosis has been defined as "multiple
white matter lesions separated in space and time". In
this disease the immune system eats away the
protective myelin sheath that covers the nerves in the
central nervous system. The activation of CD4+
autoreactive T cells and their differentiation into a Th1
phenotype are crucial events in the initial steps, and
these cells are probably also important players in the
long-term evolution of the disease. Although Th1 cells
induce the initiation of MS, damage to the myelin of the
central nervous system is most likely mediated by other
components of the immune system such as: antibodies,
complement, CD8+ T cells, macrophage-secreted
factos and other factors produced by innate immune
cells.
Damage to myelin in the white matter of the CNS
interferes with the communication between the brain and the rest the body. Many patients have
experience demyelination of the optic nerve which controls vision. Inflammation of this nerve
causes optic neuritis and visual defects. Damage to spinal cord pathways (myelopathy) is yet
88

MERP Medical Immunology

another common presentation. This can include damage to descending motor pathways,
producing spastic paralysis or damage to sensory tracts (usually the dorsal columns). In the latter
case, paresthesias (sensation of tingling, tickling, pricking) may be very distressing.
3. Celiac disease (bottom)
Celiac disease is defined as a disorder in which there is an abnormality of the small intestinal
mucosa manifested by contact with the gluten of wheat and certain other cereal grains. Nutrients from
food are normally absorbed
by the villi in small intestine.
If the villi are damaged, the
person cannot absorb
nutrients properly and ends
up malnourished, no matter
how much he or she eats.
Glutens have a high
proline and glutamine
content. The high proline
content renders these
proteins resistant to
complete proteolytic
digestion by gastric enzymes
in the human intestine. This
can result in the
accumulation of relatively
large peptide fragments (as
many as 50 amino acids in
length) in the small intestine.
These peptide can penetrate
into the gut mucosa. In the
mucosa gluten peptides are
modified by tissue
transglutaminase. Certain
MHC II alleles, such as HLA-DQ2 and HLA-DQ8 expressed by APCs, can then bind and present
gluten peptides to populations of CD4+ T cells in the lamina propria of the small intestine. Intestinal
CD4 T cells are then activated which in turn cause inflammation of the intestine. In celiac disease, in
addition to activated T cells, antibodies to transglutaminase can also be found.

89

MERP Medical Immunology

Practice questions:
1. Fran walks outside on a beautiful day and takes a deep breath of ragweed pollen, to which she
has a strong Type I hypersensitivity. Which event below will NOT occur within 30 minutes due to
this hypersensitivity?
a.
b.
c.
d.
e.

A local inflammatory response in the nose is induced, resulting in a runny or stuffy nose.
IgE specific for ragweed pollen is synthesized by B cells in the local lymph nodes.
Mast cells respond to the antigen-IgE signal by releasing preformed histamine
Ragweed pollen antigen binds to IgE present on mast cell FceRI in the respiratory tract.
Systemic effects of hypersensitivity such as anaphylactic shock may occur.

2. The principal difference between cytotoxic (type II) and immune complex (type III) hypersensitivity
is:
a.
b.
c.
d.
e.

The class (isotype) of antibody


The site where antigen-antibody complexes are formed
The participation of complement
The participation of T cells
The time frame of sensitization phase

3. A patient with rheumatic fever develops a sore throat from which beta-hemolytic streptococci are
cultured. The patient is started on treatment with penicillin (first time treatment), which he
continues for the prescribed course, and the sore throat resolves within first several days.
However, 14 days after initiation of penicillin therapy the patient develops a fever of 103F, a
generalized rash, and proteinuria. This MOST probably resulted from:
a.
b.
c.
d.
e.

recurrence of the rheumatic fever.


a different infectious disease.
an IgE response to penicillin.
an IgG-IgM response to penicillin.
a delayed hypersensitivity reaction to penicillin.

4. You see a 26 year-old female in the clinic complaining of cold intolerance, weight loss, palpitations
and bulging eyes (exopthalmos). You suspect Graves disease. What is the mechanism behind
her symptoms?
a. The pituitary overproduces TSH due to stimulation by IgG.
b. The thyroid overproduces T3 and T4 due to stimulation by IgG
c. The pituitary overproduces TSH due to a lack of feedback inhibition by the thyroid
d. T4 is peripherally converted to more bioactive T3 by autoimmune stimulation
5. You see a patient in the ER who has been bitten by a rattlesnake. You administer murine antivenom antibodies intravenously. The patient is saved, but you notice a diffuse petechial rash on all
four limbs at the one-week follow-up appointment. What is the cause of this rash?
a. Micro-thromboembolus from direct Ab-mediated platelet activation
b. Cytotoxic T-cell mediated vascular damage
c. Localized swelling from mast cell degranulation
d. Accumulation of antigen-antibody complexes in the vasculature
90

MERP Medical Virology

VIROLOGY MODULE
VACCINES
Learning Objectives

List the composition of some common, routinely administered vaccine


Understand the components of toxoid vaccines
Understand the components of recombinant vaccines
Understand the components of conjugate vaccines
List advantages and disadvantages of live versus killed vaccines

A historical blurb
First vaccine was developed in the 18th century. It was a vaccine against smallpox disease
which was caused by a virus from the Poxviridae family. In this vaccine, rather than smallpox, cowpox
virus was used as an immunizing agent (cowpox is structurally similar to smallpox,). At first lesions
from cowpox infected milk-maids or lesions from udders of cattle that had cowpox were used for
inoculation of susceptible people. Eventually the live virus was purified and used directly instead of
lesion fluid. This is an example of a live vaccine. Prior to the discovery that cowpox protects against
human smallpox, a more dangerous procedure was used. Lesions from smallpox-infected people
were scratched and directly inoculated into non-infected people. Today vaccines exist in many other
forms, in addition to live vaccines.
Common types of vaccines:
Most bacterial and viral vaccines are administered by injection.
Viral vaccines
Killed (also called inactivated) vaccine is composed of viral particles that have been treated
with chemicals, heat, or irradiation such that they are no longer alive (ex. Influenza virus
vaccine, rabies virus vaccine).
Live (also called attenuated) vaccines contain live virus with a reduced ability to grow in
humans. These are more potent than killed vaccines because they more closely mimic a real
infection. Most viral vaccine are live (ex. vaccine against measles, mumps, rubella).
Subunit (also called recombinant) vaccines are vaccines that use only a particular
component of the virus rather than an entire virus. Usually these components are given as
proteins, which were translated in bacteria or yeast (ex. Hepatitis B, HPV vaccine).
Bacterial vaccines
Live-attenuated vaccines- There is a small number of whole cell live-attenuated bacterial
vaccines: ex. Salmonella, Mycobacterium tuberculosis.
Subunit vaccinesToxoid vaccines - Since many bacterial diseases result from toxic proteins secreted by
bacteria, rather than the bacterium itself, a purified inactivated version of a particular
toxin, called toxoid, can be used as a vaccine against such bacteria. These vaccines
are made by purifying a toxin and inactivating it with a chemical called formalin
(example of toxoid vaccines are diphtheria toxoid against Corynebacterium diptheriae
and tetanus toxoid against Clostridium tetani).
91

MERP Medical Virology

Capsular conjugated and unconjugated vaccines - Many bacteria have very


pathogenic capsules (a polysaccharide structure covering the surface of bacteria). This
structure is not immunogenic in that it cannot fix complement very well, nor stimulate
other components of the innate system, but it can elicit an adaptive immune response.
Therefore subunit vaccines have been developed where injected purified capsule
stimulates and antibody response. Often these vaccine are administered as conjugate
vaccines where the capsule is conjugated to a carrier protein to make the capsule more
immunogenic (more on this later).
Adjuvants
To be effective a vaccine must create a state of inflammation. Therefore helper substances that
induce inflammation are used in conjunction with a vaccine. These substances are called adjuvants.
An example of an adjuvant is alum-a form of aluminum hydroxide.
Advantages and disadvantages of live and killed vaccines
Live- attenuated vaccines can, in some instances, revert to becoming pathogenic. This
happens rarely, and if the vaccine is thought to provide much more benefit than harm, it is still
recommended.
In general, live-attenuated vaccines have some advantages over inactivated vaccines: they
are more effective, provide life-long immunity and do not need repeated boosters.
On the other hand the advantages of inactivated vaccines include their stability and ease of
transportation, as well the lack of a risk associated with vaccine-related illness. However these
vaccines are not as potent as live vaccines. When a person is vaccinated by either of the above
mentioned vaccines, he develops active immunity.
Passive immunization
Passive immunization is used when exposure to pathogen has already occurred and there is
not enough time to induce active immunity, or prophylactically in children with immune deficiencies.
Passive immunization involves administration of specific antibody which has been produced in a
human or animal in response to vaccination or environmental exposure to the pathogen. Examples
include antivenin for snake bite, Rhogam (human anti-Rh) to block formation of IgG anti-Rh
antibodies in an Rh-negative pregnant mother, and human gamma globulin given to children who
have immune deficiencies. Passive immunization can confer fast but short-term protection. When the
antibodies come from another species, serum sickness can develop.

92

MERP Medical Virology

Administration of vaccines

93

MERP Medical Virology

LECTURES 9 and 10: INTRODUCTION TO VIROLOGY


Learning Objectives
1. Describe the viral life cycle.
Define viruses
Explain differences between receptor mediated endocytosis and receptor mediated
fusion
Differentiate between lysis, budding, exocytosis
Differentiate between lytic, persistent, latent and chronic infections
2. Describe the main differences between nonenveloped and enveloped viruses.
Difference in stability
Difference in entry
Difference in exit
Difference in transmission
Difference in immune response required
Difference in structure
3. Describe the differences between viruses that have RNA versus DNA as their genome.
Describe the difference in stability within a cell
Describe the difference latency vs. no latency
Be able to answer why the replication of a particular virus is in the nucleus vs. the
cytoplasm
Describe the mechanism and importance of temporal regulation of transcription
4. Describe in detail the steps in the replication of the RNA genome, stressing the differences
between positive and negative RNA.
5. Describe in detail the steps in the replication of the DNA genome.
6. For each viral family describe the steps necessary to produce viral RNA and viral proteins.
Although historically viruses were grouped according to the disease symptoms they produced and the
range of host species they infected, today classification is based largely on architecture of the virus
particle (the virion) and the nature of the genetic material. Further distinctions can then be made on
the basis of biological properties and antigenic relationships.
Below you will find some general features of all viruses:

Filterable agents

Obligate intracellular parasites

Outside the cell, they consist of particles called virions

Use energy and protein synthesis machinery of the host

The protein outer shell of a vision is called a capsid, a capsid is always encoded by the viral
genome. Sometimes on the outside of the capsid the virion may contain other ingredients (e.g.,
lipids, carbohydrates), but these are derived from the host cells.
94

MERP Medical Virology

A virions genomic material may be DNA or RNA genomes, but not both.

Viruses have genes that are few in number (3 - 100 depending on the species). They encode
those proteins needed for viral reproduction that the host cell will not supply.

Viruses assemble their components (genomes and capsids) before they exit the cell and
therefore, while they copy their genomes in a similar fashion to eukaryotic cells, the overall
processes of the viral replication cycle is referred to as assembly.

Viral infection can be lytic (cause cell death), persistent (no cell death) or transforming
(immortalization of a cell).

Viral life cycle (see figure right)


1. Adsorption (Attachment)
Viruses can only enter cells that express viral receptors
and are able to replicate in cells with appropriate
biosynthetic machinery. Below are some examples of
viral attachment proteins and their respective receptors
on host cells.

2. Penetration
Viral entry depends on the nature of the virus (enveloped versus naked). Following attachment, viruses may enter the
host cell through receptor mediated endocytosis (naked and enveloped viruses are capable of this) or by receptor
mediated fusion (only some enveloped viruses are capable of doing this).
3. Uncoating
Uncoating is a process where viral capsid is degraded by viral enzymes or host enzymes. This can occur in the vesicle
after receptor mediated endocytosis or in the cytoplasm after receptor mediated fusion.
4-5. Early transcription/translation

95

MERP Medical Virology


In this step, host (or rarely viral) DNA dependent RNA polymerase transcribes viral genes that are responsible for viral
replication and genes that encode proteins that serve as transcription factors for late genes. These are than translated by
hosts translation machinery.
6. Replication
In the case of DNA viruses, viral genome is replicated either by host or viral DNA dependent DNA polymerase. In the case
of RNA viruses the viral genome is always replicated by viral RNA dependent RNA polymerase.
7-8. Late transcription/translation
Late transcription and translation usually applies to DNA viruses. In this step, host DNA dependent RNA polymerase
transcribes late viral genes that code for structural proteins and then the host translation machinery translates this late
mRNA.
9. Assembly
Viral genomes are packaged with viral capsids and if the virus is enveloped, the nucleocapsid is delivered to the plasma
membrane.
10. Release
Viruses may escape from the host cell by causing cell rupture (lysis). Most naked viruses cause cell lysis as they exit.
Enveloped viruses typically "bud" from the host cell. During the budding process, a virus acquires the phospholipid
envelope containing the embedded viral glycoproteins. This enveloped can be derived from one of several membrane
bound compartment within a cell. Depending on the efficiency of viral replication, budding may or may not result in cell
lysis. Some enveloped viruses can move between cells via syncytia formation, during which the infected cell fuses with a
non-infected cell to form a multinucleated giant cell. For those viruses capable of forming syncytia, CD8 response is
critical to control viral infection

Enveloped versus non-enveloped viruses


Viral properties (such as stability) in the external environment, as well as its ability to enter and
exit the cell, depend on its external structure.

Non-enveloped/Naked viruses
There are several characteristics of naked viruses that are important for their pathology:

Basic structure of naked


viruses

Outer most layer is made


of a protein capsid

The capsid consists of 12


vertices, 20 triangular
faces, and 30 edges. The capsid is usually made of many different proteins assembled
symmetrically into an icosahedral shape.

96

MERP Medical Virology

Capsids are usually environmentally stable and resist changes in


temperature, acidic conditions, detergents and drying.

Naked viruses are usually released by cell lysis.

Naked viruses are easily spread on fomites, hand-to-hand touch,


dust and small droplets.

Naked viruses can dry out and remain infectious.

Naked viruses survive the adverse acidic conditions in the gut.

Naked viruses can resist poor sewage treatment.

Antibody may be sufficient for immunoprotection against naked


viruses.

Enveloped viruses
There are several characteristics of enveloped viruses that are important for their pathology:
Basic structure of enveloped viruses

Outer most layer of these viruses is made up of a lipid bilayer membrane embedded with
glycoproteins.

Enveloped viruses are environmentally labile/unstable and are disrupted by heat, acid,
detergents and drying.

Enveloped viruses can be released by budding, exocytosis or cell lysis.

Enveloped viruses modify host cell membrane during replication.

Enveloped viruses must stay moist to retain infectivity.

Enveloped viruses are spread in large droplets such as bodily secretions, organ
transplantations and blood.

Enveloped viruses do not need to kill the cell to spread; can spread from cell to cell via
syncytia.

97

MERP Medical Virology

Syncytia formation by enveloped viruses

Enveloped viruses may need Ab and cell mediated response for control and protection.

Enveloped viruses may elicit hypersensitivity and inflammation causing immunopathology.

Viral replication steps 2 and 3 depend on whether the virus is enveloped or naked (Figure below)
2. Penetration
Viral entry depends on the nature of the virus
(enveloped versus naked). Following attachment,
viruses may enter the host cell through receptor
mediated endocytosis (naked and enveloped
viruses are capable of this) or by receptor
mediated fusion (only some enveloped viruses
are capable of doing this).
3. Uncoating
Uncoating is a process where viral capsid is
degraded by viral enzymes or host enzymes. This
can happen inside the endocytic vesicles or
inside the cytoplasm depending on the virus.

98

MERP Medical Virology

Viral replication steps 4 through 8 depend on whether the virus is DNA or RNA (Figure below)
4-5. Early transcription/translation
In this step, host DNA dependent RNA polymerase
transcribes viral genes that code for proteins responsible for
viral replication and genes that encode proteins that serve as
transcription factors for late genes. The mRNAs for these
genes are translated by hosts translation machinery.

6. Replication
In the case of DNA viruses, viral genome is replicated either
by host or viral DNA dependent DNA polymerase. In the
case of RNA viruses the viral genome is always replicated by
viral RNA dependent RNA polymerase.

7-8. Late transcription/translation


Late transcription and translation usually applies to DNA
viruses. For most DNA viruses, host DNA dependent RNA
polymerase transcribes late viral genes that code for structural proteins and then the host translation machinery
translates this late mRNA. Note that RNA viruses rarely have early and late transcription but rather carry out
transcription and/or translation in one step.

DNA versus RNA viruses


Properties of DNA viruses

DNA molecule is stable

Because it is stable, DNA genome can

DNA virus
replication

remain in the infected cell for a long time

Many DNA viruses establish a persistent


infection.

DNA genome resides in nucleus (except


for POX which resides in the cytoplasm).

Viral genes interact with host proteins to


get help with transcription and replication
(except POX which carries out its own
transcription).

For DNA viruses, viral gene transcription is temporally regulated (early genes code for
replication machinery, late genes code for structural proteins).
99

MERP Medical Virology

Larger DNA viruses can promote their own replication via a DNA dependent DNA polymerase
that these viruses encode.

In order to best utilize host machinery, all DNA viruses must have a way to make the target cell
divide, or they must infect a previously dividing cell.

Properties of RNA viruses

Positive RNA virus


replication

Unlike DNA, RNA molecule is


transient and labile.

Most RNA viruses replicate in


cytoplasm (except Orthomyxo,
which replicated in the nucleus)

Cells cannot replicate RNA. RNA


viruses must encode RNA d.
RNA pol which has low fidelity so
RNA viruses are prone to
mutation.

The genome structure and


polarity determines how viral
mRNA is generated and proteins
are processed.

RNA viruses can be negative, positive or double stranded depending on the structure of their
RNA genome.

All (-) RNA viruses are enveloped.

All (-) RNA viruses must carry RNA dep. RNA polymerase in the virion.

Retroviruses are unique. These viruses have a positive sense RNA genome but they must be
converted into a dsDNA by the viral encoded and carried enzyme Reverse Transcriptase
(RNA dependent DNA polymerase) prior to integration into the host cell.

100

MERP Medical Virology

Negative RNA virus replication

Viral replication steps 9 (Assembly) depends on both the nature of the viral coat structure (i.e., naked
or enveloped) and on whether its DNA or RNA whereas step 10 (Release) depend on whether the
virus is naked or enveloped (figure right)
9. Assembly
Viral genomes are packaged with the viral capsids and
if the virus is enveloped, the nucleocapsid is then
delivered to the plasma membrane or ER to acquire
the envelope.
10. Release
Viruses may escape from the host cell by causing cell
rupture (lysis). Most naked viruses cause cell lysis as
they exit. Enveloped viruses typically "bud" from the
host cell. During the budding process, a virus acquires
the phospholipid envelope containing the embedded
viral glycoproteins. Depending on efficiency of viral
replication, budding may or may not result in cell lysis.
Note that some viruses exit via exocytosis instead of
budding, which means that they acquire their
membrane in an internal organelle, such as ER, and
then are released out of the cell via a vesicular pathway. Some enveloped viruses can move between cells via
syncytia formation, during which the infected cell fuses with a non-infected cell to form a multinucleated giant cell.
For those viruses capable of forming syncytia, CD8 response is critical to control viral infection

101

MERP Medical Virology

Figure: Budding of an enveloped


virus

General clinical progression of viral infections


Many viruses initiate infection in the oral mucosa or upper respiratory tract. Disease signs may
accompany viral replication at the primary site. The virus may replicate and remain at the primary site,
or it may disseminate to other tissues via the bloodstream or via the mononuclear phagocytes and
lymphatic system. Alternatively, some viruses spread via neurons. The transport of virus in the blood
is termed viremia. This primary viremia may not result in a large number of virions. However, in the
blood, viruses taken up by the phagocytic macrophages may be delivered to other tissues.
Replication of a virus in these other tissues, such as the endothelial lining of blood vessels, or the
liver can cause the infection to be amplified and initiate the development of a secondary viremia
which can then cause sufficient virions to be released to infect more target tissues such as skin, brain
etc.
The outcomes of viral infections are shown
in the figure to the right. Viral infection of a target
organ may cause lysis of the target cells within the
organ. Alternatively, some viruses establish
persistent infections in which the infected cell is
not killed. These persistent infections can be
divided into chronic and latent types. For a chronic
infection to happen, the virus must be released
from the cell gently through exocytosis or through

Outcomes of viral infections

budding from the plasma membrane. If the infected


cell is killed by CTL or ADCC chronic infection of that cell will end. Certain DNA viruses can cause
102

MERP Medical Virology

latent infections of a cell. This occurs if the infected cell lacks the machinery for transcribing all the
viral genes. The factors required by such a virus to make progeny may be expressed only in activated
but not resting cells. For example, HSV establishes a latent infection in neurons that lack the nuclear
factors required to transcribe the early viral genes, but stress and other stimuli can activate the cells
to allow viral replication. This then allows HSV to come out of its latent stage and produce progeny.
Below is a map of viral families and their characteristics.

Practice questions:
1. A patient is sick with severe diarrhea. The examination of a stool sample reveals an icosahedralshaped virus with long fibers protruding from the virus. What else is a likely characteristic of this
virus?
a.
b.
c.
d.
e.

It is naked
It is enveloped
It is an RNA virus
It is a DNA virus
It is a retrovirus

2. All of the following describe killed vaccines except:


a.
b.
c.
d.
e.

Very stable under different environmental conditions


May be composed of dead virus or viral particles
May be used to protect against either viral or bacterial infections
Less commonly used as viral vaccines of choice
More potent than live vaccine because they mimic infection
103

MERP Medical Virology

LECTURE 11: POXVIRIDAE and PARVOVIRIDAE


Learning Objectives
1. Describe clinically and biochemically Poxviridae family.
Describe the structure of Pox
Describe the unique features of Pox
Describe the reasons for eradication of smallpox
Exaplain the importance of vaccinia
Describe the spread, clinical presentation, population at risk, treatment for Molluscum
contagiosum
2. Describe clinically and biochemically Parvovirus B19.
Describe the cell target of Parvovirus
Describe the diseases progression of Parvovirus
List human diseases caused by Parvovirus
Describe the role of immune response in Parvovirus infections
POXVIRIDAE
Unique properties of Poxviruses:

Largest, most complex viruses

Brick-shaped

The receptor for pox virus is


a glycosaminoglycan
expressed in the host
memebrane

The only DNA virus that


replicates in cytoplasm

Because it replicates in the


cytoplasm it encodes and carries all proteins needed for mRNA synthesis

Assembles in cytoplasmic Guarnieri bodies (inclusion bodies)

Encodes soluble interferon, IL-1beta, and TNF-alpha receptor homologues that block the host
cytokines

Virus stimulates cell growth by encoding a protein similar in sequence to a growth factor called
EGF. This protein is secreted and binds to growth factor receptors on infected cells.

104

MERP Medical Virology

Replication of Poxviruses:
A range of pox viruses cause febrile illnesses in man
and animals with a prominent vesicular rash.
In humans, the most prominent was smallpox virus
which caused a severe disease but has now been
eliminated by intensive international vaccination.
Current interest in poxviruses centers around their
possible use as vaccine vectors.

Human diseases
1. Smallpox
This is the only disease that has been globally
eradicated.
Transmission of smallpox is through close contact
with infected persons, usually via infection with a
droplet from the pharyngeal secretions. The ulcers in
pharynx shed the virus. Incubation period for smallpox is 10 - 12 days.
Major illness is the most common form of smallpox. It is characterized by abrupt onset of fever
and prostration with a macular rash on the third
day (head, limbs, hands and feet rather than
trunk, including palms and soles).
Rash progresses to vesicles which
become pustular, ulcerated, scabbed, and
eventually healed but with scarring ("pock
marked face"). Often skin pustules
become contaminated with bacteria that
normally reside in the skin. Vesicles are
also found inside the mouth. An infected
person needs cell-mediated and humoral
response for resolution. Viral replication
results in cell lysis. Smallpox has 15 to

105

MERP Medical Virology

40% mortality rate (Figure on the left illustrates smallpox pathogenesis).


As result of an unprecedented international WHO campaign of 'seek and contain' by
vaccination, the last antigens of a natural case of smallpox was diagnosed in 1977. Vaccinia is a viral
strain which has been used for immunization against smallpox. In man, it caused a localized
pustule with scar formation. In immunocompromised persons or eczematous persons it sometimes
caused a severe generalized vaccinia infection. Routine immunization of all children in USA stopped
in about mid 1980's.

2. Cowpox is acquired by humans, usually through milking cows; it then manifests as ulcerative
lesions (sometimes called "milkers nodules") on the hands of dairy workers. It is a local infection, not
nearly as morbid as smallpox. Because of antigenic similarity between cowpox and smallpox, people
who recover from a cowpox infection are protected against smallpox.

Molluscum contagiosum (image on the right) is a minor infection that


presents with infectious warty papules on the skin. These papules have a
central umbilication and are found in a cluster of 5 - 20. Disease is
transferred by direct contact with infected human and as a venereal
disease. The lesions are not found on palms and soles. Diagnosis is usually
based on clinical findings.
PARVOVIRIDAE B19
Unique properties of Parvoviruses

Parvoviruses are among the smallest, simplest eukaryotic viruses

Particles are icosahedral, 18-26nm diameter and consist only of protein (50%) + DNA (50%).

They are naked viruses with an icosahedral capsid


106

MERP Medical Virology

They have a ssDNA genome

The virus uses hosts DNA d. DNA pol for replicating its genome

The capsid resists inactivation

It can cross the placenta

It infects mitotically active erythroid precursors in the bone marrow and establishes a lytic infection

Replication occurs in the nucleus

All parvoviruses are highly dependent on cellular functions for genome replication

Replication of Parvoviruses

Human diseases:

All are caused by one serotype (B19) but

infection can result in different disease manifestation.

B19 is spread by respiratory route and from

mother to fetus.

It replicates in mitotically RBC precursors.

(Major sites for replication include: bone marrow and


fetal liver).
Individuals at risk:
1. Children: Children with B19 present with erythema
infectiosum (also called slap cheek and fifth
disease) which is a mild systemic rash illness.
2. Parents of kids with B19 who have not been
Postulated replication of parvovirus (B19) based on
information from related viruses. The internalized
parvovirus delivers its genome to the nucleus, where the
sinagle-stranded (plus or minus) DNA is converted to
double-stranded DNA by host factors and DNA
polymerases present only in growing cells. Transcription,
replication, and assembly occur in the nucleus. Virus is
released by cell lysis.

exposed in childhood can present with a rash and


arthritis.

For both children and adults, the most


common disease caused by B19 is erythema

infectiosum, sometimes called "fifth disease" from a historical enumeration of the rashes. B19 is
spread by the respiratory route, and the rash appears after about 17 days. Its distinctive feature is the
red cheeks, with circumoral pallor. On the rest of the body it is a lacy, pink macular rash that fades
quickly, but may reappear after a warm bath. Adults with fifth disease may also have a rash and its
often accompanied by painful joint pain usually in the hands, feet, or knees. Some adults will have
107

MERP Medical Virology

only joint pain but no other symptoms. The joint pain usually lasts 1 to 3 weeks, but it can last for
months or longer.

B19 causes a biphasic disease (see figure below, left):


Phase 1: Infectious phase is due to influenza-like symptoms which is caused by viremia
Phase 2. Noninfectious phase is due to circulating immune complexes of Ab and virions which
results in erythematous rash, and arthritis. Adults usually feel more symptoms than children.

Slap cheek

B19 serology, clinical features and hematological changes

108

MERP Medical Virology

3. People with chronic anemia will get severe


reduction of RBCs, drop in hemoglobin (Hb),
and will often require a blood transfusion. This
is called aplastic crisis (a severe reduction or
blockage in RBC production in the bone
marrow). The shut-down in red cell production
is temporarily until the virus is eliminated by the
immune system, usually within 10 days.
However in people with anemia symptoms may
be more severe and include malaise, high fever
and joint pain.

Mechanism of spread of parvovirus within the


body.

4. Pregnant women - transmission of B19 virus from mom to fetus, may cause
hydrops fetalis (congestive heart failure and edema, right figure) due to rapid
death of fetal RBC precursors.

If mom has parvovirus Ab, then fetus is protected. These antibodies


protect the fetus and the infection does not result in congenital defects.

10% of primary maternal infections will result in fetal loss.

Hydrops fetalis stems from fetal anemia, since there is less oxygen
carrying RBCs, the heart needs to pump a much greater volume of blood
to deliver the same amount of oxygen. This can cause heart failure. It is a serious fetal
condition defined as abnormal accumulation of fluid in 2 or more fetal compartments, including
liver and/or heart.
Antibody to B19 is important for resolution and prophylaxis. There is no vaccine for this

infection. Laboratory diagnosis of acute B19 is also based on the presence of IgM antibodies.

109

MERP Medical Virology

LECTURE 12: PAPILLOMAVIRIDAE, POLYOMAVIRIDAE and ADENOVIRIDAE


Learning Objective
Describe clinical and biochemical features of Papilloma and Polyomaviridae families
Describe the various cells that Papilloma virus targets and the replication cycle in the cells
Describe the diseases that various types of HPVs can cause
Describe the mechanism behind cervical carcinoma
Describe how the viruses belonging to this group can immortalize cells
Describe the composition of HPV vaccine
Describe the population and diseases that Polyoma viruses cause
Describe clinical and structural features of Adenoviridae family
Describe how the viruses belonging to this group can immortalize cells
PAPILLOMAVIRIDAE AND POLYOMAVIRIDAE
Both of these viruses used to belong to the same family,
called Papovaviridae. Now they are now split into individual
families: PAPILLOMAVIRIDAE and POLYOMAVIRIDAE.
Since these viruses are very similar, we will characterize
the properties unique to them collectively and then describe
the disease caused by each family separately.

Human papovaviridae and their disease

Virus

Disease

Papillomavirus

Warts

Polyomavirus
BK virus
JC virus

Renal disease*
Progressive
multifocal
leukoencephalopathy *
*Disease in immunosuppressed patients

110

MERP Medical Virology

Unique properties of Papilloma and Polyoma:

Naked icosahedral capsid

Human papilloma viruses (HPV) have more


than 100 serotypes

Polyoma viruses have two main serotypes:


JC and BK

These viruses divide through early and late


staged.

Depending on the cell type, they can cause


lytic or latent infections in humans. Papilloma can also immortalize certain human cells.

These viruses promote cell division by binding and inactivating host cell factors that control cell
growth and cell death, specifically through inactivation of cellular pRB and E2F proteins.
o Normally, pRB protein restricts the cell's ability to replicate DNA by preventing its
progression from the G1 to S phase of the cell division cycle. pRB binds and inhibits
E2F family, blocking S phase.
o Normally, P53 plays a role in apoptosis and cell cycle arrest.

Papillomas E6 binds and inactivates p53 while E7 binds and inactivates pRB.
Polyoma T antigen inactivates both p53 and pRB.

Less prevalent human polyoma viruses include the KI, WU, and the Merkel cell carcinoma
polyomaviruses. Only recently has a polyomavirus been
associated with human cancers.

Papilloma viruses:
These viruses may be acquired by close contact,
sexual contact, and passage through infected birth canal.
They infect epithelial layer of skin and mucus membranes,
and their tissue tropism depends on their serotype.
The various viral replication stages depend on
epithelial cell growth and differentiation stage. Papilloma
viruses are persistent in basal layer keratinocytes and active
in differentiated keratinocytes. At least 40 identified HPV
types infect the genital tract. Studies found that if a college
woman has at least one different partner per year for four
111

Development of papilloma (wart). Human


papillomavirus infection promotes the outgrowth of the
basal layer, increasing the number of prickle cells of the
stratum spinosum (acanthosis). These changes cause
the skin to thicken and promote the production of keratin
(hyperkeratosis), thereby causing epithelial spikes to
form (papillomatosis). Virus is produced in the granular
cells close to the final keratin layer.

MERP Medical Virology

years, the probability that she will leave college with an HPV infection is greater than 85%

Mechanisms of
spread of
papillomaviruses
within the body.

Human Papillomaviridae diseases


1. Warts (Image, right)
Most types of HPV cause benign outgrowth called warts. Warts are
usually on hands and feet, may have a long incubation period of 3-4
months. The shape of these warts is dome-shaped or flat. They
often regress spontaneously or may be removed.
2. Benign head and neck tumors- these tumors are epithelial cell tumors of the oral cavity and tumors
of the larynx which may extend into trachea and the bronchi. Although they are benign, they could be
a problem in children due to obstruction of the airways. They are
causes by HPV6 and HPV11. Each year, about 300 infants are born
with the virus on their vocal cords because of maternal transmission
(there is a 1-3% chance that an infected mom will pass HPV to baby).
See Figure on the right.
3. Anogenital warts are genital warts (called condyloma acuminatum)
and occur on the squamous epithelium of perianal area and the
genitals. These are usually caused by HPV6 and 11 which
are not associated with malignancy in healthy people.
Sometimes HPV is hidden from the immune response
which results in a persistent infection.
4. Cervical carcinoma -HPV DNA can sometimes be found
integrated into the host DNA in the cells that became
cancerous. Cervical carcinoma begins as a genital tract
112

MERP Medical Virology

infection (a common STD) and is usually due to HPV-16 and HPV-18. In most cases these infections
will resolve on their own without causing cancer. In rare cases, infection does lead to cancer (usually
takes 15 years from original infection). HPV virus is the most common cause of cervical cancer and is
the most common infection of the reproductive tract. It is also the most common sexually transmitted
infection (STI) in US.
Cervical cancer is thought to develop in stages from
neoplasia to severe dysplasia and cervical carcinoma.
Perinuclear cytoplasmic vacuolization called
koilocytosis is the first indication of abnormality.
Different stages of cervical cancers are also identified
by cytological changes. It is more common in women
than men, and is usually asymptomatic in both. When
symptoms are seen women experience irregular,
intermenstrual (between periods) or abnormal vaginal
bleeding, pelvic pain, fatigue, weight loss, loss of
appetite, as well as vaginal discomfort or odourous
discharge.
Progression of human papillomavirus
(HPV)-mediated cervical carcinoma. a.
HPV infects and replicates in the basal
epithelial cells of the cervix, maturing and
releasing virus as the epithelial cells
progress through terminal differentiation.
Growth stimulation of the basal cells may
or may not produce a wart. b. In some
cells, the circular genome integrates into
host chromosomes, inactivating the E2
gene. c. Expression of the other genes
without virus production stimulates growth
of the cells and possible progression to
neoplasia.

113

MERP Medical Virology

Clinical Syndromes Associated with Papillomaviruses

Outcomes of genital infection with HPV. About 70% of cervical cancers are
caused by HPV infections with two HPV types, 16 and 18. Although most
cervical cancers are caused by HPV 16 and 18, infection with these
serotypes most often leads to resolution.

Vaccine: A recombinant vaccine (Gardasil) is available for serotypes 6, 11, 16, 18. In 2015 Gardasil 9
was introduced. Covering nine HPV types, five more HPV types than Gardasil, Gardasil 9 has the
potential to prevent approximately 90 percent of cervical, vulvar, vaginal and anal cancers. It is
approved for the prevention of cervical, vulvar, vaginal and anal cancers caused by HPV types 16,
18, 31, 33, 45, 52 and 58, and for the prevention of genital warts caused by HPV types 6 or 11.
Gardasil 9 adds protection against five additional HPV types31, 33, 45, 52 and 58 which cause
approximately 20 percent of cervical cancers and are not covered by previously FDA-approved HPV
vaccines.

Diagnosis:
Warts can be diagnosed microscopically based on histological appearance. Hyperplasia of the prickle
layer of the skin and excess keratin production is visible. For specificity, PCR is the method of choice.
Tissue specimen such as cervical swab (PAP smear) can be used for PCR.

114

MERP Medical Virology

Polyoma viruses:
Two main types exist: BK and JC, both of which
enter through inhalation of infectious particles via
the respiratory tract. They are then spread by
viremia to other organs, predominantly the
kidneys. While both can establish a latent infection
in the kidney they are normally blocked from
replication by the immune system of
immunocompetent hosts. The infections with BK
and JC viruses in immunocompetent individuals

Mechanisms
of spread of
polyomaviruses within
the body. CNS,
Central
nervous
system; PML,
progressive
multifocal
leukoencephal
opathy.

are asymptomatic. However in


immunocompromised people, these viruses can
cause severe disease.

Human diseases
JC virus causes progressive multifocal leukoencephalopathy (PML) which is a subacute
demyelinating disease in people with severe immunosuppression (often seen in AIDS patients). To do
so, JC crosses the blood-brain barrier and replicates in the endothelial layer of the capillaries. In the
brain lytic infection of oligodendrocytes causes demyelination. This causes multiple neurological
symptoms such impaired coordination and paralysis.
PML is diagnosed by histological examination of the brain tissue in biopsy.

One can see areas of demyelination surrounded by oligodendrocytes and lesions in the white
matter (See Figure on the bottom left).

To detect the virus in brain tissue, PCR can also be used.

astrocyte

115

MERP Medical Virology

BK virus

This infection is usually asymptomatic, but with impairment of the cellular immune system the
virus can reactivate and lead to tissue damage.

In recipients of bone marrow and solid organ transplants, the reactivation can be associated with
disease in urinary tract and kidneys.

BKV was first discovered in 1971 from the urine of a kidney transplant recipient who had
developed ureteral stenosis (narrowing of the ureteral lumen) 4 months after transplantation.

Specific identification of the virus is now routinely done in organ transplant recipients using PCR
analysis or immunofluorescence of BK in the urine.

Biopsy of the kidney can also indirectly reveal the presence of BK virus.

ADENOVIRIDAE
Unique Properties

Naked icosahedral capsid, linear, non-segmented, ds DNA


genome

The capsomers are composed of hexons and penton fibers.


Penton fibers are found at each of the vertices of the
icosahedron

Human adenoviruses are grouped A through G by DNA


homologies and by serotype (more than 55 human types) according to various properties
including the oncogenicity of viruses in newborn rodents.

Serotype is mainly a result of differences in the penton fibers which determine the nature of tissue
tropism and disease.

E1A binds to and inactivates a cellular protein, pRB, the product of the retinoblastoma gene.
Remember that pRB prevents excessive cell growth by inhibiting cell cycle progression until a cell
is ready to divide.

E1B binds to and inactivates p53. Remember that p53 is another tumor suppressor protein which
initiate apoptosis and inhibits the cell cycle.
116

MERP Medical Virology

Spread by aerosol, close contact, fecal-oral, finger-to-eye, inadequately chlorinated swimming


pools, fomites.

Infects mainly mucoepithelial cells in the respiratory tract, GI tract, conjunctiva, and cornea

Usually causes direct cell damage by lysis.

Can be persistent in some tissues such lymphoid tissues (tonsils, adenoids, Peyers patches).

The toxic activity of the penton base protein can result in inhibition of cellular mRNA transport and
protein synthesis, cell rounding, and tissue damage.

Antibody is important for prophylaxis and resolution.

Human diseases

Mechanism
of
adenovirus
spread
within the
body.

Rare

Adenovirus infections are very common, most are asymptomatic. Most people have been
infected with at least 1 type by the age of 15. Virus can be isolated from the tonsils and adenoids that
have been surgically removed, indicating latent infections.
There are many different serotypes that cause various clinical diseases. Collectively these
viruses can infect and replicate in epithelial cells of the respiratory tract, GI (because they are
resistant to acid), eyes, liver. Some persist for years in adenoids and are shed after initial infection.

117

MERP Medical Virology

Examples of diseases include:


1. Acute febrile pharyngitis is a disease characterized
by cough, nasal congestion, sore throat, fever and
headache. It is usually in infants and children and is
a common respiratory disease.
2. Pharyngoconjunctival fever- This disease is similar
to acute febrile pharyngitis but also conjunctivitis is
present. It frequently presents as outbreaks in
swimming pools in summer camps.
3. Acute respiratory disease occurs in military recruits in epidemic form. People with this disease get
fever, fatigue, cough and pharyngitis. (Link to article: http://www.cnn.com/2007/HEALTH/conditions/12/19/killer.cold/index.html)
4. Pneumonia is often a complication of acute respiratory disease, in children 10% of pneumonias
are due to Adenovirus.
5. Eye infections- These could present as serious infections such as epidemic keratoconjunctivitis.
Both palpebral and bulbar conjunctivas (membranes that cover inner layer of eyelid and outer
layer of the eye) are inflamed. It is very contagious and is characterized by keratitis (inflammation
of cornea that leaves round subepithelial opacities in the cornea for up to 2 years).

6. Gastroenteritis and diarrhea- two serotypes account for 5-15% of cases of viral gastroenteritis in
kids. The diarrhea caused by Adenovirus often persists longer than diarrhea caused by other
viruses. Although many viral subtypes can replicate in intestinal cells and are present in stools,
most are not associated with disease.
Diagnosis:
These are usually diagnosed clinically but virus can be isolated from stool or throat swab and
grown on a variety of epithelial cells. One will see characteristic cytopathic effects: rounding and
clustering of swollen cells. Adenovirus also causes characteristic intranuclear inclusion bodies.
Clinical sample can also be used directly and assayed by immunofluorescence, and hemagglutination
tests. PCR and electron microscopy are esp. useful for diagnosing GI infections.
118

MERP Medical Virology

LECTURE 13: HERPESVIRIDAE


Learning Objectives
Describe clinically and structurally the family of Herpesviridae. List the features that are common to all
Herpesviridae. Differentiate clinically among the different disease caused by Herpesviridae.

Describe HSV disease/clinical presentation


Describe where HSV causes latency/recurrence
Describe the effect of HSV on the central nervous system
Describe the role of HSV in neonatal disease
Describe Tzanck smear and inclusion bodies
Describe VZV disease/clinical presentation (specifics about the rash and how it differs
from other rashes)
Know where VZV causes latency/recurrence (chicken pox versus shingles)
Describe VZV diagnosis and prevention
Describe the clinical features of diseases caused by EBV
Know where EBV causes latency and recurrence
List the target cells for EBV
Describe the importance of T cells in control of all Herpes (including EBV)
Describe the link between EBV and Burketts lymphoma
Describe EBV diagnosis (detail the importance and time course of heterophile antibodies,
atypical lymphocytes, EBV specific antibodies)
Describe CMV disease mechanism and list populations at risk
Describe the mechanism of CMV spread
Be able to describe the characteristic histology of CMV
HHV6 (roseola) describe the rash and differentiate between this rash and all the other
childhood rashes

Unique Properties of Herpes viruses

Herpesviruses are large enveloped


viruses with icosadeltahedral capsids
and double-stranded DNA genomes.

Herpesviruses encode many enzymes,


including their own DNA d. DNA
polymerase, that promote viral DNA
replication and are good targets for
antiviral drugs.

DNA replication and capsid assembly occurs in the nucleus.

Virus is released by exocytosis which may lead to cell lysis. The virus may also spread from cell
to cell via syncytia.

While Herpes viruses cause lysis of certain cells, Herpes family will cause persistent infections in
which certain cells harbor the virus long term.

Herpesviruses can also cause immortalizing infections (ex. Epstein-Barr virus).


119

MERP Medical Virology

Herpesviruses are ubiquitous.

Cell-mediated immunity is required for control.

Herpes replication cycle:


Fusion of Herpes and membrane

Attachment of the viral


attachment proteins to host
receptors mediates fusion virus
into the host cell (steps 1- 4 in
Figure above and Figure to the
right). Fusion with the plasma
membrane releases the core and
the tegument proteins into the
host cytoplasm (steps 4, 5,
right).
The capsid is transported to the
nuclear pore where viral DNA is
released into the nucleus (steps
6, 7).
Host RNA polymerase II
mediates transcription of the
immediate early and early viral
mRNA which is translated in the
cytoplasm (9-12). These encode
proteins involved in replication
of the viral DNA.
Genome replication occurs by
bidirectional replication (13).
Next the host transcribes and
translates late genes, encoding
structural proteins (14-17).
Virus assembles in nuclear viral
factories (19) and buds through
(19a, b) or is exocytosed through
(20a, b) the inner lamella of the
nuclear membrane.
Golgi can be modified by the
insertion of herpes glycoproteins
(19d). Virus travels through
Golgi (21).
Virus is exocytosed at the
plasma membrane (22, 23).

120

MERP Medical Virology

Herpes viruses can trick immune system by:

Direct infection of immune cells

Inhibition of T cell antigen recognition:

Inhibition of cell-surface class I MHC expression

TAP inhibition (transporter associated with antigen presentation)

Inhibition of natural killer cell attack

Inhibition of apoptosis in infected cell (may interfere with cytotoxic T cell attack)

Sequestration of chemokines

Encoding cytokine and chemokine homologs

Complement- and interferon- interacting molecules


Types of infections caused by
Human Herpes viruses

121

MERP Medical Virology

HSV 1 and 2
Both HSV-1 and-2 cause lesions (clear vesicle
surrounded by red area).

Infection is
initiated by
breaks in the
skin or
through
mucosal membranes. Although both HSV1 and HSV2
are common, they can cause serious disease if they
spread to eye or brain, or if they infect
immunosuppressed people or neonates. These viruses
can be latent in trigeminal ganglia (HSV1) or sacral
ganglia (HSV 2).

122

MERP Medical Virology

Exercise: Look at the figure on the right, which is another depiction of Herpes cell cycle and work
through each step on your own.
Replication of herpes simplex
virus

Review of replication cycle (right figure):

The virus binds to specific receptors

After binding, the virus fuses directly with


the plasma membrane (no entry into low
pH endosomes/lysosomes is necessary).

The nucleocapsid then delivers the DNA


genome to the nucleus.

Transcription and translation occur in


three phases: immediate early, early, and
late. Immediate early proteins promote
the takeover of the cell; early proteins
consist of enzymes, including the DNAdependent DNA polymerase; and the late
proteins are structural and other proteins,
including the viral capsid and
glycoproteins.

The genome is replicated before


transcription of the late genes.

Capsid proteins migrate into the nucleus,


assemble into icosadeltahedral capsids, and are filled with the DNA genome. The capsids filled
with genomes bud through the nuclear and endoplasmic reticulum (ER) membranes into the
cytoplasm, acquire tegument proteins, and then acquire their envelope as they bud through the
viral glycoprotein-modified membranes of the trans-Golgi network.

The virus is released by exocytosis or cell lysis.

It also appears to be able to pass through intercellular junctions and thereby spread from cell to
cell.

123

MERP Medical Virology

Disease mechanism for HSV (right figure):

Clinical course of genital herpes infection. The time course and


symptoms of primary and recurrent genital infection with herpes simplex
virus type 2 are compared. Ann Intern Med 98:958-973, 1983

The virus is spread in bodily fluids


such as saliva and vaginal secretion.

Disease is initiated by direct contact


with the infected fluid and usually
begins in the mucoepithelium of the
oral or genital tract.

Viral infection causes direct


cytopathologic effect (CPE) on the cell.
CPE is a direct change in the host cell
which may range from lysis of the cell
to the formation of multinucleated giant
cell (syncytia).

HSV 1 and 2 lesions usually begin with


a clear vesicle containing infectious virus with a red (erythomatous) base. These are usually
painful.

Patient may experience tender regional lymph nodes during lesion eruption.

Pus-containing (pustular), encrusted lesions and ulcers may develop.

Systemic symptoms are due to interferon secretion by the infected cells and recruited immune
cells.

Although some released virions will be blocked by antibody, some virions can avoid antibody
by cell-to cell spread, which helps maintain the infection.

Cell-mediated immunity is required for resolution

In most infected individuals the virus establishes latency in sensory neurons.

The virus can be reactivated by stress or immune suppression.

Recurrent disease is usually shorter in duration and is more abrupt.

In addition to the direct damage of the infected cells by the virus, cell-mediated
immunopathological effects contribute to symptoms

124

MERP Medical Virology

Examples of human diseases


1. Herpetic gingivostomatitislesions in the mouth, palate,
pharynx, gingivae (gums of
mouth) and tongue, often
seen in kids
2. Cold sores, fever blisterslesion in corner of mouth,
next to lips
3. Herpes pharyngitis- severe
sore throat, fever, chills, headache, and malaise, usually in adults
4. Herpetic whitlow- infection of the finger through a cut in the skin (often in nurses who attend HSV
patients or thumb sucking children)
5. Eczema herpeticum- severe disseminated HSV skin infection acquired by kids with underlying
eczema condition. It causes spread of infection from skin to adrenal glands, liver and other
organs.
6. Genital herpes- genital lesions accompanied by itching, pain and in females vaginal discharge
7. Herpes encephalitis- (usually HSV-1) - lesions on temporal lobes leading to destruction of the
lobe, increased red blood cells in CSF and seizures
8. Herpes meningitis (usually HSV-2) - a complication of genital HSV infection. It

Neonatal HSV

usually resolves by itself.


9. Neonatal HSV- can be acquired in utero, but more commonly through the
passage of infant through vaginal canal, or postnatally. Infection of the
neonates mucoepithelium will result in a disseminated infection in the liver,
lung, CNS which can result in severe disabilities.
10. Keratoconjunctivitis- From the skin, the virus reaches the cornea via the sensory ophthalmic
branch of the trigeminal nerve, it then causes infection of conjunctiva, corneal ulcers and
vesicles in the eyelids.

125

MERP Medical Virology

Latency with HSV 1

After a primary infection, Herpes stays dormant in peripheral nerve ganglia. Following one of a
variety of stimuli, vesicles erupt on the muco-cutaneous junctions of the nose or mouth. These
are more localized than the primary infection and heal more rapidly (7-10 days). The eruption is
often preceeded by paraesthesia of the involved area.

Locations of latent HSV-1 and HSV-2

126

MERP Medical Virology

Diagnosis

Direct analysis of clinical sample will show characteristic CPE in a Tzanck smear (Tzanck
smear is scraping of base of lesion) or in a PAP smear or biopsy

CPE includes syncytia (Figure below: A), ballooning cytoplasm, and Cowdry Type A
intranuclear inclusion (Figure below: B)

Tissue sample can also be stained for viral antigen using IF

Viral DNA can be amplified from the sample using PCR


A

Syncytia

Cowdry Type A

Treatment:

To treat Herpes, Acyclovir is administered. This is a modified nucleotide (with a guanine base) that is
phosphorylated inside a cell by HSVs thymidine kinase. Although the drug is absorbed by all cells,
since the nucleotide can only be phosphorylated by a viral enzyme, acyclovir functions specifically in
the infected cells. The last 2 phosphates are added to acyclovir by the by hosts enzymes. Once
acyclovir is converted to the triphosphate form it is
incorporated by viral DNA d. DNA polymerase. If, by
chance hosts polymerase gets access to Acyclovir
triphosphate it will have 200 fold lower affinity for it than
the viral DdDpol. This means that the host cell can still
survive despite being treated with acyclovir. Valtrex is a
modified acycolvir with an added valine group. Valtrex is
better absorbed by GI.

127

MERP Medical Virology

Varicella-zoster (VZV)
Disease mechanism for VZV

VZV causes chicken pox (varicella) and upon


recurrence it causes zoster (shingles).

Unlike HSV, VZV spreads predominantly through


the respiratory tract from where it enters
circulation and causes viremia and lesions all
over the body. The lesions are characteristic of
the chicken pox childhood rash.

VZV causes a lytic infection in the epithelial cells


of the lungs and keratinocytes, these cells are the

Chicken pox
Shingles if
reactivated

source of contagion. The other cells that host the


virus are not lysed by the virus.

In addition to lysing cells, VZV can cause syncytia

Interferon-, natural killer cells and T cells limit

Mechanism of spread of varicella-zoster virus (VZV) within the


body. VZV initially infects the respiratory tract and is spread to the
reticuloendothelial system and T cells and then by cell-associated
viremia to the skin.

the spread of the virus in the tissue, but antibody is important for limiting the viremic spread of
VZV. Passive immunization with varicellazoster immune globulin (VZIG) within 4 days
of exposure is protective. Cell-mediated
immunity is essential for resolving the
disease. The virus causes more disseminated
and more serious disease in the absence of
cell-mediated immunity. The more
pronounced immune response in adults is
Time course of varicella (chickenpox). The course in young children, as
presented in this figure, is generally shorter and less severe than that in
adults.

responsible for the more severe symptoms


seen in adults as compared to children.

VZV is latent in the dorsal root and cranial nerve ganglia (groups of sensory nerve cell bodies
located outside of the central nervous system and in the brain).

Upon reactivation, the virus is released along a particular neural pathway and infects the skin
causing a rash along the dermatome.

Shingles results from immune-suppression and presents as lesions along a dermatome.

128

MERP Medical Virology

Pathophysiology of human diseases

1. Varicella (chicken pox) - a childhood exanthema


Symptoms:

14 days incubation period

Fever and rash appear at the same time.

Presents as a maculopapular rash (vesicular rash):


o Macules are areas of skin discoloration that are usually
less than about half an inch (one cm) in diameter. They
are neither elevated nor depressed
o Papules are small, round elevations of the skin that don't
appear to contain any fluid.
o Rash becomes pustular and forms crusts
o Lesions last for about a week

Rash is more severe on the trunk than extremities

Rash present on the scalp

In adults, a complication is interstitial pneumonia (due to


inflammation at the primary site of infection).

Live chicken pox vaccine is now routinely given to kids.

2. Herpes zoster (shingles)

129

MERP Medical Virology

Reactivation of the VZV causes it to spread along a nerve tract, causing burning sensation prior to
rash eruption.

The typical rash appears in 2 to 3 days,


after the virus has reached the skin

Rash consists of red patches of skin with


small blisters that look very similar to early
chickenpox.

Rash often increases over the next 3 to 5


days.

Blisters break forming small ulcers that


begin to dry and form crusts.

The crusts fall off in 2 to 3 weeks, leaving


behind pink healing skin.

Lesions appear along a single dermatome


and are only on one side of the body (unilateral).

The trunk is most often affected, showing a rectangular belt of rash from the spine around one
side of the chest to the sternum.

Older patients may experience postherpatic neuralgia which is a chronic pain syndrome
affecting the area innervated by infected nerves.

Live vaccine available for shingles.

130

MERP Medical Virology

Diagnosis: Clinical presentation:


Cowdry type A intranuclear
inclusion body, as well as syncytia
in cells from the skin lesion can be
used to make diagnosis. Skin
lesion scraping can be analyzed
by direct fluorescent Ab to
membrane antigen. Serological
tests that detect Ab to VZV can be
used to screen people for
immunity to VZV. These tests are
usually ELISA or IF.
The figure on the right shows
various ways in which an
organism can infect the. The
orange layer shows epidermis, the
yellow layer is the dermis. A
pathogen can get to the skin from
the systemic infection in the blood
(ex. VZV), they can get to the skin
directly by being introduced into
the epithelium, and stay locally in the tissue (ex. HPV). Some pathogens are brought to the skin as
part of an immune complex, which then stimulates inflammation and a rash (ex. Hepatitis B).
Epstein-Barr Virus (EBV)
Disease mechanism of EBV

EBV is shed from salivary glands


into saliva, so sharing of
toothbrushes and cups initiates
infection.

In a susceptible individual, EBV from


infected saliva enters oral epithelial
cells and then spreads to nave
resting B cells in the tonsils.

EBV uses complement receptor


called CD21 as a means of
attachment to epithelial cells of the
oropharynx and nasopharynx as
well as attachment to B cells.

EBV causes lytic infection of epithelial cells and in some B cells.

131

MERP Medical Virology

In epithelial cells lytic replication often directly


follows viral entry. In most B cells, however,
lytic replication only takes place after
reactivation of the virus from latency.

EBV establishes life-long latency in memory


B cells.

EBV may be reactivated when the memory B


cell is activated and turns into plasma cell
(especially in the tonsils or oropharynx) which
then may cause the shedding of virus into the
saliva.

Although EBV may not directly kill nave B


cells, T cells kill and limit B cell outgrowth. T cells are required for control of infection.

Elevated T cell response (lymphocytosis) contributes to symptoms of infectious mononucleosis.


o Asymptomatic shedding is a characteristic of EBV.
o More than 90% of EBV-infected people intermittently shed the virus for life.

EBV can rarely promote immortalization of B cells.


o Continual activation of B cells by EBV (and the absence of T cell response) causes
lymphoma
o EBV is associated with lymphoma in immunosuppressed people and African children
(Burkitts lymphoma) and nasopharyngeal carcinoma in China.

Human Diseases
EBV disease results from either an overactive immune response, causing infectious mononucleosis
or inadequate immune response which can lead to lymphoma.

Over-reactive immune response:


1. Heterophile Ab-positive infectious mononucleosis
This disease is usually milder in children than adults.
Classical symptoms are: lymphadenopathy, exudative
pharyngitis and high fever. Splenomegaly may be
present. A great amount of energy is required to power
the T-cell response, leading to great fatigue (Figure,

132

MERP Medical Virology

right). Rarely, EBV can cause complications like laryngeal obstruction, rupture of spleen and
neurological disorders.

Under-reactive immune response:


1. Lymphoproliferative disease
On infection with EBV, people lacking T cell immunity (such as people who are undergoing
immunesuppressive therapy, people with congenital deficiencies of T-cell function) or people with
chromosomal translocation of the c-MYC oncogene may get lymphoproliferative disease.
Specific B cell lymphoma of the jaw and
face called African Burkitts lymphoma is
endemic in children living in malarial regions
of Africa. It usually starts as tumors of the
jaw (mandibular or maxillary masses are
common) which causes facial bone
deterioration and loosening of the teeth. The
swelling of the lymph nodes in the neck and
below the jaw is common. The lymph nodes are non-tender and rapidly
growing. Other symptoms include night sweats, unexplained high
temperatures and weight loss.
Nasopharyngial carcinoma is endemic in Asia. It occurs in adults, and unlike Burkitts
lymphoma (which is of lymphocyte origin) has epithelial origin.

133

MERP Medical Virology

Diagnosis of Mononucleosis
Mono is diagnosed on symptoms that include
a triad of: fever, lymphadenopathy and exudative
pharyngitis (and a fourth possible symptom splenomegaly). Lab findings include atypical
lymphocytes (called Downy cells, see bottom
figure) which are overly activated T cells.
Additionally, the presence of lymphocytosis (a
sharp increase in mononuclear cells) also
indicates EBV.
An additional test, known as the Monospot
test, detects heterophile antibodies which are
antibodies characterized by many different

Clinical course of infectious mononucleosis and laboratory


findings of those with the infection. Epstein-Barr virus infection
may be asymptomatic or may produce the symptoms of
mononucleosis. The incubation period can last as long as 2 months.
EA, Early antigen; EBNA, Epstein-Barr nuclear antigen; VCA, viral
capsid antigen.

specificities. They include antibodies that


recognize antigens horse and sheep red blood
cells (see figure on bottom right). The heterophile
antibodies are transient antibodies of IgM isotype.

The findings of EBV-specific antibodies in the blood of the patient confirm diagnosis. EBV viral
DNA can also be detected directly by PCR in patient samples such as saliva, peripheral blood and
biopsies.

Monospot test. A few drops of the patient's


plasma are mixed with RBCs from a goat or
horse or sheep. The patients plasma on the
left (but not on the right) contains
heterophile IgM antibodies that agglutinate
sheep or horse red blood cells.

134

MERP Medical Virology

Cytomegalovirus (CMV)

CMV can cross the placenta and is the


most common viral cause of congenital
defects.

In US, about half of expectant mothers


have not been infected with CMV prior
to conceiving.

About 1% to 4% of these previously


uninfected mothers have primary (or
first) CMV infection during their
pregnancy.

About one third of women who become


infected with CMV for the first time

Outcomes of cytomegalovirus (CMV) infections. The outcome


of CMV infection depends very heavily on the immune status of
the patient.

during pregnancy pass the virus to their unborn babies.

Women who had CMV before getting pregnant can also pass the virus to their unborn babies, but
this is less common.

Although about 1 in 150 babies in US are born with CMV infection, only 1 in 800 children in US
are born with symptoms of CMV. This is due to the protective role of maternal IgG.

CMV causes mild disease in children and adults, but very serious disease in immunocompromised
patients.

Transmission of CMV occurs via blood, organ transplants, and other secretions: urine, saliva,
semen, cervical secretions, breast milk, and tears.

CMV establishes a lytic infection in fibroblasts, epithelial cells and macrophages, and a latent
infection in lymphocytes, macrophages and bone marrow stromal cells.

Disease mechanism of CMV

CMV infects epithelial cells and secretory cells, resulting in a persistent infection and viral
shedding.

Infection of the genitourinary system leads to clinically inconsequential viruria. Despite ongoing
viral replication in the kidney, renal dysfunction is rare.

Cell-mediated immunity is required for control and contributes to symptoms.

CMV generally causes subclinical infection and may be shed asymptomatically.


135

MERP Medical Virology

Having no evidence of virus in the bloodstream has a high negative predictive value for CMV
disease.

Human diseases
1. Congenital defects: 0.5-2.5% of infants in US are infected with CMV prior to birth but do not
necessarily have the disease. Less than
10% of the infected newborns show
birth defects including
hepatosplenomegaly with cirrhosis, and
microcephaly with mental or motor
retardation as well as hearing loss.
Other disease signs include:
thrombocytopenia, microcephaly,
intracerebral calcification, and rash. The
congenital disease is sometimes
referred to as cytomegalic inclusion disease.

Perinatal infection: About 20% of women harbor CMV in the cervix. These women often
experience reactivation during pregnancy or labor. Infants may acquire CMV through birth canal
and infected cervix during delivery or through breast milk after delivery. This usually does not
result in a serious disease. Infants can also acquire CMV through transfused blood, which may
result in pneumonia and hepatitis.

Infection of children and adults: CMV is a sexually transmitted disease, and because it is often
asymptomatic, many adults are infected with the virus. In some cases the infection results in
heterophile-negative mononucleosis, with similar symptoms to EBV, but less severe pharyngitis
and lymphadenopathy.

Infection in blood and organ recipients: transmission via transfusion and transplantation can
result in CMV infection. Transmission in blood is usually asymptomatic, whereas transmission in
organ transplants may result in pneumonia and mild hepatitis. Blood is screened for CMV, and
CMV negative blood is reserved for transfusion to pregnant women and immunocompromised.

Infection of immunocompromised host. This is a serious problem. In this population, CMV


causes symptomatic, systemic infection which can result in recurrent disease. In
immunocompromised, CMV can cause pneumonia, hepatitis, retinitis, encephalitis, colitis.

136

MERP Medical Virology

Diagnosis

Blood, bronheoalveolar lavage* or urine can be tested for viral


antigen (or DNA) using IF, ELISA or PCR.

Seroconversion (test for IgM or increasing titer of IgG) is usually


an excellent marker for primary CMV infection.

CMV can also be grown in culture and then examined for CPE

Histological hallmark of CMV is an enlarged cell that contains


central owls eye basophilic intranuclear inclusion body. Such a cell can be found in any
tissue of the body, in urine, and are thought to be of epithelial origin

*Bronheoalveolar lavage- diagnostic procedure conducted by placing a small fiberoptic scope into
the lung of a patient, and injecting sterile water into the lung and removing the fluid. The sterile
solution removed contains secretions, cells, and protein from the lower respiratory tract.

137

MERP Medical Virology

Human Herpes Virus 6 (HHV6)

HHV6 causes roseola and is another example of a childhood exanthema.

Infection occurs early in life (before 3 years of age).

Virus replicates in the salivary glands, is present in saliva of most adults and is spread to children
easily by oral secretions.

HHV6 also infects lymphocytes, monocytes, epithelial and endothelial cells.

Disease is characterized by rapid onset of high fever, followed by generalized rash which lasts 2448 hr. The rash usually spares the face.

Disease resolves without complications, however the infection remains persistent for life. The
persistency is usually in T cells.

In adults, it may cause very mild mononucleosis and lymphadenopathy.

Time course of symptoms of exanthem subitum (roseola) caused by human herpesvirus 6 (HHV-6).

138

MERP Medical Virology

LECTURE 14: HEPADNAVIRIDAE


Learning Objectives
Be able to describe clinically and structurally the causative agent of Hepatitis B.

Describe the unique features of this virus including its replication cycle (role of
RTase).
Describe the significance of HBsAg, HBcAg, HBeAg, HBsAb, HBcAb, HBeAb
Interpret the serological charts for acute and chronic Hepatitis B infected individuals
Describe Hepatitis B as a cause of PHC
Describe the vaccine composition and its administration
Describe the role of passive immunization in Hepatitis B

Hepatitis B
Unique properties:

HBV is less sensitive to detergents than


other enveloped viruses

It is spread by blood, sexual intercourse,


saliva, breast milk, transfusion and
needle stick injury.

HBV has a strict liver tropism.

Virions (also known as "Dane particles")


contain a circular, partially dsDNA
genome.

HBV encodes and carries reverse transcriptase.

It replicates through circular full length RNA intermediate.

HBV genome can integrate into host genome.

10% of infected people will have chronic infection with shedding.

Persistent regeneration of the liver can result in cancer.

HBsAg subunit vaccine available.

As treatment for high risk newborns-passive immunization is given.

Important HBV Antigens


o HBsAg; surface (coat) protein. It is produced in excess to what the virion need and is released as
small spheres and tubules.
o HBcAg; inner core protein, structural protein derived from pre-core-mRNA
139

MERP Medical Virology

o HBeAg; secreted protein derived from pre-core-mRNA and is made in a 1:1 ratio to core Ag;
function unknown
Replication cycle of Hepatitis B (Figure: right)
1. Virus attaches to liver cells via a receptor.
2. It enters mostly likely through endocytosis.
3. Partially dsDNA completes synthesis either
in the cytoplasm by viral reverse
transcriptase (RTase) or travels to the
nucleus and hosts DNA dependent DNA
polymerase (DdDpol) completes its
synthesis.
4. Cellular RNA polymerase II transcribes a
longer-than-genome-length mRNA called
the pregenome (or replication
intermediate) and shorter, subgenomic
transcripts, all of which serve as mRNAs.
5. The shorter viral mRNAs are translated by
ribosomes attached to the cell's
endoplasmic reticulum. These proteins are
destined to become HBV surface antigens
in the viral envelope.
6. The pregenome mRNA is translated to
produce a reverse transcriptase
polymerase protein P which then binds to
a specific site at the 3' end of its own
transcript (RNA-P complex). The

Replication of hepatitis B virus (HBV). After entry into the


hepatocyte and uncoating of the nucleocapsid core, the
partially double-stranded deoxyribonucleic acid (DNA) genome
is completed by enzymes in the core and then delivered to the
nucleus. Transcription of the genome produces four
messenger RNAs (mRNAs), including an mRNA larger than
the genome (3500 bases). The mRNA then moves to the
cytoplasm and is translated into protein. Core proteins
assemble around the 3500-base mRNA, and negative-sense
DNA is synthesized by a reverse transcriptase activity in the
core. The ribonucleic acid (RNA) is then degraded as a
positive-sense (+) DNA is synthesized. The core is enveloped
before completion of the positive-sense DNA and then
released by exocytosis. HBsAg, hepatitis B surface antigen.

pregenome mRNA also codes for the core


and e antigens.
7. The RNA-P protein complex is packaged into the core and while packaging continues reverse
transcription takes place.
8. The viral DNA is synthesized by the viral reverse transcriptase using the encapsidated
pregenome as template. Remember that RTase has three functions: RNA to DNA synthesis,

140

MERP Medical Virology

DNA to DNA synthesis and RNAse H activity during which it destroys RNA in a RNA:DNA
hybrid.
9. The newly synthesized nucleocapsid acquires the viral envelope in the endoplasmic reticulum
(ER) and Golgi apparatus, with release of mature virions from infected cell by exocytosis.
10. This mode of virus production does not result in cell death, thus contributing to chronic
infection of the liver.
Clinical Features

Incubation period of HBV is 2 - 5


months.

HBV causes a more severe disease


than Hepatitis A.

Asymptomatic infections occur


frequently.

Infection is parenterally transmitted


(taken into the body in a manner other
than through the digestive tract).
Transmission occurs via blood (blood
transfusions, serum products, sharing
of needles, razors), tattooing, renal
dialysis, organ donation, sexual
intercourse, vertical transmission
(perinatal transmission from a carrier
mother to her baby during delivery). In vertical transmission for Hepatitis B the virus is passed
from mother to baby during delivery, not prior to
delivery.

Pathogenesis
The virus replicates in the liver and the whole virus, as
well as excess viral surface protein are shed in large
amounts into the blood. Viremia is prolonged and the
blood of infected individuals is highly infectious.
The diagram on the right illustrates the
transmission of Hepatitis B. After the introduction of
141

MERP Medical Virology

the virus into the blood, it infects the liver. If an individual has previously formed antibodies against
the virus (ex. through vaccination), they will block the transmission from blood to liver. Without
previous vaccination or previous exposure and resolution, the virus is shed back into the blood from
the infected liver and then is excreted into various bodily fluids, completing the cycle.
Clinical presentation
A. Acute infection with resolution
Acute symptoms iinclude: fever, malaise (95% of patients), nausea (80% of patients), anorexia
(90% of patients), vomiting, abdominal pain (right upper quadrant, 60% of patients) and rash in 15%
of the patients. Classical icteric symptoms (jaundice, dark urine, pale stool) follow. In hepatitis, due to
inflammation of the biliary ducts there is a defect in secretion of bilirubin into biliary tract which causes
bilirubin accumulation in the blood resulting in jaundice. In healthy individuals, liver conjugates
bilirubin and when conjugated bilirubin is secreted into the bile tract, it is converted to dark colored
stercobulin by intestinal bacteria. This causes stool to have its characteristic dark color. When liver
does not function properly, conjugated bilirubin does not reach the intestines, resulting in pale stool.
Accumulated conjugated bilirubin also gives urine a darker yellow color when the kidneys filter the
bilirubin rich blood.
In few cases (1%) fulminant hepatitis (severe liver damage) may occur. Because of the excessive
amount of HBsAg produced, hypersensitivity reactions caused by immune complex depositions, may
promote rash, arthritis, glomerulonephritis as well as other symptoms of Type III hypersensitivity.

142

MERP Medical Virology

Serological events in acute hepatitis:


Viral antigens:
1) Surface antigen (HBsAg) is secreted in excess into the blood as 22 nm spheres or tubules. Its
presence in serum indicates that virus
replication is occurring in the liver. It can be
detected very early during an infection, even
prior to symptoms.
2) 'e' antigen (HBeAg) secreted protein is
shed in small amounts into the blood. Since
its levels correlate to the amount of viral
progeny in that cell, its presence in the blood
indicates that a high level of viral replication
is occurring in the liver.
3) Core antigen (HBcAg) core protein is not
found in blood.
The serologic events associated with the typical course of acute
hepatitis B disease.

4) During the window period there is no


detectable HBsAg or HBsAb because these
molecules are complexed together.

Viral antibodies:
1) Surface antibody (anti-HBs) becomes detectable late in convalescence, and indicates immunity
following infection. It remains detectable for life and is not found in chronic carriers. Note that in
vaccinated people only HBsAb will be present.
2) e antibody (anti-HBe) becomes detectable as viral replication falls. It indicates low infectivity in a
carrier.
3) Core IgM rises early in an infection and indicates recent infection. Its peak follows symptoms of
hepatitis because the release of core antigen from the liver cell is necessary for the antibodies to be
produced. The killing of liver cell by CD8 T cell causes symptoms.
4) Core IgG rises soon after IgM, and remains present for life in both chronic carriers as well as those
who clear the infection. Its presence indicates exposure to HBV. It usually appears 3-6 months after
infection.

143

MERP Medical Virology

B. Serological events in chronic hepatitis:


Following acute infection, approximately 5-10% of infected individuals fail to eliminate the virus
completely and become persistently infected. Continual destruction of the liver nodules leads to
scarring, obstructed blood supply to the liver, cirrhosis, liver failure and is some cases cancer. Since
chronic Hepatitis B can also be asymptomatic, it can be discovered accidentally by observing
elevated liver enzymes in a routine blood test. Many chronically infected people are asymptomatic
and are a major source of infection. Those who are at particular risk for chronic hepatitis include:
Babies (95% chance of developing chronic hepatitis if infected)
Young children
Immunocompromised patients
males > females
Chronic infection may take one of two forms:
Chronic Persistent Hepatitis - the virus persists, but there is minimal liver damage
Chronic Active Hepatitis - There is aggressive destruction of liver tissue and rapid progression to
cirrhosis or liver failure.

Viral antigens in chronic hepatitis B:


1) HBsAg - When an individual is infected with Hepatitis B for longer than 6 months, he is considered
chronic, and unless the infection is resolved, will continue to synthesize HBsAg which can be
detected in the blood. This time frame is somewhat arbitrary but is important in classifying hepatitis as
acute or chronic. Presence of Hepatitis B surface antigen and simultaneous presence of anti-HBc
IgGs also signifies chronic hepatitis.

144

MERP Medical Virology

2) HBeAg in chronic infection,


levels of HBeAg may vary and
correlate to viral reproducibility.
3) HBcAg - Core antigen is not
detectable in the blood during either
acute or chronic infection.

Viral antibodies in chronic


hepatitis B:

1) HBsAb - Surface antibody would


not be found in chronic carriers

Development of the chronic hepatitis B virus carrier state. Routine serodiagnosis


is difficult during the hepatitis B surface antigen (HBsAg) window, when HBs and
anti-HBs are undetectable.

unless they have resolved the


infection.
2) HBeAb - e antibody (anti-HBe) in blood indicates low infectivity of the patient, however if HBsAg is
present, the patient is still considered infectious and is a carrier of the virus.
3) HBc IgM Core IgM is not present in chronic carriers. Due to isotype switching, 3 - 6 months post
initial infection, all anti-core antibody should be of IgG isotype.
4) HBc IgG Core IgG is evident in chronic carriers and persists for the rest of the individuals life.

Patients who become persistently infected with HBV are at risk of developing hepatocellular
carcinoma (HCC). Since the virus persists in the hepatocytes on-going liver damage occurs due to
the host immune response fighting against the infected liver cells. HBV is thought to be a factor in the
development of HCC because:

a) 80% of patients with HCC are carriers of hepatitis B,


b) Virus DNA can be identified in hepatocellular carcinoma cells,
c) Virus DNA can integrate into the host chromosome.

145

MERP Medical Virology

Liver cirrhosis and HCC can lead to portal hypertension: Vascular resistance and blood flow are
2 important factors in its development.

a.

c.

b.

Portal hypertension. Image a: The hepatic portal vein carries the blood from the GI tract and spleen to the
liver before this venous blood enters the inferior vena cava and the general circulation. Images b and c:
Portal hypertension is an increase in the pressure within the portal vein. Increased pressure in the portal vein
causes large veins (varices) to develop across the esophagus and stomach to bypass the blockage. The
varices become fragile and can bleed easily. In cirrhosis, the scar tissue blocks the flow of blood through the
liver and slows its processing functions.

Prevention/Treatment
1) Active Immunization with recombinant HBsAg is made by genetic engineering in yeast.
The administration of three doses induces protective levels of antibodies in 95% of vaccine recipients.
Universal immunization of infants was introduced in April 1995. Infants receive three doses before
they turn 1 year old.
2) Passive Antibody -Hepatitis B immune globulin should be administered to non-immune
individuals following single episode exposure to HBV-infected blood.

146

MERP Medical Virology

Interpretation of Hepatitis B Serologic Test Results

Another representation of acute and chronic Hepatitis B serology

147

MERP Medical Virology

LECTURE 15: PICORNAVIRIDAE and ORTHOMYXOVIRIDAE


Learning Objectives
Describe biochemical and structural features of Picornaviridae.
o
o
o
o
o
o
o

Relate the structure of these viruses to their stability in the environment


Describe the relationship between RNA genome and viral stability inside the host cell
Describe the immune responses important for clearing these viruses
List the mode of transmission of Picornaviridae
Describe the advantages and disadvantages of live and killed polio vaccines
Identify the diseases caused by this family based on description of symptoms
Differentiate between Hepatitis A and B

o
o
o
o

Describe biochemical and structural features of Orthomyxoviridae.


Describe the mechanism and consequences of genetic drift versus shift
Draw out the replication cycle of Orthomyxoviridae
List the symptoms and complications of flu.

For all positive strand RNA viruses the


naked genome, if artificially introduced
into a cell, is sufficient for infection.
All negative strand RNA viruses must
first transcribe their genome to positive
strand mRNAsw.

PICORNAVIRIDAE
Examples of viruses that belong to the Picornaviridae family:

Polio (3 serotypes)

Coxsackie A > 22 serotypes

Coxsackie B > 6 serotypes

Rhinovirus > 100 serotypes

Hepatitis A (one serotype)

148

MERP Medical Virology

Unique properties of Picornaviruse:

Picornaviruses bind to receptors that are members of immunoglobulin superfamily (such as ICAM1).

These viruses are small naked, icosahedral with a positive RNA genome.

Genome alone is sufficient for infection.

They replicate in the cytoplasm.

Most Picornaviruses are cytolytic (except Hepatitis A).

Viral, rather than immune pathology causes symptoms.

Antibody is important for control and prevention of future disease. Serum Ab important in
preventing/controling viremia.

All but Rhinoviruses are resistant to low pH, detergents, mild sewage treatment and heat.

Rhinoviruses grow best at 330Celcius and stay in respiratory tract.

Many Picorna viruses are spread by fecal-oral route via ingestion of contaminated food or contact
with infected hands or fomites.

Some Picorna viruses are transmitted through inhalation of infectious aerosol.

Infection is often asymptomatic or mild.

Picrona viruses that enter via GI tract are shed in feces for a long time.

Replication cycle (Figure right):

After entering the cytoplasm and


uncoating, the single RNA is

a.

b.

translated into a polyprotein (a)


c.

that is subsequently cleaved (b).

Vpg (virion protein genome) is a


d.

protein primer attached to the


e.

5'end of the RNA.

After being translated, the RdR


polymerase synthesizes a

f.

negative strand RNA template called replication intermediate (c), which is then copied to positive
RNA (d).

Virus assembles in cytoplasm (e), and leaves the cell through cell lysis (f).

149

MERP Medical Virology

Polio:

Like other enteroviruses, polio enters the body through the


gastrointestinal tract (Figure right). The virus replicates efficiently
in the intestinal tract and is typically shed in the stool for two to
four weeks, and sometimes for several weeks longer. Shedding
may be intermittent and is affected by the immune status and
immune competence of the individual. Poliovirus divides within
gastrointestinal cells for about a week After division the virus
spreads to the intestinal lymphoid tissue including the M cells of
Peyer's patches, and the deep cervical and mesenteric lymph
nodes, where it multiplies abundantly. The virus then enters the bloodstream.
Since virus also replicates in the upper respiratory tract, polioviruses can also be spread through
upper respiratory tract secretions. Virus can be recovered from throat swabs and washings during the
early acute phases of infection. Respiratory tract secretions are infectious and may provide a source
of virus for close contact spread through direct person-to-person contact, large-particle aerosols, or
fomites. From the pharynx the virus can spread to the tonsils (specifically the follicular dendritic cells
residing within the tonsilar germinal centers.
Poliovirus must cross blood brain barrier in order to cause paralytic disease. In the nervous tissue
it has a narrow tissue tropism recognizing receptors on the motor neurons in the anterior horn of the
spinal cord. The main receptor for polio is called CD155 and is believed to be present on most
human cells. Therefore receptor expression does not explain why poliovirus preferentially infects
certain tissues. This suggests that tissue tropism is determined after cellular infection. It is suggested
that Type I interferons (specifically that of interferon alpha and beta) is an important factor that
defines which types of cells support poliovirus replication. If a cell does not bind these interferons,
then it can replicated the virus very effectively.

150

MERP Medical Virology

Poliovirus infection can have 4 outcomes:

Polio outcomes

1. Asymptomatic illness - 90% of cases.

Minor
illness

Limited to gut and oropharynx.

2. Abortive polio (minor illness) - 5 % of polio cases

Nonspecific illness, headache, vomiting, fever,


malaise

Symptoms 3-4 days after exposure

Lasts a few days

3. Nonparalytic polio (aseptic meningitis) occurs in 1-2%


of people with polio.

Virus progresses from GI to blood and into


CNS (meninges) causing back pain, muscle spasms in addition to minor illness

4. Paralytic polio (major illness) - 0.1 - 2% of cases. 3 - 4 days after minor illness subsided, a
patient may experience more debilitating symptoms in
addition to meningitis sometime a patient progresses
directly to paralytic polio. In paralytic polio the virus travels

Polio outcomes
Major
illness

from blood to motor neurons in anterior horn cells of


spinal cord and to the primary motor cortex of the brain.
Outcomes of paralytic polio are:
a) Spinal polio
Most commonly there is damage to alpha motor neurons
so there is no stimulation of the skeletal muscle
controlling limb movement. This causes flaccid paralysis
in which limbs are weakened but the respiratory center is not affected. It may take years for
recovery.
Other symptoms include:

Severe muscle pain


Spasm of involved muscle
Hyperesthesia (increased sensitivity to stimuli)

b) Bulbar polio - polio destroys cranial nerves and cervical nerves:


Nerves affected in bulbar polio include glossopharyngeal nerve, accessory nerve,
trigeminal nerve, facial nerve and cervical nerves. This results in muscle atrophy of the
muscle controlling the eyes, facial muscles, pharynx, vocal cords, and diaphragm.
151

MERP Medical Virology

Results in death in 75 % of patients

c) Postpolio syndrome sequel of polio that may occur much later in life (30-40 yr. later).
People suffer deterioration of the originally affected muscles. Poliovirus at this point is not
present, but the syndromes is believed to result from the loss of originally effected neurons.
Figure: Mechanism of paralytic polio:
A . Three normal motor neurons and the muscle cells they supply;
B. Acute stage of polio. Some motor neuron are invaded by the
poliovirus and get damaged. The muscle cells (shaded) fed by the
middle neuron become stranded. In image C the orphaned
muscle cells are re-connected to the surviving motor neurons by
the growth of new terminal axon sprouts (TAS) creating giant
motor units. When these muscle cells become re-connected and
start working again, the individual regains lost strength. Note the
enlarged or hypertrophied muscle cells which develop in response
to exercise.
D. PPS. In the two remaining motor neurons, some terminal axon
sprouts are dying leading to new weakness.

Diagnosis:
Polio may be isolated from patients pharynx in
the first few days, or from feces for as long as 30 days. The virus is rarely found in CSF. Serology (IF,
neutralization assays, ELISA) are useful in diagnosing polio.
Prevention:
There are two commonly used Polio
vaccines: 1. IPV = Inactive Polio Vaccine (killed
vaccine), administered as an injection and 2. OPV
= Oral Polio Vaccine (live attenuated vaccine)
administered as drops by mouth. Both include the
three known serotypes of polio. OPV was
developed in 1958 by Dr. Albert Sabin. IPV, also
called the Salk vaccine, was developed by Dr.
Jonas Salk in 1952. The vaccine is a suspension
for subcutaneous injection. IPV contains strains of
the 3 types of polioviruses (Types 1, 2, and 3),
originally grown in monkey kidney cell culture and inactivated by exposure to formaldehyde.
152

MERP Medical Virology

Coxsackie viruses:

There are two groups of coxsackieviruses: A and B.

Infected individuals shed the virus in feces and respiratory secretions. These secretions can
contaminate surfaces and remain on them for a long time.

Most are systemic and depending on the serotype Coxsackie viruses may cause a febrile upper
respiratory tract infection with sore throat, a runny nose, gastrointestinal disease, rashes, and/or
organ infections.

The key to prevention of coxsackievirus infection is good hand washing and covering the mouth
when coughing or sneezing.

The virus is not destroyed by the acid in the stomach, and it can live on surfaces for several hours.

Once a person gets the virus, symptoms develop in a few days.

Coxsackie A often associates with systemic infection,


accompanied by rash with or without gastroenteritis.
Diseases
Vesicular lesions:

Herpangina a disease that presents as a fever,


sore throat, lesions in soft palate and uvula

Hand-foot mouth disease - symptoms include


fever, blister-like sores in the mouth (herpangina),
and a skin rash.

Non-specific rashes This virus often causes a


nonspecific generalized red rash or clusters of fine red spots. The rash appears towards
the end of the disease, and sometimes looks like a sunburn.

Conjunctivitis
People who have infected eyes (conjunctivitis) can spread the virus by touching their eyes and
touching other people or touching a surface. It often presents as red hemorrhages in the
whites of the eye. This infection may appear in both eyes.

Complications:
Meningitis- Patients complain of a headache and fever with mild neck stiffness.
153

MERP Medical Virology

Coxsackie B typically more serious than coxsackie A and infects deeper tissues.

Diseases:

Pleurodynia: also known as Bornholms disease or devils


grip, is characterized by thoracic and chest pain because
of the inflammation of the muscles in the chest.

Myocarditis: infects myocardium and causes inflammation


if the heart. Symptoms include: chest tightness, heart
palpitations, enlarged heart, arrhythmia, ECG ST_T
changes on the ectopic rhythm.

Pericarditis is an inflammation of the pericardium, the thin


sac (membrane) that surrounds the heart and the roots of
the great vessels. Chest pain, pleuritic pain that is worse
with inspiration, fever and chills are very common.
Histology of damaged heart

Structure of the heart

Complications:
Meningitis: Patients complain of a headache and fever with mild neck stiffness.

Hepatitis A: - "Infectious Hepatitis"


Incubation period is 3-5 weeks (the mean is 28 days). Hepatitis A is resistant to: acid pH 1,
ether, chloroform, salt water, drying, temperature (at 4oC for weeks, 56oC for 30 min) but is
inactivated by: chlorine, formalin, UV radiation and peracetic acid.
154

MERP Medical Virology

Clinical features:
Milder disease
than Hepatitis B

Hepatitis A spread: Virus enters via the gut; replicates


in the alimentary tract. There is a short viremia stage.
Virus spreads to the liver, where it multiplies in
hepatocytes. It is excreted through bile in stool.

Asymptomatic
infections are
very common,
especially in
children
Adults, especially
pregnant women, may develop more severe disease.
Although convalescence may be prolonged, there is no chronic form of the disease.
Fulminant hepatitis is rare: 0.1% of cases.
Complete recovery occurs in 99% of cases.
Virus is excreted in the stools for two weeks preceding the onset of symptoms.
The virus is transmitted predominantly by fecal-oral, enteric route.
Contamination of food or water with sewage is a common source of the virus. Shell fish from
sewage-polluted water can carry Hepatitis A.
Diagnosis is made based on the presence of HAV-specific IgM in the patient's blood. Prevention is
usually through active immunization with a killed vaccine, although a live, attenuated cell culturederived vaccine has recently become available but is not in general use.
Remember the 4 Nos of Hepatitis A
1. Not chronic
2. No cancer
3. Not fatal
4. Not lytic (unlike other Picornas)

155

MERP Medical Virology

Summary for pathogenesis of Picornaviridae enteroviruses

Rhinovirus:
The primary route of entry for human rhinoviruses is the upper respiratory tract (mouth and nose).
There are at least 100 serotypes of rhinoviruses but most share a common receptor ICAM-1 (InterCellular Adhesion Molecule 1) also known as CD54 repressed on respiratory epithelial cells. Infection
occurs rapidly, with the virus adhering to surface receptors within 15 minutes of entering the
respiratory tract. As the virus replicates and spreads, infected cells release
chemokines and cytokines which in turn activate inflammatory mediators. Cell lysis occurs at the
upper respiratory epithelium.
Rhinoviruses are spread by aerosol and on fomites and are the most common cause of the
common cold. Secretory IgA is transiently protective while interferon and other innate immune
components generated in response to the virus may limit the spread and contribute to symptoms,

156

MERP Medical Virology

ORTHOMYXOVIRIDAE
Unique properties of Orthomyxovirus
(Influenza A and B)

Enveloped virion with eight negative-sense RNA


nucleocapsid segments

The outer surface of the particle consists of a lipid


envelope and two prominent glycoprotein spikes:
haemagglutinin (H) and neuraminidase (N).

Hemagglutinin glycoprotein is the viral attachment protein and fusion protein. Note: this protein is
unlike fusion proteins that fuse at normal pH because it fuses with vesicular membrane at low pH.
Therefor fusion happens within the vesicle, rather than at plasma membrane. The H protein elicits
neutralizing, protective antibody response.

Entry and exit of influenza virus

Neuraminidase is a sialidase that cleaves the sialic acid


off the cell membrane during the budding stage of the
virus. This prevents viral particles from clumping with the
cell and attaching to each other.

95% of the surface spikes are hemagglutinin and


only 5% are neuraminidase

The inner side of the envelope is lined by the matrix


protein. The matrix protein is a structural protein that links
the viral envelope with the virus core. It plays a crucial
role in virus assembly, and interacts with the RNP
complex as well as with the viral membrane.

The genome segments are packaged into the core. The


RNP (RNA + nucleoprotein (N)) is in a helical form with the 3 polymerase polypeptides associated
with each segment.

Influenza A infects humans, mammals and birds (mutates frequently due to genetic shift and drift,
see below)

Influenza B only infects humans and only undergoes genetic drift (see below)

The particles are relatively labile (half-life of a few hours @ R.T.), not resistant to drying, etc.

Tamiflu (oseltamivir) is a current treatment against Influenza A and B. It targets the NA protein.

157

MERP Medical Virology

Replication cycle of Orthomyxovirus


Unique feature: RNA virus that transcribes and
replicates its genome in the target nucleus

Figure above:

(1) Virus binds sialic acid and enters via receptor mediated endoctysis
(2) Vesivular uncoating step utilizes a well-studies M2 protein (found in Influenza A only) and is
a target of drug Amantadine
All of the -RNA's have identical 5'ends containing 17-22 nucleotides. In addition, the 3'ends
have a high degree of conservation.
(3) In the nucleus, the RNA dependent RNA polymerase snatches hosts 5 CAP which serves
as a primer for RNA synthesis of 8 mRNA segments
(4) mRNA is transported to the cytoplasm and is translated
(5) RNA is replicated to progeny genomes via full length +RNA intermediate
Two classes of (+)sense RNA are made in infected cells:
a. Incomplete (meaning not full-length), 3' polyadenylated mRNA transcripts which are
exported to the cytoplasm and serve as mRNAs. The presence of a specific
polyadenylation sequence ~20nt from the 5' end of the (-)sense vRNA template strand
causes transcription termination.
b. cRNA = complete RNA, non-polyadenylated (+)sense copies of the (-)sense vRNA
(made by read-through of the polyadenylation signal). Full (+)RNA serve as template
for the synthesis of progeny (-) sense vRNAs (viral RNAs)
(6 and 7). Virus assembles in the cytoplasm and exits by budding.

158

MERP Medical Virology

Genetic shift and drift


The infidelity of RdRpol results in genetic drift (minor mutation). Additionally, the segmented
genome of Influenza A promotes genetic diversity through genetic shift (major reassortment).
Influenza viruses are changing by antigenic drift all the time, but antigenic shift happens only
occasionally.
Antigenic drift refers to small, gradual changes that
occur through point mutations in the two genes which contain
the genetic material to produce the main surface proteins:
hemagglutinin, and neuraminidase. These point mutations
result in minor changes to these surface proteins that often
leaves previous influenza antibodies partially protective.
Antigenic drift produces new virus strains that may not be recognized by antibodies to earlier
influenza strains. This process works as follows: a person infected with a particular influenza virus
strain develops antibody against that strain. As new virus strains appear, the antibodies against the
older strains might not recognize the "newer" virus, and infection with a new strain can occur.
Antigenic shift refers to an abrupt,
major change to produce a novel influenza A
virus subtype in humans that is not currently
circulating among people. Antigenic shift
usually occurs through mixing of human
influenza A and animal influenza A virus
genes to create a new human influenza A
subtype virus through a process called
genetic reassortment. Pigs and birds are
believed to be particularly important
reservoirs, generating pools of
genetically/antigenically diverse viruses
which get transferred back to the human population after reassortment. A reassortment of the highly
aggressive and deadly H5N1 avian virus with a human influenza virus is feared to cause a world
pandemic. However, this has not yet taken place.
A global influenza pandemic
(worldwide spread) may occur if three
conditions are met:
159

MERP Medical Virology

1. A new subtype of influenza A virus


is introduced into the human population.
2. The virus causes serious illness in
humans.
3. The virus can spread easily from
person to person in a sustained manner.
Conditions 1 and 2, but not 3 have been met.
In contrast to H5N1 bird flu virus, the 2009 pandemic with H1N1 swine flu was due to an
emergence of a previously existing swine flu virus in a human population. Also unlike H5N1, H1N1 is
easily spread, and does not cause as severe of an infection as H5N1. No direct recombination with
H5N1 was shown in swine H1N1, although swine H1N1 does contain some structural similarities to
the avian flu that infected humans in 1918 (Spanish flu).
Influenza A/H1N1

Transmission

Reassortment between human H3N2, North


American avian, and classical swine viruses
resulted in triple reassortant H3N2. H3N2 and
H1N2 swine viruses circulated in North American
pig populations. A triple reassortant swine virus
and H1N2 reassorted with a Eurasian avian-like
swine virus to produce swine H1N1.

Symptoms

Swine flu is an example of several genetic


shifts creating a new swine virus, although
this virus jumped to humans it did not result in
a very serious flu.

160

MERP Medical Virology

Disease mechanism:

Influenza usually causes a fever that lasts


3-8 days and unless complications occur,
recovery is complete within 10 days.

The symptoms of Influenza in adults


include malaise, sore throat, high fever,
muscle aches and fatigue. In children, the
symptoms may also include bronchiolitis,
croup, otitis media (ear infection) and
abdominal pain.

The virus is spread by small aerosol


droplets expelled during talking, breathing
and coughing.

It must first pass through inhibitory


mucoproteins, (secretions), however the

Pathogenesis of influenza A virus. The symptoms of influenza are


caused by viral pathologic and immunopathologic effects, but the
infection may promote secondary bacterial infection. CNS, Central
nervous system.

neuraminidase hydrolyses the mucoproteins, rendering them ineffective as inhibitors.

Virus enters the nasopharynx and spreads to susceptible cells whose membranes contain specific
receptors. It establishes infection in upper respiratory tract and targets mucus-secreting, ciliated
epithelial cells, causing a loss of primary defense system.

The symptoms of influenza result from infection and destruction of cells lining the upper
respiratory tract, trachea and bronchi. If the virus did not have neuraminidase, the secretory
mucoproteins would inhibit the virus.

When lower tract is involved, lower tract infection can cause severe shedding of bronchial and
alveolar epithelium. This can often result in pneumonia. Although pneumonia can have a primary
viral cause, it is usually due to a bacterial secondary infection. After flu bacteria can infect the
respiratory tract much easier since the virus has already destroyed the natural barriers of the
lungs.

Systemic symptoms are caused by interferon and lymphokine response to the virus.

Facial nerve inflammation is a rare complication of flu.

Influenza B viruses infect humans only and can cause disease, but generally not as severe as A
types. Unlike influenza A viruses, influenza B viruses do not have distinguishable serotypes.

161

MERP Medical Virology

Flu can be prevented by a killed (injection) and live (nasal mist) vaccines that contain predicted
yearly strains of influenza A and B. However these vaccines are only 50% effective.

Time course of influenza A virus infection. The classic


"flu syndrome" occurs early. Later, pneumonia may
result from bacterial pathogenesis, viral pathogenesis, or
immunopathogenesis.

162

MERP Medical Virology

LECTURE 16: PARAMYXOVIRIDAE, TOGAVIRIDAE AND FLAVIVIRIDAE


Learning Objectives
Describe biochemical and structural features of Paramyxoviridae.

List the differences between the systemic and local infections that belong to this family of
viruses
Describe the properties of the family including envelope structure, fusin protein, receptor
mediated fusion, syncytia formation, RNA replication
Describe disease mechanism for measles transmission, spread within the body, description
of the rash, complications, and prevention
Describe disease mechanism for mumps transmission, spread within the body, infection of
the organs, complications, prevention
Describe disease mechanism for RSV transmission, population at risk, complications
Describe disease mechanism for Parainfluenza virus transmission, population at risk,
complications

Describe biochemical and structural features of Togaviridae and Flaviviridae.

Describe vectortransmitted versus humantohuman transmitted infections


Describe clinical features of Dengue fever (explain why second time is worse), yellow fever,
WNE
Identify how rubella is different from other childhood rashes
List the differences between Hepatitis A, B and C
PARAMYXOVIRIDAE

Unique properties of Paramyxovirus

Enveloped, non-segmented, negative-sense


RNA virus

The virion RNA must be copied into the


complementary positive-sense mRNAs before
proteins can be made. Thus, besides needing to
code for an RNA-dependent RNA-polymerase,
these viruses also need to package this enzyme
into the virion so that they can make mRNAs
upon infecting the cell.

RNAd.RNA polymerase is capable of capping


and polyadenylation

The linear arrangement of genes (6 genes) are separated by a repeated sequence, a


polyadenylation signal at the end of the gene, and the intergenic non-translated sequence
followed by a transcription start signal at the beginning of the next gene

Replication occurs in the cytoplasm


163

MERP Medical Virology

All paramyxoviridae possess two distinct envelope proteins. One is involved in cell attachment and
the other mediates fusion with the

host cell membrane, in a pH-independent manner.

The attachment proteins of the paramyxoviridae bind to sialic-acid containing receptors on cells

Attachment induces cell fusion, causing multinucleated giant cells

All members of paramyxoviridae are transmitted by respiratory droplets and initiate infection in the
respiratory tract

Cell mediated immunity causes symptoms but is also essential for control of the infection

Replication cycle (Figure right)

a
b

The VAP protein recognizes receptors on


cell surface (a, b).

The F protein facilitates fusion between


membranes at physiological pH

Because the F protein functions at

d
f

physiological pH, this can result in syncytia

during paramyxovirus infections

The viral RNA polymerase uses the


negative sense RNA in the nucleocapsid

as a template for transcription (c).

Viral mRNAs are transcribed; these are

capped, methylated and polyadenylated


(d).

The viral mRNAs are translated to give


viral proteins (e).

There is no distinction between early and late functions in gene expression.

Viral RNA replication involves full length positive strand synthesis (f).

The positive RNA intermediate is used as a template for full length negative strand (g). New full
length minus strands may serve as templates for replication, or templates for transcription, or they
may be packaged into new virions.

Since this is a negative-strand RNA virus, RNA polymerase and RNA modification enzymes
are packaged in the virion (h).

The viral glycoproteins are translated as transmembrane proteins and transported to the cell
plasma membrane (i).

164

MERP Medical Virology

M (matrix) protein enables nucleocapsids to interact with the regions of the plasma membrane
which have the viral glycoproteins inserted.

The virus buds out through membrane (j).

Disease mechanism

The virus initially replicates in the epithelial tissues of the upper or the lower respiratory tract. In
the case of measles and mumps, this is followed by replication of the virus in the lymphoid tissues
leading to viremia and growth in a variety of other sites. Parainfluenza or RSV replicate
predominantly in the respiratory tract and are not systemic.

Infections with measles and mumps viruses are systemic. Mumps usually causes a benign
systemic febrile illness with swelling of salivary glands. Measles causes a childhood rash.

Measles and mumps exist as a single serotype. MMR (mumps, measles, rubella) vaccine contains
live, attenuated forms of all three of these viruses.

One important and unique feature of measles virus infection, in particular, is the virus ability to
persistently infect brain cells, which has been implicated in subacute sclerosing panencephalitis
(SSPE).

165

MERP Medical Virology

Specific human diseases:


Measles

Highly contagious disease spread by


respiratory droplets

Most serious childhood exanthem

Infects epithelial cells of the respiratory


tract. Can infect conjunctiva, respiratory
tract, urinary tract, small blood vessels,
and CNS

Prior to rash patients experience high fever, three Cs


(cough, coryza, conjunctivitis) and photophobia
First rash appears in the mouth, it is called Koplik
spots (small while lesions, surrounded by red halo)

A day later rash appears behind the ears, and spreads all
over the body, sometimes becoming confluent

Rash is caused by T-cell response to virus-infected epithelial


cells lining the capillaries

Cell-mediated response is necessary for resolution

Antibody is not sufficient because measles can spread from


cell to cell

Sequelae in central nervous system may result from


immonopathogenesis

Vaccine: Measles/Mumps/Rubella live vaccine

166

MERP Medical Virology

Mumps

Enters epithelial cells of respiratory tract

Spreads systemically by viremia (right figure)

Up to 1 in 5 people infected with the mumps


virus have no signs or symptoms

When signs and symptoms do develop, they


usually appear about two to three weeks after
exposure to the virus and may include:

Fever, fatigue, swelling of parotid gland (due


to the viral infection of parotid gland, right
bottom)

Rarer complications are: orchitis (infection of the testes) and


meningitis

Cell mediated immunity is essential for control of infection and is


responsible for causing a portion of the symptoms

Antibody is not sufficient because mumps can spread cell to cell

Vaccine: Measles/Mumps/Rubella live vaccine

167

MERP Medical Virology

168

MERP Medical Virology

Parainfluenza

There are 5 serotypes

Infection is limited to the respiratory tract, no


viremia.

The incubation period is 1-7 days.

The virus colonizes the nose and the


nasopharynx, it invades the epithelium and
results in cell damage, edema, and loss of
cilia.

A fibrinous exudate develops with downward


spread of cell damage and edema. The
resulting airway obstruction, inflammation of
the larynx and laryngeal muscle spasm
account for the typical symptoms of croup.

Typical symptoms include: cold-like symptoms, nonproductive to minimally productive barking


cough, coryza, inspiratory stridor, fever, nasal congestion, pharyngeal erythema and more rarely
bronchitis and wheezing.

Pathophysiology includes: airway inflammation, necrosis and sloughing of respiratory epithelium,


edema, excessive mucus production and interstitial infiltration of the lung.

Tachypnea (when lower airways become involved) is stimulated by excess CO2.

Protective immunity is of short duration.

Steeple sign in Parainfluenza croup. Steeple sign


on CXR results from subglottic narrowing of the
trachea (arrow) and is indicative either of
laryngotracheobronchitis (croup). Viral infection
causes croup which leads to the swelling of the
larynx, trachea, and large bronchi due to
infiltration of white blood cells. Swelling produces
airway obstruction which, when significant, leads
to dramatically increased work of breathing and
the characteristic turbulent, noisy airflow known as
stridor.

169

MERP Medical Virology

Respiratory syncytial virus


(RSV)

RSV is most serious in


infants from birth to 12
months of age and causes
90,000 hospitalizations and
4500 deaths each year in the
US.

Premature babies infected with RSV are at risk for respiratory distress.

RSV infection is limited to the respiratory tract

There is no viremia

In adults RSV typically causes a mild upper respiratory infection but in children lower respiratory
infections are more common.

Spread of the virus down the respiratory tract occurs through cell-to-cell transfer of the virus along
intracytoplasmic bridges (syncytia from the upper to the lower respiratory tract).

RSV is the most important viral cause of lower respiratory disease in infants and small children.

Narrow airways of young infants are readily obstructed by virus-induced and immune pathology.

Most common disease, bronchiolitis, is likely mediated by hosts immune response.

Maternal antibody declines rapidly in infants and does not offer long protection in the newborn.

Natural infection does not prevent re-infection.


In infants, most common symptoms include:

Fever (typically low-grade)

Cough

Tachypnea

Cyanosis

Retractions

Wheezing sound on expiration

Bronchiolitis

Pneumonia (results from cytopathological spread of virus)

170

MERP Medical Virology

Pathophysiology of RSV

Comparing and contrasting various diseases of the respiratory tract

171

MERP Medical Virology

TOGAVIRIDAE and FLAVIVIRIDAE

Toga and Flaviviridae are enveloped RNA


viruses consisting of more than 400
members, which are transmitted primarily (but
not exclusively) by arthropod vectors such as:
mosquitoes, sand-flies, fleas, ticks, lice, etc.
These viruses were previously grouped
together under the name 'Arboviruses'. They
are relatively fragile (e.g. not resistant to
desiccation, soap or detergents) viruses and
therefore many are reliant on vector for transmission. All are enveloped.

Togaviruses: Single-stranded, (+)sense, non-segmented RNA, resembles cellular mRNAs: i.e. 5'
cap, 3' poly-A.

Flaviviruses: Single-stranded, (+)sense non-segmented RNA, 5' cap but not polyadenylated.
Genetic organization differs from Toga's structural proteins at 5' end of genome, N.S. at
3' end.

Both replicate in the cytoplasm.

Toga - bud at the plasma membrane, Flavi


bud at the internal membrane.

These viruses can be asymptomatic, or cause


nonspecific symptoms (fever, chills), but could
also lead to encephalitis, arthritis and
hemorrhagic fever.

Rubella (Toga) and Hepatitis C (Flavi) have a


strictly human-to-human transmitted.

Can survive for long periods in hosts such as


ticks by replicating in this host (without damage
to the insect).

Although these are enveloped viruses and exit


by exocytosis, they usually cause lysis of the cell because they change the permeability, and
introduce several other insults to the cell that ultimately lead to cell death.

There are live attenuated vaccine for yellow fever and Japanese encephalitis (but not routinely
administered).
172

MERP Medical Virology

Replication cycle of Flaviviridae (note the difference between exocytosis, shown here, and
budding, shown earlier)
A. Virions attach to the
surface of a host
cell
B. Enter the cell by
receptor-mediated
endocytosis.
Endosomal vesicle
triggers
conformational
changes in the
virion, and results
in particle
disassembly.
C. In the cytoplasm,
the positive-sense
RNA is translated into a single polyprotein that is processed co- and post-translationally by
viral and host proteases.
D. Genome replication occurs on intracellular membranes.
E. Virus assembly occurs on the surface of the endoplasmic reticulum (ER) when the structural
proteins and newly synthesized RNA buds into the lumen of the ER.
F. The resultant non-infectious, immature viral and subviral particles are transported through the
trans-Golgi network (TGN).
G. Mature virions and subviral particles are subsequently released by exocytosis.

173

MERP Medical Virology

Most human-disease causing Togaviridae and Flaviviridae are transmitted by arthropods such
as mosquito

Mosquito life cycle


Mosquitoes acquire virus by blood meal from
a viremic host.
The virus then infects the epithelial cells of
the mosquitos gut, goes to circulation and
infects the salivary glands.
It then persists in the salivary glands which
release the virus in to the viral saliva.
Only those species that can get infection of
the salivary glands, will transmit the virus to
humans. This happens when the mosquito
bites its host and regurgitates viruscontaining saliva into the hosts bloodstream.

Below are examples of common arboviruses.

174

MERP Medical Virology

Disease mechanism
Togaviruses and Flaviviruses Mosquito transmitted

Togaviruses and Flaviviruses:

Typical transmission of arboviruses that cause meningitis/


encephalitis

Encephalitis

Virus is transmitted from the salivary


glands of the mosquito to the
bloodstream of the vertebrate host.

Virus travels to the skin and


reticuloendothelial system (spleen and
lymph nodes), where the primary
infection occurs. Replication in these
organs leads to a more pronounced viremia leading to systemic infection.

Acute onset of fever, headache, and vomiting progress to signs of meningeal involvement (stiff
neck and back)

More pronounced CNS involvement than meningitis alone

After acute symptoms, evidence of neuronal damage such as: drowsiness, coma, paralysis,
convulsions, ataxia and organic psychoses may occur.

CSF pleocytosis is usual, with up to 1000 leukocytes/mm3.

Mononuclear cells usually predominate, although early in fulminant encephalitis,


polymorphonuclear leukocytes predominate, glucose concentration in the CSF is normal, and the
protein is increased.

The peripheral blood shows a moderate polymorphonuclear leukocytosis.

With recovery from acute viral encephalitis, evidence of neuronal injury and death becomes
apparent as residual neurologic defects, impairment of intelligence, and psychiatric disturbances.

The severity of these sequelae varies according to the causative virus.

Encephalitis can be caused by Togaviridae (Venezuelan equine, Western equine and Eastern
equine), and by Flaviviridae (West Nile and Japanese)

Flavivirus infections:
Dengue fever

Endemic to North and Central Africa, S.E. Asia and some areas in Caribbean

There are 100 million Dengue cases per year. 300,000 cases per year are of a more serious
infection called Dengue hemorrhagic fever (DHF) or Dengue Shock Syndrome (DSS).
175

MERP Medical Virology

Dengue fever is also known as break-bone fever, the symptoms of dengue include:
o

fever, headache, rash, platelet loss, bleeding from mucous membranes, and serious
bone pain.

Severe disease is marked by two problems:

dysfunction of endothelium and disordered blood clotting.

If a person with Dengue fever is re-challenged with

another strain of Dengue fever, he will develop dengue


shock syndrome (DSS)/ DHF which results in severe
weakening and rupture of vasculature and internal
bleeding.

Because the principal targets of dengue are cells of

the monocyte/macrophage lineage, a newly acquired serotypes of Dengue virus can be


opsonized by antibodies that were produced against the previous dengue virus serotype
(which bind to the virion but do not neutralize its infectivity); hence, this virus is more avidly
taken up via Fc receptors into the very cell in which it replicates best (see below).

Above: Antibody dependent enhancement in Dengue fever. Opsonizing antibodies against Dengue
facilitate its entry into target macrophages.

176

MERP Medical Virology

177

MERP Medical Virology

Yellow fever:

Yellow fever (YF) was the first arthropod-borne viral


disease identified.

Transmission is human-mosquito-human. Thus, like


dengue fever and unlike most other arboviruses, YF
reaches high titers in human blood. The major organ
target is the liver, and massive necrosis of
hepatocytes leads to a decrease in the rate of
formation of prothrombin as well as to jaundice.

After a brief remission a minority of patients progress


to severe jaundice, massive gastrointestinal
hemorrhages, hypotension, dehydration, proteinuria, and signaling kidney failure.

Infection of the skin, bone marrow and blood vessels is noted.

Hemorrhagic degeneration of liver, kidney and heart are seen.

Mortality from this severe form of the disease is 20 - 50%.

Massive gastrointestinal hemorrhages is called

black vomit.
Figure left: Phase 1 (period of infection) - corresponds to
the host's initial viremia and typically presents with nausea
and vomiting, fever, headache, dizziness, and myalgia.
Leukopenia may be identifiable at the beginning of the acute
symptoms, while liver enzymes typically increase later. The
viral load will typically reach its maximum 2 to 3 days after the
initial infection. Importantly, those cases that end in fatality will
often have a higher and extended duration of viremia relative
to non-fatal cases.

Phase 2 is known as the remission, can be identified by a


break in the fever and relief from the clinical symptoms. The
remission can last up to 2 days, but is not a necessary part of
the natural history of disease. Some persons do
not experience any remission in their clinical course. On the
other hand, some infected persons will recover entirely during
this remission and are referred to as abortive infections.
These persons also do not demonstrate the jaundice
associated with the severe third phase, from which the

178

MERP Medical Virology


disease and the virus derive their names.

Phase 3 is referred to as the intoxication phase, and is the period of greatest clinical severity. Approximately 15% of
infected individuals enter the intoxication period, when more advanced liver dysfunction begins to manifest. Jaundice is
typically present, along with the return of nausea, vomiting and fever. Due to the associated coagulopathy, bleeding
diathesis is also common at this stage. Multiple organ failure is common, with kidney, heart and neurologic involvement,
depending on the extent of liver damage. As mentioned above, the case-fatality is 20% among endemic populations.

Diagnosis
For yellow and dengue fevers patient blood can be directly examined for viral genome by RT-PCR and
for viral antigens by monoclonal antibodies that are directed against the individual strains of viruses.
Serological methods including ELISA can be used to diagnose a recent infection, but serological crossreactivity among viruses limits the distinction of the actual viral species.
In the cases of encephalitis, the measurement of IgM in a patients CSF can be used for diagnosis.
Both Toga and Flavi viruses are difficult to grow and analyze in tissue culture.
Togavirus and Flavivirus disease exceptions:
Rubella

Rubella is a disease unique to other Togaviruses because it is not spread by arthropods. It has a
limited host range infecting mammalian
cells only. Rubella is transmitted by aerosols
and is highly contagious.

Pathogenesis: After early viremia, virus


multiplies in many organs, particularly lymph
nodes (lymphadenopathy), including the
placenta.

In children, it causes a mild febrile illness


characterized by maculopapular rash and
swollen glands (less severe than measles).
The rash is a continuous maculopapular and
pink. It is seen 14-25 days after infection in
95% of infected patients. Patients are
infectious for most of this time as well as several weeks after the onset of the rash.

In adults, in addition to the rash, rubella may cause more severe disease including bone pain and
arthralgia (joint pain). Rash is mediated by immune complexes rather than direct viral damage.
179

MERP Medical Virology

The most serious outcome of rubella is the fact that it


crosses the placenta and causes congenital defects
(most risk prior to week 20 post gestation) which include
cataracts, mental retardation, deafness, and heart
defects.

Studies have demonstrated that cells infected with


rubella in the early fetal period have reduced mitotic
activity.

Live attenuated MMR vaccine prevents Rubella.

For women infected during first trimester of pregnancy, therapeutic abortion may be
recommended.

Spread of rubella virus within the host (right). Rubella enters


and infects the nasopharynx and lung and then spreads to the
lymph nodes and monocyte-macrophage system. The resulting
viremia spreads the virus to other tissues and the skin.
Circulating antibody can block the transfer of virus at the
indicated points (X). In an immunologically deficient pregnant
woman, the virus can infect the placenta and spread to the fetus.

180

MERP Medical Virology

Hepatitis C - (formerly known as Non-A, Non-B hepatitis)

Hepatitis C is a disease unique to other Flaviviruses because it is not spread by arthropods.

Hepatitis C is a worldwide problem. The hepatitis C virus (HCV) is a major cause of both acute
and chronic hepatitis. WHO estimates that 170 million individuals worldwide are infected with
HCV.

Transfusion of blood contaminated with HCV was once an important source of transmission. Since
1990, all blood products are screened for Hepatitis C. Since the screening, needle-stick
transmission via non-sterile needles or needle stick injuries are the predominant ways to acquire
HCV. HCV may also be transmitted via tattooing, sharing razors, and acupuncture

Incubation period is 6-8


weeks

Infection with HCV is selflimited in only a small


minority of infected
persons.

In most infected people,


viremia persists and is
accompanied by variable
degrees of hepatic
inflammation and fibrosis.
Findings from studies
suggest at least 50% of
hepatocytes may be
infected in HCV patients with chronic hepatitis.

In 70% of affected people acute illness lasts 2-3 years, followed by a silent period of 10-15 years,
followed by chronic illness.

Development and persistence of strong virus-specific responses by cytotoxic T lymphocytes and


helper T cells is important for viral clearance.

Combination therapy with interferon-alfa (called PEG-IFN) and the nucleoside analogue ribavirin
is the current standard of care in patients infected with HCV. The addition of protease inhibitors to
the combination of PEG-IFN and ribavirin is becoming the new standard of care.

Newer drug called Harvoni has been approved in 2014. It is a combination of ledipasvir which is
an inhibitor of HCVs transciption activator NS5A and sofosbuvir which inhibits RNAdRNA pol.
181

MERP Medical Virology

LECTURE 17: RHABDOVIRIDAE


Learning objectives:
Describe biochemical and clinical features of Rhabdoviridae.

Describe the spread of rabies


Describe the importance of long incubation period
Characterize rabies infection
Describe the serological status of patient during all stages of rabies
Be able to identify advantages and disadvantages of passive and active immunization for
rabies

Unique properties of Rhabdoviridae

Bullet shaped, ss(-) RNA virus

Enveloped

Zoonotic (reservoir: wild animals); vector: wild animal


and unvaccinated dogs and cats

Viruses must bring their own RNA polymerase into the


cell in order to make short mRNAs

RNA dependent RNA polymerase makes single gene-unit length mRNAs (i.e., each mRNA
encodes only a single protein)

This is achieved by the use of transcriptional stop and start signals, which are located at the
boundaries of all of the viral genes.

mRNAs are capped at the 5' end and polyadenylated at the 3' end

Replication occurs in the cytoplasm

182

MERP Medical Virology

Mechanism of disease of rabies


Reservoirs: carnivorous animals including skunks,
raccoons, bats and dogs

The virus is transmitted by bites.

The virus multiplies in the salivary glands of


infected animals.

Virus can also be spread by aerosol (in bat


caves), and transplantation of infected tissue.

Rabies transmission within the body - see figure


on the right:

(1-2) The virus binds to the muscle cells at the site


of the inoculation via nicotinic acetylcholine
receptors. Here the virus replicates and can
remain for a prolonged period of time (up to
several months).

After weeks to months, the virus infects the


peripheral nerves (3-5) and travels up the CNS to
the brain (6).

Rabies transmission within the body

When the virus multiplies in the brain (7) it

produces Negri bodies (inclusion bodies) that are visible in stained brain sections using a light
microscope.

During the neurological phase, the virus spreads to the skin, eye, and salivary glands from where
it is transmitted (8).

183

MERP Medical Virology

Infection of the brain leads to encephalitis


and neural degeneration although elsewhere
the virus seems to cause little cytopathic
effect. Involvement of the brain leads to
coma and death. However little inflammation
in the brain is noted.

Symptoms before death include extreme


irritability, sensitivity to light and sound,
depression, fatigue, fever and eventually
paralysis. Hydrophobia sets off spasms that
are characteristic of the disease in humans
and animals in later stages.

There are various factors that determine the


timing of the onset of symptomatic rabies but
most important are the number of virus
Rabies disease progression

particles in the infection and how close the


bite is to the brain.

It should be noted that the immune response to naturally acquired virus is slow and a good
neutralizing response is not seen until the virus has reached the brain which is too late for
survival.

Cell-mediated immunity plays little role in a rabies infection. Rabies is almost always fatal and only
three survivors of symptomatic rabies have been documented.

Treatment:

Because the virus has a long incubation period, a good immune response that eliminates the
infection can be achieved using a killed vaccine even after infection.

Post-exposure rabies treatment usually involves passive immunization administered into the
wound followed by active immunization with a killed rabies vaccine administered at a different
site.

Since the pathophysiology of rabies virus infection appears to be primarily neuronal dysfunction
rather than inflammation and cell death, the clinical syndrome of rabies encephalitis is theoretically
reversible. A fundamental requirement for recovery would be viral clearance and development of a
protective immune response.

184

MERP Medical Virology

Diagnosis
Unfortunately, due to the lack of evidence for infection during asymptomatic phase, the lab
diagnosis usually occurs too late, and is mainly used to confirm the diagnosis postmortem. Viral Ag
can be detected in CNS, skin, saliva, and blood using direct IF or RT-PCR. Antibodies to rabies can
also be detected too late in the disease in CSF, and serum using ELISA. Brain biopsy can also be
assayed for Negri bodies (the only CPE that can be detected in infected cell).
Summary: The person exposed to rabies can remain asymptomatic for up to a year, but invariably
progresses to symptomatic rabies, which can take as little as 5 days to kill. Characteristics symptom
of rabies include: fever, headache, itching (2-10 days), neurological symptoms such as: hydrophobia,
seizures, inability to swallow, paralysis followed by coma and death.

185

MERP Medical Virology

LECTURE 18: RETROVIRIDAE


Learning Objectives
Describe in detail the replication cycle of HIV.

Detail the entry, reverse transcription, integration, mRNA synthesis, replication, protein
synthesis, exit, and the role of protease in HIV
Describe the disease mechanism and stages of HIV
Be able to read and Describe HIV serology as it relates to various stages of HIV
Describe briefly the stages of HIV replication targeted by drugs

Retroviruses
The three subfamilies of human retroviruses are the Oncovirinae (HTLV-1, HTLV-2, HTLV-5),
the Lentivirinae (HIV-1, HIV-2), and the Spumavirinae. The retrovirus genome is positive RNA (but
not infectious) and has a 5-cap and is polyadenylated at the 3-end. There are two general types of
retroviruses: simple ones and complex ones. Simple retroviruses contain a core genome, consisting
of three genes: Gag (encodes structural proteins), Pol (encodes enzymes), and Env (encodes
envelope proteins). Examples of simple retroviruses are murine leukemia virus (a virus which causes
cancer in mice) and human T-cell leukemia virus (the first human retrovirus discovered in 1982 and
can cause cancer in the bodys developing T cells). Complex retroviruses have additional accessory
genes that carry out diverse functions inside an infected cell. HIV is one example of a complex
retrovirus, consisting of the three essential genes (Gag, Pol, and Env) and six accessory genes: Vif,
Vpr, Vpu, Rev, Tat, Nef. In HIV, each gene can produce more than one final protein through the
mechanism of alternative RNA splicing, post translational modifications and frame shift mutations
during translation.

Simple retrovirus

Complex retrovirus (HIV)

186

MERP Medical Virology

Unique features of HIV

Enveloped with a capsid, contains 2 copies of ss RNA (positive-strand) genome

Reverse transcriptase, integrase, and protease enzymes are carried in the virion

Replication proceeds through a dsDNA intermediate, termed a provirus

Provirus integrates into the host chromosome and becomes a cellular gene

Transcription of the genome is regulated by host transcription factors interacting with promoter
and enhancer elements in the LTR portion of the genome

HIV can also spread from cell to cell through the production of multinucleated giant cells, or
syncytia

HIV is easily inactivated and must be transmitted in bodily fluids


HIV virion

Replication of HIV
Entry of HIV into cells

1.

2.
3.

187

MERP Medical Virology

The initial co-receptor used by the virus is CCR5, which is expressed on myeloid cells (macrophages,
DC and monocytes). The viruses attaching to CCR5 are called [M]-tropic. Later, during chronic
infection of a person, the envelope gene mutates so that the gp120 binds to a different chemokine
receptor called CXCR4, which is expressed primarily on T cells. Such viruses are considered T-tropic.
Reverse transcription/Integration

RdDpol activity of RTase makes DNA strand complementary to genomic RNA


RNAse H of RTase destroys RNA
DdDpolymerase activity of RTase allow formation of double-stranded viral cDNA
Viral cDNA is integrated into host chromosome

Transcription/Replication/Translation of HIV genome

Usually takes place in activated T cells


T cells are activated by HIV or by being primed for infection
Viral mRNAs and genomic RNA are made and are transported to the cytoplasm
Late proteins, Gag, Env and Pol are translated as polyproteins

Assembly and Budding

Env polyprotein (gp160) is cleaved into gp120 and gp41 in the host cell

During budding and release:

1.

Gag and Gag-Pol polyproteins are


translated as polypeptides
Many identical Gag polyproteins are
cleaved by the protease into individual
matrix and nucleocapsid proteins. Many
nucleocapsid proteins assemble together
to form a the nuncleocapsid and many
matrix proteins together form the matrix.
Pol polyprotein is cleaved into protease,
reverse transcriptase and integrase
Virus can cause syncytia formation with
subsequent lysis of these cells

4.
5.

3.
6.

6.

7.

Viral lifecycle:

7.

http://www.youtube.com/watch?v=RO8MP3wMvqg
Summary:
1-3. Attachment /fusion/uncoating
4. Viral RT converts ssRNA to dsDNA
5. Viral dsDNA integrates
6. Cellular transcription/genome replication
7. mRNA export (both short mRNA and viral
genome)
8. Cellular translation, protease cleavage
9. Viral assembly
10. Viral budding (After envelopment and release
from the cell, the viral protease cleaves the Gag
and Gag-Pol polyproteins to release the reverse
transcriptase and form the virion core, thus
ensuring the inclusion of these components into
the virion. The protease step is required for the
production of infectious virions)

9.

10
.

188

8.

2.

MERP Medical Virology

HIV infection
Sexually active people (homosexual and
heterosexual), intravenous drug abusers and their
sexual partners as well as the newborns of HIVpositive mothers are at highest risk for HIV infections.

1.
2.

3.

1) During sexual transmission, HIV infects a mucosal


4.

surface, enters and rapidly infects cells of the


mucosal-associated lymphoid tissue (MALT). HIV
enters the body mainly through the vagina,
penis, colon-rectum, or blood.
2) Mucosal APCs will pick up the virus and transfer it

5.

to the regional lymph nodes, where the virus will


infect many lymph node CD4+ T cells and replicate in
them.
3) High viremia results from the increased virus replication in the lymph nodes and infected CD4+ T
cells leaving the lymph nodes to enter the blood. At some point in infection, the virus shifts its tropism
from M-tropic (R5) to T-tropic (X4 virus). It now mainly infects CD4 T cells via CD4 and CXCR4
chemoreceptor.
4) CD4+ T cells are gradually destroyed by the pathological effects of the virus.
5) Loss of CD4+ T cells leads to immune system dysfunction and rise of opportunistic infections
Infection of cells of the brain including microglial cell and neurons eventually causes the release of
neurotoxic substances and promote inflammatory response in the brain.

CD8 T cells rather than antibodies are critical for controlling HIV disease progression. CD8 T
cells can kill infected cells by direct cytotoxic action and can produce suppressive factors that restrict
viral replication. However because CD8 T cells require CD4 T cells to help them divide, CD8 T-cell
numbers decrease with decreasing CD4 T-cell numbers, and their reduction correlates with and is an
indicator of disease progression to AIDS.

189

MERP Medical Virology

Disease Progression
HIV infection and disease progression
is divided into 3 stages based on virological,
immunological, and clinical characteristics:

Stage 1 = Acute phase (first 3 months)

Initial HIV symptoms occur 2-4 weeks


after infection, resembling influenza or
mono, and usually subside after several
weeks. 95% of people seroconvert by 4
weeks (i.e. develop anti-HIV antibodies).

During acute stage, HIV replication


explodes as the virus gains access to
many target cells in the lymph nodes.

This surge in viral replication is eventually


partially controlled by the adaptive
immune response (mainly CTLs and to some extent plasma cells making anti-viral Abs).

Stage 2 = Chronic phase (lasts years)


This stage is also termed asymptomatic or latency because many infected individuals dont
present with overt clinical symptoms. During this time, HIV replication is counterbalanced by
HIV destruction via the immune system. Nevertheless, CD4+ T cells continue to decline and
HIV continues to grow in lymph nodes. Antibody to HIV is readily detected at this stage. When
CD4 T cell count drops below
500 cells per microliter,
Opportunistic infections seen in HIV

opportunistic infection may


begin to appear, and
increased number of viral
RNA appears in the blood.
Stage 3 = AIDS
When CD4+ T cells drop
below 200 cells/l, life-threatening
opportunistic infections appear
190

MERP Medical Virology

(i.e., Kaposis sarcoma, CMV, toxoplasmosis, Pneumocystis pneumonia). Most of these infections
DO NOT cause disease in immunocompetent hosts, but can cause severe infections in people with
compromised immune systems, such as AIDS patients.

Why doesnt the immune system control HIV infection?


1) Subversion: HIV infects and destroys the very immune cells needed to help control the virus.
2) Mutations: HIV acquires numerous mutations
during reverse transcription that lead to antigenic
drift of viral proteins, thereby making it harder for
the immune system to recognize viral antigen.
Specifically the genes coding for the reverse
transcriptase and gp120 are heavily mutated.
Additionally, the gp120 of HIV is extensively
glycosylated which changes its antigenicity.

3) Latency: If HIV infects resting memory CD4+


T cells, the virus will remain latent or quiescent
with viral replication minimal in these cells. The
immune system will not be able to recognize
these infected cells. At some later point, these
resting memory cells can be awakened and
begin producing substantial amounts of virus.
4) Immune exhaustion: Persistent exposure of immune cells
to HIV antigen causes the immune system to prematurely
shut down due to upregulation of inhibitory receptors, inducing
cell anergy.

191

MERP Medical Virology

CD4 T cells have a


critical role in activating
and regulating cellmediated immune
responses, especially
toward intracellular
pathogens. Human
immunodeficiency
virus (HIV)-induced loss
of CD4 T cells results in
loss of the functions shown, especially the delayed-type hypersensitivity responses and the cytokine
control of immune responses.
The destruction of immune function as a result of HIV infection leads to increased
susceptibility to opportunistic infection and eventually to death

AIDS affects most systems within the body including CNS, lungs, GI and skin.

HIV transmission to the fetus

192

In the absence of treatment, the


risk of HIV transmission from
mother to child is 25-30%.
Antiretrovirals medication lower
mother-to-infant transmission to
less than 2%.
The vast majority of transmission
of HIV from mother to child
occurs at the time of birth when
the membrane is ruptured and
the blood systems can mix

MERP Medical Virology

A number of diagnostic tests can be used to determine whether a person is infected with HIV:

Blood tests are the most common way to diagnose HIV. The most sensitive of these tests is a
PCR-based method to detect HIV RNA (2 above) in the blood.

Depending on the estimated stage of disease, either an indirect ELISA to detect patients antiHIV Abs (1 above) or a sandwich ELISA to detect HIV Ag (4 above) can also be used. A
positive ELISA test is usually confirmed by Western Blot.

193

MERP Medical Virology

AIDS diagnosis occurs when certain criteria are met:


1. Evidence of virus infection & < 200 CD4+ T cells/l or
2. Evidence of virus infection & one AIDS-defining illness.
Treatment
For years, most AIDS experts have recommended starting antiretroviral drugs when the CD4
count fell below 350. One reason for waiting was that HIV-fighting drugs can cause unpleasant side
effects. The new guidelines say treatment should begin as soon as possible, regardless of the CD4
count. Drug treatment cant totally get rid of the virus. But it can keep the amount of virus in the body
low enough to slow down and often prevent destruction of the immune system. It will also limit the
spread of HIV from person to person. Some exciting new data indicate that the use of one or more of
these drugs can prevent infection in individuals who do not have HIV but who are high risk for it. In
fact, the FDA recently approved the use of Truvada (a combination of two antiretroviral drugs) to
reduce the risk of becoming infected with HIV by sexual transmission of the virus.
There are 25 anti-HIV drugs clinically available today to treat HIV infection, and it is highly
recommended that a combination of drugs (encompassing different classes) be used to minimize the
emergence of drug-resistant virus.
Classes of anti-HIV drugs:
1) nucleoside RTase inhibitors (NRTI) = inhibit HIV RT and the process of reverse transcription
2) non-nucleoside RTase inhibitors (NNRTI) = inhibit HIV RT and the process of reverse
transcription
3) integrase inhibitors (II) = inhibit HIV integrase and the process of proviral integration into host
DNA
4) fusion/binding inhibitors = inhibit HIV binding to CD4 or gp41-mediated fusion
5) protease inhibitors (PI) = inhibit HIV protease and the process of HIV protein maturation
6) CCR5 antagonists = prevent interaction of gp120 with CCR5R
HAART = 2 NRTIs + a PI or NNRTI or II

194

MERP Medical Virology

Virology practice questions:


1. A patient presents to your office with chest pain and fever. You suspect a viral cause for his
condition. Which of the following would differentiate the suspected virus from Rhinovirus?
a.
b.
c.
d.
e.

A gene for RNA dependent RNA polymerase


Presence of envelope
Ability to replicate in the nucleus
Resistance to acidic environment
Stability inside the cell

2. Which of the following is true regarding viral pathogenesis?


a. Viruses that are acquired through respiratory do not cause viremia
b. Humoral immunity but not cell mediated immunity (CMI) acts
against viruses
c. Viruses that are acquired via skin contact can cause viremia
d. RNA viral mutations can only occur in the cytoplasm
e. RNA viruses are more likely than DNA viruses to cause persistent infections
2. Which of the following is a major difference between Hepatitis B and Hepatitis C?
a.
b.
c.
d.
e.

Tissue tropism
Polymerase
Symptoms
Immune response
Transmission

4. A virus was isolated from the stool of a patient with flaccid paralysis. What else is a likely
characteristic of this virus?
a.
b.
c.
d.
e.

Two versions of oral vaccines are available for public use


Viral infection of the motor neurons can lead to latency
Immunity can be transferred by serum from recovered person
Sub-clinical infection is uncommon
Relatives of the patient are likely to develop paralysis

5. Regarding viral hepatitis which is the wrong answer?


a.
b.
c.
d.
e.

HBV-Abs can be diagnosed in the lab by ELISA


Hepatitis B vaccine will prevent HepB-induced hepatocellular carcinoma
Hepatitis B can cause liver cancer and integrate into host genome
Blocking RNAseH activity would prevent the spread of Hepatitis B virus
In chronic Hepatitis B one finds HBsAg and pentameric anti-HBc antibodies

6. Which of the following is correct regarding varicella and zoster diseases?


a. They are two diseases caused by one virus
b. They have the same clinical picture
c. Zoster is a disease of children, whereas varicella is a disease of
195

MERP Medical Virology

elderly and immunosuppressed patients


d. Varicella and zoster can be prevented by killed vaccinate
e. Drugs that function as DNA polymerase inhibitors are toxic to host cells
7. A 31-year-old man presents to your free clinic complaining that his urine is dark and his stools are
pale. On physical examination, you note icteric sclera and conclude that he likely has hepatitis. A
hepatitis serology panel is performed:
Lab Test
HBsAg
anti-HBcIgM
HBV DNA

Result
positive
positive
1010 copies/mL

You see the patient three months later, at which time a second hepatitis serology panel is performed:
Lab Test
Result
HBsAg
negative
anti-HBs IgG positive
anti-HBcIgG positive
HBV DNA
not detectable
What is the most likely interpretation?
a. The man was vaccinated for HBV
b. The man has chronic HBV
c. The man has acute HBV and will become chronically infected
d. The man had acute HBV but cleared the infection
e. These results are inconclusive
f. The man had chronic HBV but cleared the infection
8. Refer to the serological data below, obtained from a patient recovering from Hepatitis B. The x-axis
indicates the number of months before and after acute symptoms (acute symptoms are marked by
month 0). The y-axis indicates Hepatitis B specific antigens and antibodies detected in the patient.
Which of the following letters point
to the curve(s) that represent(s) the
antibody(s) one use(s) as a marker
of Hepatitis B vaccination?
a. A
b. B
c. C and F
d. C and D
e. E
f. F
9. The replication cycle of which of
the following virus is illustrated
below?
196

MERP Medical Virology

a.
b.
c.
d.
e.

Hepatitis A
HIV
Hepatitis B
Rubella
Herpes simplex 1

10. Pick the correct match for the following rashes and their description:
Measles
Chicken pox
Erythema infectiosum (B19)
Roseola

A. Starts with Kopliks spots in the mouth


B. Infects erythroid precursors
C. Uses reverse transcriptase
D. Rash follows fever, spares the face
E. can be reactivated from neurons as shingles
F. caused by a naked RNA virus

a. MeaslesA; chicken poxF; B19C; roseolaB


b. MeaslesC; chicken poxB; B19A; roseoa E
c. MeaslesE; chicken poxF; B19B; roseolaA
d. Measles F; chicken poxB; B19E; roseolaD
e. MeaslesA; chicken poxE; B19B; roseolaD
11. Each of the following clinical symptoms is associated with infection by picornaviruses except :
a.
b.
c.
d.
e.

Myocarditis
Hepatitis
Paralysis
Childhood rash
Common cold

12. What is the first virus-specific synthetic event that occurs in cells infected with the HIV virus?
a.
b.
c.
d.
e.

DNA syntheses
Synthesis of RNA strand
Synthesis of + RNA strand
Translation to generate viral proteins
Synthesis of mRNA

197

MERP Medical Virology

13. A virus isolated from blood is RNAse and detergent sensitive. Which of the following is a virus that
was isolated?
a. Hepatitis A
b. Hepatitis B
c. Coxsackie A
d. Coxsackie B
e. Polio
f. Small pox
g. Measles
14. Which of these viruses is not transmitted via a blood transfusion?
a. HIV
b. Hepatitis B
c. CMV
d. Hepatitis C
e. HPV16
15. You identify a new dsDNA, enveloped virus that is persistent and is found in all organs of the
body, including the basolateral lining of the GI tract. Which of the following means of transmission
was most probably not utilized by this virus?
a. Sexual contact
b. Fecal/oral
c. Contact with spit
d. Respiratory secretions
e. Blood

198

MERP Medical Bacteriology

199

MERP Medical Bacteriology

BACTERIOLOGY MODULE
LECTURE 19: INTRODUCTION TO BACTERIOLOGY: STRUCTURE AND METABOLISM
Learning Objectives:
Describe the differences between the gram-positive and the gram-negative bacteria.
Describe various bacterial structures and know detail function these structures serve
Describe the differences in transcription mechanism of eukaryotes vs. prokaryotes (Describe
lac operon)
Describe the differences between exotoxins and endotoxins
Describe the difference between aerobic and anaerobic bacteria in terms of the enzymes they encode
and growth conditions they prefer.
Detail the role of oxygen in the process of glycolysis, anaerobic fermentation, aerobic
fermentation
Describe the mechanisms by which bacteria are able to disarm oxygen radicals
Identify organisms as strict anaerobes, strict aerobes, facultative anaerobes or air tolerant
Describe the various stages of the growth curve of bacteria in liquid media.
Calculate the number of bacteria, generation times etc. based on information given
Bacterial shapes
Bacteria are the smallest living things and lack mitochondria, nucleus, Golgi, ER and lysosomes. This
structure is distinguished from eukaryotic cells and is called prokaryotic.
1. There are several common shapes of bacteria: coccus (spherical), coccibacillus, vibrio
(comma shaped), bacillus (rods), spirillum, and spirochete.

2. In addition to shape, bacteria can arrange in aggregates (due to their stickiness). For cocci,
these include diplococci (pairs) streptococci (chains), and staphylococci (irregular or grapelike).
The basic structure of bacteria can be divided into two main parts: the envelope which containing
many complex and unique molecules and the interior which nuclear body and cytosol.

200

MERP Medical Virology

Envelope: cell membrane, cell wall, capsule, flagella, pili

Protects bacteria against chemical and biological threats


Carries out many bacterial processes
Contains appendages that make it possible for bacteria to colonize surfaces
All bacteria have a cell membrane and cell wall, the other components are optional

CELL MEMBRANE:
An electron transport system of the cell that is vital for cell growth.

Cell membrane is similar to eukaryotic cells except that it has many more proteins and no
sterols (except Mycoplasma which has sterols).

The bacterial chromosome is attached to cell membrane.

The membrane has proteins that control the entrance and exit of solutes and proteins.

CELL WALL:
Structure essential for bacterial life

Almost all bacteria have a rigid peptidoglycan


(murein) layers that surround the cytoplasmic
membrane. This structure is totally unique to
prokaryotes.

Cell wall consists of alternating sugars Nacetylglucosamine (NAG) and N-acetylmuramic


acid (NAM) connected via -(1,4)-glycosidic
bond. The NAM, NAG layers may be cross-linked
into sheets by amino acids.

Gram-positive bacteria have a thick multilayered


cell wall (figure right) with embedded teichoic acid.
The teichoic acids consist of polymers of glycol
phosphate or ribotol phosphate with extra moieties
such as amino acids. A type of teichoic acid called
lipoteichioc acid also has a fatty acid that anchors
it into the cytosolic membrane. Both of these
elements promote bacterial adhesion to the host.

201

MERP Medical Bacteriology

Gram-negative bacteria have a


more complex outer structure.
o They have a thin peptidoglycan
layer, no teichoic acid, BUT they
have an additional outer
membrane (OM) external to the
peptidoglycan.
o The space between cytoplasmic
(or inner) membrane (IM) and OM
is called the periplasmic space
and contains peptidoglycan wall
(not extensively cross-linked), as
well as many metabolic enzymes and virulence factors.
o Outer leaflet of OM contains lipopolysaccharide (LPS), also called endotoxin. LPS consists
of toxic Lipid A, core polysaccharide, and O antigen. Lipid A is made of hydrophobic
fatty acid chains that anchor the LPS into the bacterial membrane, the core domain
contains an oligosaccharide component that attaches directly to lipid A and commonly
contains sugar heptose, the O antigen is a repetitive glycan polymer. The lipid A domain is
responsible for much of the toxicity of Gram-negative bacteria.
o OM serves as a barrier to organic molecules and hydrophobic molecules. Active transport
and diffusion of nutrients is facilitated by proteins called porins that traverse the entire OM.

The cell wall is the main feature that distinguishes Gram positive (G+) and gram negative (G-)
bacteria.

G(+) or G(-) are distinguished based on a gram stain.

The gram stain is called a differential stain since it differentiates between gram-positive and
gram-negative bacteria. Bacteria which stain purple with the gram staining procedure are termed
gram-positive; those which stain pink are said to be gram-negative. The terms positive and
negative have nothing to do with electrical charge, but simply designate two distinct morphological
groups of bacteria.

202

MERP Medical Bacteriology

The gram staining procedure involves four basic steps (figure bottom):
1. The bacteria are first stained with the basic dye crystal violet. Both gram-positive and gramnegative bacteria become directly stained and appear purple after this step.
2. The bacteria are then treated with gram's iodine solution.
This allows the stain to be retained better by forming an
insoluble crystal violet-iodine complex. Both gram-positive and
gram-negative bacteria remain purple after this step.
3. Gram's decolorizer, a mixture of ethyl alcohol and acetone,
is then added. This is the differential step. Gram-positive
bacteria retain the crystal violet-iodine complex while gramnegative are decolorized.
4. Finally, the counterstain safranin (also a basic dye) is applied.
Since the gram-positive bacteria are already stained purple,
they are not affected by the counterstain. Gram-negative
bacteria, which are now colorless, become directly stained by
the safranin. Thus, gram-positive appear purple, and gram-negative appear pink.
With the current theory behind gram staining, it is thought that in gram-positive bacteria the crystal
violet and iodine combine to form a larger molecule that precipitates out within the cell. The
alcohol/acetone mixture then causes dehydration of the multilayered peptidoglycan, thus decreasing
the space between the molecules and causing the cell wall to trap the crystal violet-iodine complex
within the cell. In the case of gram-negative bacteria, the alcohol/acetone mixture, being a lipid
solvent, dissolves the outer membrane and the cell wall (and may also damage the cytoplasmic
membrane to which the peptidoglycan is attached). The single thin layer of peptidoglycan is unable to
retain the crystal violet-iodine complex and the cell is decolorized.

203

MERP Medical Bacteriology

CAPSULE:
Capsule is a polysaccharide

Generalized structure of bacteria (is this a gram-positive or a gram-negative?)

slime layer which serves as a


major virulence factor, but is
NOT essential for survival.

Capsules are found on both


G(+) and G(-) bacteria.

Capsules protect against


phagocytosis and resist
complement binding.

Several bacteria including


Streptococcus pneumoniae,
Haemophilus influenzae and
Klebsiella pneumoniae, produce
a glycocalyx capsule that inhibits phagocytosis.

In addition to, or sometimes instead of the capsules, other bacteria use aletrnative structures to
protect themselves against phagocytosis. Mycobacterium tuberculosis accomplishes this by the
insertion of lipids into its cell wall and Streptococcus pyogenes has the M protein in its cell wall to
decrease phagocytosis. Staphylococcus and Streptococcus use leukocidins to destroy leukocytes
and macrophages and hemolysins to disrupt erythrocytes.
Flagella:
These structures are responsible for locomotion: bacteria may have none, one, or many flagella.
These structures are found on certain G(+) and G(-) bacteria and are made of helically coiled
proteins that are anchored into bacterial inner membrane.

Proteins called flagellins differ between strains and therefore can be used for differentiation of
bacteria.

Pili
Pili are made of proteins called pilin that surround a hollow core.

They are much smaller than flagella in diameter, and are all over the surface of the bacteria. Their
main function is to promote attachment to host cells often in a specific manner. Pili that serve for
attachment are called fimbriae.

Pili can also function in genetic exchange of DNA between bacteria; these are called F pili or sex
pili. Sex pili are usually found one per cell.
204

MERP Medical Bacteriology

Pili can be found on certain G(+) and G(-) bacteria.

Endospore/Spore

Some gram-positive bacteria may form


an endospore (right diagram) within the
cytoplasm. This is a structure that allows
the organism to resist adverse conditions
and is the dormant state of bacteria.

Spores are made of a material called


calcium dipicolinic acid several layers of
peptidoglcycan and keratin.

Spores are dehydrated structures. They


resists chemicals, boiling and radiation,
conditions that would normally kill the
vegetative cells from which they formed.

Nearly all household cleaning products, alcohols, and detergents have little effect.

10% bleach is effective against endospores when they are left in the bleach for at least 10
minutes. Endospores can also be destroyed by autoclaving. Some endospores are able to survive
boiling at 100C for hours, although the longer the number of hours the fewer that will survive.

The presence of water, glucose and alanine often cause the germination for the spore.
Interior core; cytosol, nucleoid, plasmids

Cytosol:
The cytosol contains many ribosomes which are made of 5 S rRNA, 16 S rRNA, 23 S rRNA, as
well as a single 50 S and a single 30 S protein subunit. There is little structural but considerable
functional homology between prokaryotic and eukaryotic ribosomes. Most ribosomes at any given
time are involved in the translation of mRNA.

Nucleoid:
Bacterial cells lack a membrane defined nucleus. However a discrete region in the bacterial
cytoplasm seems to contain the genetic material and this nucleiod region can often be distinguished
on EMs of cells. Most cells have only one main chromosome which consists of a single, circle of
deoxyribonucleic acid.
205

MERP Medical Bacteriology

Plasmids:
Bacteria may have pieces of genetic material other than chromosome. These smaller pieces of
DNA are known as plasmids and are defined as extrachromosomal pieces of DNA which are capable
of autonomous (or regulated) replication.
Examples of roles of plasmids:

Antibiotic resistance - Some plasmids code for proteins that degrade antibiotics-a big
advantage for pathogens.

Some encode for proteins which confer virulence factors on the host. For example- E. coli
plasmid Ent P307 codes for an enterotoxin which makes E. coli pathogenic.

Conjugative plasmids - These allow exchange of DNA between bacterial cells

DNA replication, transcription and translation


Similar to eukaryotic cells, bacteria need to carry out the essential processes of 1) DNA replication,
2) transcription and 3) translation.
1) Replication of DNA:
Replication starts at replication forks and proceeds bi-directionally similar to eukaryotic DNA.
Bacterial replication involved an RNA primer and Okazaki fragments, helicase, and primase. Since
bacterial chromosome is circular, replication introduces a lot of torsion in the strands, which is
relieved by the enzyme called gyrase.
2) Transcription: Bacteria use DNA dependent RNA polymerase for transcription. This enzyme is
composed of several subunits:
Sigma factor a protein within DdRpol that
recognizes a particular nucleotide sequence in

Structure of bacterial
DNA dependent RNA polymerase

the promoter of a gene and then serves to initiate


transcription. Different bacteria will have a
different number of sigma factors. E. coli, for
example, has seven sigma factors.
Alpha and beta subunits are enzymes that
carry out DNA polymerization.
Differences between eukaryotic and prokaryotic transcription:
1. In bacteria a single RNA polymerase makes all RNA. In humans there different RNA
polymerases for mRNA, tRNA, rRNA.
206

MERP Medical Bacteriology

2. Bacterial mRNA has a much shorter half-life


Bacterial transcription

than eukaryotic mRNA.


3. Organization of genes is different in
prokaryotes and eukaryotes. In bacteria
many genes are organized into multicistronic
operons (Figure right).

An Operon is a unit of transcription containing


several genes (cistrons) that are transcribed as a
single mRNA. The operon usually consists of a
promoter, operator and terminator.

The operator can specifically regulate


transcription by binding a repressor which
inhibits transcription or binding an activator to initiate transcription.

The repressors and activators are themselves products of other genes that are outside the
operon and may not be part of multicistronic sequences.

Operons can be either 1) inducible: which means they are normally in the OFF state because
they are repressed by a repressor, unless something in bacteria turns them ON or
2) repressible: which means that the operon is usually ON and is transcribed, unless
something shuts it OFF. An example of an inducible operon is the LAC operon and an example
of a repressible operon is the tryptophan operon.

LAC OPERON:
Bacteria prefer to use glucose rather than other less well-metabolized sugars as carbon
source.
When growing in an environment containing both glucose and lactose, bacteria metabolize
glucose and at the same time prevent the expression of the lac operon, the products of which
transport and metabolize lactose.
Regulation of the LAC operon:

Negative regulator: The operon is usually in a repressed state because it is normally subject to
negative regulation by the lac operon repressor protein called LacI.

LacI is encoded by the lacI gene, which is located immediately upstream of the lactose
operon and transcribed by a separate promoter.
207

MERP Medical Bacteriology

In the absence of lactose, LacI binds specifically to the operator region of the lac
promoter and blocks transcription.

Presence of lactose releases LacI from the operator, thereby alleviating the
repression. Therefor the lac operon is switched on only if lactose is available as a
carbon source for cell growth, but remains unexpressed if glucose, the cell's preferred
carbon source, is also present.

Positive regulator: The transcription factor cAMP-dependent catabolite activator protein


(CAP) is a positive regulator of the LAC operon. It is only active when cAMP is bound to it.

Because glucose inhibits adenylate cyclase, when bacteria grow in glucose the
cytoplasmic levels of cAMP are low and so CAP is not activated.

When the glucose is depleted, the cAMP concentration rises, resulting in the formation
of activated cAMP-CAP complexes, which then binds the DNA sequence within the
promoter, recruiting RNA polymerase binding and initiating transcription. CAP is an
example of a global regulatory protein that controls the expression of multiple genes; it
controls the expression of over 100 genes in E. coli.

208

MERP Medical Bacteriology

Virulence factors:
Virulence factors are genetic traits which enhance the ability of bacteria to cause disease.
Virulence factors include: adhesion molecules, structural molecules and secreted factors. Many
bacteria use virulence factors to cause disease by directly destroying tissue; some others release
toxins, which are then disseminated by the blood to cause system-wide pathogenesis.
Certain surface structures of bacteria are powerful stimulators of host responses such as
cytokines which can be protective but are often the significant causes of the disease symptoms (e.g.,
sepsis). For example, on infection with gram-positive bacteria, peptidoglycan and its breakdown
products, as well as teichoic and lipoteichoic acids are
released and can stimulate endotoxin-like pyrogenic
acute-phase response. Acute-phase proteins are a
class of proteins whose plasma concentrations increase
(positive acute-phase proteins) or decrease (negative
acute-phase proteins) in response to inflammation.
Cytokines such as IL1, IL6 and IL8, and TNF act on the
liver to secrete these proteins. Acute phase proteins

Differences between exotoxins and endotoxins

Exotoxin Endotoxin
Produced by
both Grampositive and
Gramnegative
bacteria

Produced
only by
Gramnegative
bacteria

Released
from cell

Integral part
of cell wall

include pro-coagulants and bacteria binding proteins that


ihibit bacterial growth but also add to inflammation.

Protein

Lipid A of
lipopolysaccharide

Production of disease results from the combination of


damage caused by the bacteria and the consequences of
the innate and immune responses to the infection.
Bacterial toxins are generally divided into two

Many types
of exotoxin
based
on structure
and function

Only one
type of
endotoxin

Heat labile

Heat stable

Specific
receptors on
host target
cells

Diverse range
of host cells
and systems
affected

Specific
effects in
host

Diverse range
of effects in
host

Toxoids can
be made
by treating
with formalin

Toxoids
cannot be
made

categories: exotoxins which are secreted proteins


produced by some gram-positive and some gramnegative bacteria and endotoxins (ie LPS), produced
only by only by gram-negative bacteria.

Exotoxins include degradative enzymes that cause lysis


of cells or specific receptor-binding proteins that initiate
toxic reactions in a specific target tissue. Below are some
features of exotoxons:

In many cases, the toxin is completely responsible for


causing the characteristic symptoms of the disease.
209

MERP Medical Bacteriology

Because a toxin can be spread systemically through the bloodstream, symptoms may arise at a
site distant from the site of infection.

A specific type of secreted toxins called cytolytic toxins include membrane-disrupting enzymes
which break down sphingomyelin and other membrane phospholipids. Hemolysins insert into and
disrupt erythrocyte and other cell membranes.

Pore forming toxins can promote leakage of ions and water from the cell and disrupt cellular
functions or cell lysis.

Many exotoxins are dimeric and complesed of A and B subunits. The B portion of the A-B toxins
binds to a specific cell surface receptor, and then the A subunit is transferred into the interior of
the cell, where cell injury is induced.

Endotoxin: The lipopolysaccharide (LPS) produced by gram-negative bacteria is a powerful


activator of acute-phase and inflammatory reactions and is termed endotoxin.
Gram-negative bacteria release endotoxin during infection.
Endotoxin binds to specific receptors (CD14 and
Effects of

TLR4) on macrophages, B cells, and other cells


and stimulates the production and release of
acute-phase cytokines such as IL-1, TNF-, IL-6,
and prostaglandins.
At low concentrations, endotoxin stimulates the
mounting of protective responses such as: fever,
vasodilation, and the activation of immune and
inflammatory responses.
High concentrations of endotoxin can activate the
alternative pathway of complement and
production of anaphylotoxins (C3a, C5a),
contributing to massive vasodilation and capillary
leakage.
In combination with TNF and IL-1, this can lead to
hypotension and shock.

Disseminated intravascular coagulation (DIC) can also result from the activation and
dysregulation of blood coagulation pathways.

210

MERP Medical Bacteriology

Therefore, high levels of gram-negative bacteria are very dangerous and can lead to gramnegative bacterial sepsis.

Energy production:
Fueling reactions provide bacteria with energy and precursor metabolites used to make amino
acids, nucleotides, sugars, fatty acids and other building blocks. Essential minimal nutrients of
bacteria include: source of carbon, nitrogen, water and ions.

Sources of energy:
All bacteria (aerobic and anaerobic) use Embden-Meyerhof-Parnas (EMP) glycolytic pathway
to make pyruvate from glucose. Then pyruvate can be metabolized by fermentation (1), aerobic
respiration (2), or anaerobic respiration (3).
1. Fermentation: large amount of organic acids and alcohols are produced from pyruvate which
receives electrons from NADH. This pathway has low ATP-generating efficiency, because ATP
only comes from
EMP pathway.

Energy production by bacteria

2. Aerobic
Respiration: In the
presence of oxygen,
pyruvate is
completely oxidized
to water and CO2 by
the Tricarboxylic
Acid Cycle (TCA).
Electrons from
NADH and FADH
are transferred
through a chain of
carriers to an
ultimate acceptor O2. Very efficient at generating ATP because ATP is made not only during
EMP pathways but also during oxidative phosphorylation. Bacteria that grow aerobically and
thus utilize respiration must have protective enzymes to protect themselves from being
destroyed by toxic radicals.
211

MERP Medical Bacteriology

3. Anaerobic respiration is a membrane-bound biological process coupling the oxidation of


electron donating to the reduction of suitable external electron acceptors (other than molecular
oxygen). In contrast to anaerobic respiration, in fermentation, the oxidation of molecules is
coupled to the reduction of an internally-generated electron acceptor, usually pyruvate.

Depending on bacterial oxygen requirement bacteria can be classified as:


1. Obligate anaerobes
2. Obligate aerobes
3. Facultative anaerobes
4. Oxygen tolerant (Airtolerant)

Obligate anaerobes:
Bacteria that are not able to utilize molecular oxygen and are harmed by it are called obligate
anaerobe. These organisms cannot tolerate atmospheric oxygen pressure. In order to make ATP,
they usually ferment or undergo anaerobic respiration.

Aerobes:
An aerobe is a microorganism that can survive on oxygen and may utilize molecular oxygen as its
final electron acceptor, i.e., as in cellular (aerobic) respiration.

1. Obligate aerobes:
An obligate aerobe is a microorganism that cannot live in the absence of molecular oxygen. This
basically means that they cannot obtain energy via fermentative processes. More precisely, obligate
aerobes are organisms that have an electron transport system are able to grow in the presence of
atmospheric oxygen concentrations. These organisms use O2 as a final electron acceptor and cannot
ferment.
2. Facultative anaerobe:
A facultative anaerobe is an aerobic microorganism that can utilize fermentation or anaerobic
respiration when molecular oxygen is absent, and utilizes aerobic cellular respiration when O2 is
present.

212

MERP Medical Bacteriology

3. Airtolerant:
Aerotolerant

Flow chart for figuring out bacterial oxygen requirements

bacteria have
an exclusively
anaerobic
(fermentative)
type of
metabolism but
they are
insensitive to
the presence of
O2. They live
by fermentation
alone whether
or not O2 is
present in their
environment.
The response of an organism to O2 in its environment depends on how well the cell is able to
deal with the various oxygen radicals generated by bacteria. All cells contain enzymes capable of
reacting with O2. For example, oxidations of flavoproteins result in the formation of H2O2 (peroxide) as
one major product and small quantities of an even more toxic free radical, superoxide or O 2.-. In
aerobes the accumulation of superoxide is prevented by the enzyme superoxide dismutase. All
organisms which can live in the presence of O2 (whether or not they utilize it in their metabolism)
contain superoxide dismutase and catalase or another less efficient enzyme called peroxidase.
Obligate anaerobes lack these enzymes and undergo lethal oxidations by oxygen radicals.
Enzymes necessary to disarm oxygen radicals

213

MERP Medical Bacteriology

The presence of catalase can be quickly measured on either directly


isolated specimen or grown culture. In this test, H2O2 is added to the
bacteria, if catalase is present, H2O2 is broken down and the
generation of oxygen bubbles (O2 gas) will be evident.
Identifying bacteria in the laboratory

Catalase test

There are often consecutive steps in bacterial diagnostics. These


steps include: 1. Specimen staining, 2. Culture of specimen, 3. Performing specific biochemical and
antigen specific test.
1. Collected specimen can be evaluated directly by various methods (prior to obtaining results
from the culture):
a. Light microscopy (but too difficult to distinguish between bacteria)
b. Gram stain followed by microscopy
c. Acid-fast stain
d. Fluorescent microscopy

2. Growing bacteria from the collected specimen prior to its identification is a very sensitive
and commonly used technique for diagnosis.
Agar- acidic polysaccharide extracted from algae that dissolves at 100 oC and solidifies into a gellike substance at 45oC. Used in most common culture preparations.
o Bacterial colonies grown on agar plates may have different morphologies which is
useful for identification of bacteria.

There are various types of media that can be used to grow bacteria. These media can be classified
as:
Nutrient media contains all the ingredients necessary to support the growth of most bacteria
(usually made of digests of animal or plant products such as milk, beans), it may also contain
vitamins and salts depending on the bacteria isolated.
Selective media-

1. Nutrient agar, 2. Selective agar, 3. Differential agar

inhibits growth of
unwanted

1.

2.

organisms from
the normal flora.
Usually contains
chemical
214

3.

MERP Medical Bacteriology

additives such as anti-microbial agents that inhibit contaminating flora but not the pathogen of
interest.
Differential/ Indicator media- Demonstrates biochemical features of the organism: Most often a
carbohydrate and a pH indicator are added to see whether the organism can ferment that
carbohydrate to produce acid which then changes the color of the pH indicator and therefore the
color of the colony.
3. Biochemical test: After growing bacteria, the colonies are often subjected to various biochemical
tests, which allow the differentiation of bacteria.

Bacterial growth in liquid medium are characterized by standard bacterial growth curve:
Bacteria added to fresh media typically go through four distinct phases of growth
a. Lag phase
b. Log (logarithmic or exponential) phase
c. Stationary phase
d. Decline (death) phase
Lag phase
Transfer of bacteria from one medium to another, where there is a chemical difference
between the two media will cause a lag in cell division. This lag in division is associated with a
physiological adaptation to the new environment, by the cells, prior to their resumption of division
That is, cells may increase in size during this time, but
simply do not undergo binary fission.
Log phase (logarithmic phase, exponential phase)
Lag phase is followed by log phase during
which binary fission occurs. This phase of growth is
called logarithmic or exponential because the rate of
increase in cell number is a multiplicative function of
cell number. This can be seen in a graph of cell
number versus time where cell numbers increase at
ever increasing rates with time or generation; that is,
the rate of increase is a function of absolute cell
number such that the more cells present, the faster the population of cells increases in size. When
considering the log phase, a formula can be used to calculate the total number of bacteria generated

215

MERP Medical Bacteriology

after a certain amount of time. This formula is: A(f)= A(i)x2n, where A(f) is the final number of bacteria,
A(i) is the initial number of bacteria, and n is the number of generations in a given time.
Stationary phase
Stationary phase is a steady-state equilibrium where the rate of cell division is exactly balanced by
the rate of cell death. Cell death (or, at least, lack of cell growth) occurs because of a loss of limiting
nutrients (due to their incorporation into cells during log-phase growth) or a build-up of toxic products
such as fermentative products released during log-phase growth.
Decline phase (death phase)
Stationary phase, in a standard bacterial growth curve, is followed by a die-off of cells. Cell death in
bacteria cultures basically means that the cells are unable to resume division. This death occurs
because vegetative cells can survive exposure to harsh conditions (few nutrients or too-many toxins)
for only so long.
Practice questions:
1. Characteristics of a bacterial capsule include:
a. Only gram positive bacteria have one
b. It is composed of poly-D glutamic acid
c. It protects bacteria against ingestion by PMNs
d. It is what causes the gram stain reaction
2. How would gram-negative bacteria appear under the microscope if you accidentally forgot to add
crystal violet during the staining procedure, while doing everything else correctly?
a.
b.
c.
d.
e.
f.

Clear
Red
Yellow
Purple
Blue
White

3. Which of the following is true regarding endotoxins?


a.
b.
c.
d.
e.

They are primarily made of proteins


Most endotoxins have very specific, rather than generalized effects
Endotoxin-mediated disease may be prevented by toxoid vaccination
Endotoxins are made equally well by both gram (+) and gram (-) bacteria
Endotoxin expressing bacteria are unlikely to ever form spores

4. How would you describe an organism that can respire in the absence of oxygen but not in the
presence of oxygen?
a. Obligate fermenter
b. Facultative anaerobe
c. Strict aerobe
216

MERP Medical Bacteriology

d. Strict anaerobe
e. Facultative respirator

217

MERP Medical Bacteriology

LECTURES 20 AND 21: STAPHYLOCOCCAL AND STREPTOCOCCAL INFECTIONS


Learning Objectives
Describe the structures, virulence factors, and clinical presentation of Streptococci and Staphylococci.
Define the functions of Protein A, alpha toxin, exfoliative toxin, enterotoxin, and
superantigen
Be able to identify pictorially or based on a description common skin infections (impetigo,
boils, SSSS), GI poisoning and systemic infections that Staphylococci cause
Differentiate which Staphylococcal infections are mediated by bacterial replication vs.
secreted exotoxin
Describe the function of superantigen
Describe the media that selects for Staphylococcus aureus
Describe blood agar and the differences between alpha, beta and gamma hemolysis
Differentiate between the three Streptococcal species (S. pyogenis, S.
agalactaiae, and S. pneumoniae) based on biochemical properties and clinically
List the virulence factors for GAS (capsule, M protein, sAg and pyogenic exotoxin)
Describe Streptococcal infections including strep throat, scarlet fever, rheumatic fever, skin
infections, and systemic invasive infections
Describe the basis of GAS rapid antigen test
Describe the disease mechanism of GBS and Describe the CAMP test
Identify diseases caused by and the virulence factors for S. pneumonaeie
Describe the vaccines for S. pneumonaeie
LECTURE 20: STAPHYLOCOCCAL INFECTIONS
Staphylococcus aureus
Background:
Carried in the normal flora (human skin and
mucosal surfaces)
Breaking the natural barriers (like skin) can lead to
disease.
The anterior nares are the main site of colonization
Other sites of colonization include:
axilla, rectum, and perineum
The vaginal carriage rate is approximately
1-10% in premenopausal women
The rate is higher during menses
Morphology and Physiology

Staphylococci are gram-positive cocci, whose bacterial


cells arrange in clusters

Staphylococci are facultative anaerobes that grow by


aerobic respiration or by fermentation that yields mainly
lactic acid.
218

Morphology (top) and membrane


structure (bottom) of S. aureus

MERP Medical Bacteriology

The bacteria are catalase-positive and oxidase-negative.

Nearly all strains of S. aureus produce the enzyme coagulase

They are non-motile and do not form spores

When grown on Mannitol-Salt agar (MSA), S. aureus ferment mannitol,

MSA

and grow on high (7.5 %) salt concentration (right figure)

Resistance to drying

On nutrient again Staph aureus appear as golden yellow colonies


(below).

S. aureus is beta hemolytic on blood agar (see below)


Hemolysis of sheep red blood cells is an in vitro rupturing of RBCs in culture which may be used
as a tool to determine the species and strains of various bacteria.

Examples of hemolysis
by various bacteria (left)

219

MERP Medical Bacteriology

Virulence Factors
S. aureus causes a wide range of
suppurate skin infections, food
poisoning and toxic shock syndrome.
S. aureus expresses many potential
virulence factors:
Structural components:
(1) Surface proteins that promote
colonization of host tissues
(2) Surface antigens that inhibit
phagocytic engulfment (capsule,
Protein A)
Secreted enzymes:
(3) Invasins that promote bacterial spread in tissues
(4) Enzymes that enhance the survival in phagocytes (carotenoids, catalase production);
Secreted toxins:
(5) Membrane-damaging toxins that lyse eukaryotic cell membranes (cytotoxins such as hemolysins,
leukotoxin, leukocidin);
(7) Exotoxins that damage host tissues or otherwise provoke symptoms of disease (exfoliative toxin,
TSST, Staph enterotoxin A- SEA)
Specific examples of Virulence Factors
Adherence to Host Cell Proteins

S aureus express on their surface proteins that promote attachment to host proteins such as
laminin and fibronectin that form the extracellular matrix of epithelial and endothelial surfaces.

Certain strains express a fibrin/fibrinogen binding protein (clumping factor) which promotes
attachment to blood clots and invasion of traumatized tissue.

220

MERP Medical Bacteriology

Interaction with collagen may also be important in promoting bacterial attachment to damaged
tissue where the underlying layers have been exposed.

Membrane-damaging toxins and other exotoxins

Alpha-toxin is the best characterized cytotoxin and most potent membrane-damaging toxin of S
aureus. Subunits oligomerize to form hexameric rings with a central pore through which cellular
contents leak. Susceptible cells have a specific receptor for alpha-toxin which allows the toxin to
bind causing small pores through which monovalent cations can pass. In humans, platelets and
monocytes are particularly sensitive to alpha-toxin. After binding the toxin, a complex series of
secondary reactions ensues, causing release of cytokines that trigger production of inflammatory
mediators. These events cause the symptoms of septic shock that occur during severe infections
caused by S aureus.

Exfoliative toxin is a serine protease that that split the intracellular bridges in the stratum
epidermis. This toxin is the cause of Staphylococcal Scalded Skin Syndrome (see below for toxinmediated skin infection).

Enterotoxin (Staph enterotoxin A - SEA) causes direct effect in the gut-intestinal fluid loss with
diarrhea and vomiting.

Superantigen (TSST-1) toxin is an exotoxin that stimulates T cells, causing fluid loss, nausea and
vomiting. Up to one in five T cells may be activated, whereas only 1 in 10,000 is stimulated during
a usual antigen presentation. Cytokines are released in large amounts, causing the symptoms of
TSS. Superantigens bind directly to class II major histocompatibility complexes of antigenpresenting cells outside the conventional antigen-binding grove.

Avoidance of host defense

Coagulase is an extracellular protein which binds to prothrombin in the host to form a complex
called staphylothrombin. The protease activity characteristic of thrombin is activated in the
complex, resulting in the conversion of fibrinogen to fibrin. Coagulase is a traditional marker for
identifying S aureus in the clinical microbiology laboratory. Bacteria is protected from phagocytic
and immune defenses by causing localized clotting.

221

MERP Medical Bacteriology

Protein A is a surface protein of S


The effect of protein A

aureus which binds IgG molecules


by their Fc region. In serum,
bacteria will bind IgG molecules in
the wrong orientation on their
surface which disrupts
opsonization and phagocytosis.

Leukocidin is a toxin that


specifically damages
polymorphonuclear leukocytes.
Phagocytosis is an important
defense against staphylococcal infection and leukocidin impedes that process.

Pathogenesis

222

MERP Medical Bacteriology

Non toxin-mediated skin infections:

Staphylococcus aureus causes many


superficial and deep skin lesions. The
severity of infection depends on the
depth of the tissue infected.
o Epidermis: Impetigo (superficial
infection that causes the production
of pus-filled vesicle)
o Superficial dermis/Deep Dermis:
Furuncles=Boils=Folliculitis are
painful raised nodules that have
underlying collection of necrotic
tissue/pus. These may extend deeper than hair follicle.
Carbuncles are coalescence of furuncles with invasion of

Folliculitis

deeper subcutaneous tissue; patients have chills and fever


which suggests a systemic infection.
Erysipelas are lesions with sharply demarcated raised edge.
They are red, swollen, warm, hardened and painful.
Erysipelas does not affect subcutaneous tissue. It does not
release pus, only serous fluid. In erysipelas, the infection rapidly invades and spreads
through the lymphatic vessels. This can produce overlying skin "streaking" and regional
lymph node swelling and tenderness.
o Subcutaneous layer:

Cellulitis is inflammation of the connective tissue below dermis as well as other


subcutaneous tissue. Cellulitis results in fever and chills in addition to skin infection.

Fasciitis is an infection of the fascia surrounding the muscle.

Impetigo

Carbuncles

223

Cellulitis
Erysipelas

MERP Medical Bacteriology

Toxin-mediated skin infections:


Staph Scalded skin syndrome (SSSS, Ritters disease) is usually in babies
and results from production of exfoliatin in Staph lesions. Skin gets blistered,
these lesions contain very little bacteria, but are caused by toxin. Also few
leukocytes are present in the lesions. A slight pressure on the skin displaces
the skin. Although the disease looks awful, it usually does not leave any
scarring on the skin. The lack of SSSS in adults is explained by the presence
of antibodies specific for exotoxins and improved renal clearance of toxins.
Non-toxin mediated, non-cutaneous infections:
o Bacteremia and Endocarditis are often hospital acquired, and is
associated with bacteria from the skin getting into the blood stream (as a result of surgical
procedure or contaminated intravascular catheter). Endocarditis (inflammation of the inner
layer of the heart, damage to endothelial lining of the heart) needs to be treated promptly,
otherwise the patient has very poor prognosis.
o Osteomyelitis is an infection of the bone with Staph which can result from trauma that spreads
the bacterial infection to the bone esp. in children where growing bones are highly
vascularized. The disease is characterized by localized pain, high fever and purulent discharge
from the sinus tract overlying the infected bone.
o Pneumonia is consolidation and abscess formation in the lungs seen in very young, elderly
and in patients with underlying pulmonary disease.
Toxin mediated, non-cutaneous infections:
o Staph food poisoning (right) is the most common
form of food poisoning in US. It is caused by
ingestion of the poison, which is not readily
destroyed by the gut. The foods that facilitate
the growth of this organism and the release of
the toxin include ham, pork, potato salad, ice
cream. The food is contaminated by someone
who has a Staph skin infection or carried the
organism in the nasopharyngeal fluid. If the food
remains at room temperature, bacteria can
grow, which then, can be killed by reheating, but
the toxin is heat resistant and is therefore
not destroyed by reheating.
o Toxic shock syndrome was first noticed in children
receiving vaccine contaminated with Staph (in
Australia), then, in 1980s there was a dramatic
increase of disease in menstruating women.
Menstruation appeared to facilitate TSST-1 toxin
production. This was associated with the use of
hyperadsorbent tampons that were made from
different material than previous tampons. These
tampons helped Introduce more oxygen(toxin
production requires an aerobic atmosphere and
neutral pH) into vaginal tract and blossomed into cup224

MERP Medical Bacteriology

like shape which helped filter toxins and encouraged growth of S. aureus. These tampons
were later recalled, decreasing the incidence of disease.
Less than 1% of women carry Staph
aureus strain containing TSST in vaginal
flora. When secreted the toxin stimulates
the release of large amounts of
interleukin-1, interleukin-2 and tumour
necrosis factor. It does so by stimulating
human T cells that express Vbeta 2, which
may represent 5-30% of all host T cells.
The toxin binds primarily to the alphachain of class II MHC and the beta chain
on a TCR stimulating T cells to become
activated and subsequently activate the
antigen presenting cell also bound to the
toxin.
Disease is characterized by
the release of toxin in to the
blood which causes abrupt
onset of fever, hypotension,
and a diffuse, macular
erythematous rash. Multiple
organ systems (e.g., central
nervous, gastrointestinal,
hematologic, hepatic,
musculature, renal) are also
involved, and the entire skin,
including the palms and
soles, desquamates or peels
off. The rash is similar to a
sunburn. In menstruating
TSS cases, blood is usually
negative for bacteria. With
better understanding of the disease, the death rate dropped to 5 %.
Diagnosis
1. Gram staining of the lesion illustrates G(+) cocci in
clusters
2. Lesions usually contain many bacteria that can be easily
cultured overnight in aerobic environment in media
containing blood agar. Staph. produces hemolysin which
causes beta (complete) hemolysis of RBC.
Lesions also often contain numerous neutrophils.

225

MERP Medical Bacteriology

3. To select the growth of Staph aureus, the agar should be supplemented with 7.5 % NaCl (inhibits
growth of most organisms), and mannitol (fermented mainly by Staph, not other organisms).
4. Colonies can be tested for coagulase and catalase
When lesions cannot be accessed directly (ex. deep tissue infections) blood cultures can be done.
For food poisoning, diagnosis is made based on clinical history.
Treatment
Infections acquired outside hospitals can usually be treated with penicillinase-resistant -lactams.
Hospital acquired infection is often caused by antibiotic resistant strains and can only be treated with
vancomycin.

226

MERP Medical Bacteriology

LECTURE 21: STREPTOCOCCAL INFECTIONS


Streptococcus
General features:
G (+), cocci in chains or
pairs
Catalase negative
Air-tolerant
Three species will be
discussed (right)

Streptococcus pyogenes (Group A Strep)


Virulence Factors

Adherence (colonization) surface macromolecules

Lipoteichoic acid
M protein
227

MERP Medical Bacteriology

Enhancement of spread in tissues

Streptokinase lyses fibrin


Hyaluronidase hydrolyses hyaluronic acid, part of the ground substance in host tissues

Evasion of phagocytosis

C5a peptidase: C5a enhances chemotaxis of phagocytes.


Capsule: hyaluronic acid is produced.
M protein is a fibrillar surface protein. Its distal end bares a negative charge that interferes with
phagocytosis. It also blocks complement deposition on the cell surface. Mutations during the
course of infection alter the structure of M proteins, rendering some antibodies ineffective.
Strains that persist in carriers frequently exhibit altered M proteins.
Leukocidins, including streptolysin S and streptolysin O, are proteins secreted by the
streptococci to kill phagocytes (probably to release nutrients for bacterial growth).

Defense against host immune responses

Antigenic variation. Antibody against M protein (antigen) is the only effective protective
antibody, but there are more than 50 different M types, and subsequent infections may occur
with a different M serotype.
Antigenic disguise and tolerance provided by hyaluronic acid capsule

Production of toxins and other systemic effects


Streptococcal pyrogenic exotoxin (SPE) is an exotoxin that acts as a superantigen. Without
being processed it binds directly to MHC II and links the MHC expressing APC to the T cell receptor
of many (up to 20% of) T cells. Exaggerated production of cytokines causes the signs of shock: fever,
rash, low blood pressure. This is similar to TSST-1 toxin effects.
Pathogenesis:
Suppurative Infections (active infections associated with pus) occur in the throat, skin, and
systemically.
Throat infections:
Pharyngitis is the most common bacterial cause of Strep throat (esp. in kids 5-15 yrs of age). The
pathogen is spread by large respiratory droplet
and the symptoms are hard to distinguish from
viral pharyngitis unless bacteriological and
serological tests are done. The lymph nodes in
the neck become swollen, tonsils, uvula and soft
palate become red, swollen and covered with
yellow-white exudate. Two complications are
noted: glomerulonephritis and rheumatic
fever.
Scarlet fever - 1-3% of people with pharyngitis
will develop scarlet fever.
228

MERP Medical Bacteriology

Some strains of Strep have been lysogenized by bacteriophage that stimulates the production of
erythrogenic, pyrogenic exotoxin which causes a rash on the chest, and face but not around the
mouth. A bright red tongue is also characteristic of scarlet fever. Later in the infection, skin peeling
may be seen.
Skin infections: (see Staph section for pictures)
Impetigo: Impetigo involves the infection of epidermal layers of skin. It begins with localization of skin
by S. pyogenes which can then be introduced into subcutaneous tissue through cuts in the skin. Pus
filled vesicles develop which can rupture and crust. Pre-pubertal children are the most susceptible.
Systemic infections are unlikely.
Erysipelas- Acute infection of the dermal layer of the skin. Usually on face and legs, and may be
preceded by infection of the respiratory tract by Group A strep. Is not associated with systemic
disease but about 5% of patients may develop disseminated disease.
Cellulitis- Cellulitis occurs when the infection spreads subcutaneous tissues.
Necrotizing fasciitis (systemic) is an
infection that spreads along fascial planes
and is characterized by severe destruction of
muscle and fat. This is often referred to by the
media as flesh-eating bacteria; the organism
is introduced into tissue though cuts, burns or
surgery causing systemic symptoms (fever,
shock, and multi-organ failure). Unlike
erysipelas which can be treated with
antibiotics, this disease must be treated with
aggressively with surgical removal of infected
tissue. The strain usually carries additional
toxins. This disease is more common with
Strep than with Staph.
Toxic shock syndrome (systemic): certain very invasive
strains of Gr. A Strep can cause systemic disease
characterized by inflammation at the initial site of infection,
then fever, chills, vomiting, diarrhea and multi-organ failure.
Whereas Staph-induced (TSS) is mainly due to the toxin
entering the blood stream, Strep-induced TSS is associated
with bacterial presence in the blood and is often linked to
necrotizing fasciitis.

229

MERP Medical Bacteriology

Non-suppurative sequelae:
Some of the antibodies produced during the above infections cross-react with certain host tissues.
These can indirectly damage host tissues, even after the organisms have been cleared, and cause
non-suppurative complications.
Rheumatic fever: The most feared
complication of post -Streptococcal
pharyngitis (Strep throat) disease which
is characterized by changes in the heart and
joints leading to myocarditis and aortic valve
stenosis. The mechanism for this is molecular
mimicry, where heart muscle tissue cross
react with antibodies formed to
Streptocococcus. Major symptoms of aortic
stenosis are chest pain (angina), fainting
(syncope), and shortness of breath. In about
one third of all patients diagnosed with aortic
valve stenosis, chest pain is usually the first
sign.
Because the aortic valve is narrowed, this increases pressure in the heart. It also increases the
oxygen demand for the blood flowing into the heart because there is less blood being pumped by the
heart due to the narrowing of the aortic valve. This can also attribute to shortness of breath.

230

MERP Medical Bacteriology

Glomerulonephritis:
Post-infectious acute inflammation of the renal
glomeruli can occur. This is accompanied by
edema, hypertension and hemato and
proteinuria. The mechanism of damage is
similar to either Type II or Type III
hypersensitivities. In Type III reaction, immune
complexes with Strep antigen and anti-Strep
antibodies are deposited in the kidney
basement membrane (see Figure on the
right). In Type II reaction, antibody against
Strep cross-reacts with host cells in the
basement membrane of the kidney and
deposit there. In both cases this results in
inflammation of the kidney with dysregulation
of kidney function. Only a few M-types are
nephritogenic.
Diagnosis:
Stain: Gram staining of samples from affected tissue can provide INITIAL diagnosis. Strep. is often
part of the normal flora of the oropharynx and sometimes skin.
Culture: Specimen obtained from the oropharynx, skin or blood should be cultured on blood agar in
the presence or absence of oxygen and observed for beta-hemolysis.
Specific tests: Bacterial colonies can then be further analyzed for group-specific Lancefield antigens
using immunofluorescence (IF). Another test used in labs is bacitracin sensitivity; unlike other Strep
species, Gr. A Strep growth is inhibited by this antimicrobial agent.
Serology: usually used for diagnosis of post-strep sequel such as rheumatic fever. These test for
anti-streptolysin O (ASO) Ab.
Rapid Ag test: Many newer tests have been developed that test reactivity of lab anti-GAS antibodies
with group-specific carbohydrates in the cell wall. This can be done DIRECTLY on the throat swab
and takes ~20 min. However, due to the novelty of these tests, negative results should be confirmed
by culture. Procedure of Rapid Ag test: patient specimen is mixed with beads coated with rabbit antiGAS Ab. This mixture is then applied to a membrane and diffuses down to the lane that contains
additional immobilized rabbit anti-GAS antibodies. If the beads carry GAS antigen they become
trapped in first lane. Excess beads that do not carry GAS antigen get trapped in the last lane. This
lane contains goat anti-rabbit antibodies against bead-bound antibodies. There are always an excess
of beads, so even in infected
patients there should be a reaction
in the last lane. This lane serves as
a positive control. If both the middle
and the last lane appear red (due to
the aggregation of the beads) then
the patient is GAS positive, if only
the last lane appears red, then the
patient is negative.
231

MERP Medical Bacteriology

Sreptococcus agalactiae (Group B Strep)


Pathogenesis:
GBS is the main cause of meningitis in the first few days of
infant's life. The organism can gain access to amniotic fluid or
colonize the infant during pregnancy and delivery. Group B strains
contain different polysaccharides in their capsules, which
determines their ability to colonize and cause disease. GBS also
expresses hydrolytic enzymes that facilitate tissue destruction and
spread of the pathogen. Up to 30% of women are vaginal carriers
of this bacteria and are thus under the greatest risk of passing this
organism to the baby.
Diseases:
Early-onset disease occurs when GBS is acquired in-utero; the
disease develops during the first few weeks of life and is
characterized by bacteremia, meningitis, failure to thrive and
pneumonia.
Prenatal care includes screening for GBS in the third trimerster.
As a result of rapid diagnosis and better supportive care, the
mortality rate has decreased to 5-20%.
Many infants that survive the disease have permanent problems
including blindness, deafness, and retardation.
Not all infants born to Group B strep positive moms will have the disease (about 50% will be
infected with bacteria but not progress to disease). A high titer of maternal anti-Group B strep
antibodies usually cross the placenta and protect the infant.
In colonized pregnant women penicillin is administered during delivery to minimize contact
of bacteria with the neonate.
Late-onset neonatal disease is acquired from an exogenous source after birth. Manifestations are
bacteremia and meningitis. Survival rate is high, but neurological problems are common.

Infections in adults can cause urinary tract infections in pregnant women after childbirth. In men and
non-pregnant women infections are rare and include pneumonia and soft tissue infections. Only in
immunocompromised people can these infections lead to serious problems.

232

MERP Medical Bacteriology

Diagnosis:

Culturing of blood or CSF of affected neonates or adults, or vaginal specimen of pregnant


women in blood-enriched agar would produce large colonies that may or may not be betahemolytic.

A CAMP test (right figure) can be done on the isolates. This test
relies on the fact that most S. agalactiae strains produce a
diffusible, extracellular compound that will, in conjunction with a
specific beta-hemolysin of Staphylococcus aureus, cause
complete lysis of sheep red blood cells in an agar medium. Note
the complete hemolysis inside the blue circle in figure on the
right.

Usually a selective medium is also used for culture. This media

CAMP test

contains antibiotic that suppress the growth of other organisms.


Streptococcus pneumoniae
Epidemiology:
The bacterium generally resides in the nasopharynx and is carried asymptomatically in
approximately 50% of healthy individuals

Leading cause of community-acquired pneumonia which is a class of pneumonias in someone


who has not recently been in a hospital

Attaches to human nasopharyngeal epithelial cells via bacterial surface adhesins

Under certain situations can spread to nasal sinuses, lungs and middle ear which are normally
sterile sites

It can also be transported to the blood and the brain

The normal action of mucus and ciliated cells usually prevent bacterial entry into the lungs, but
when these barriers are damaged, bacteria gains access to lungs, and subsequently to blood

Stimulates local inflammatory response, but evades phagocytic killing

Transient colonization of throat and nasopharynx occurs by a succession of different


serotypes

Serious disease occurs most commonly in young children or elderly adults

Often preceded by respiratory viral infection (e.g., influenza, rhinovirus)

Several conditions interfering with normal clearance of bacteria predispose host to pneumococcal
infections:

Chronic pulmonary disease


233

MERP Medical Bacteriology

Alcoholism
Neutrophil dysfunction
Congestive heart failure
Diabetes mellitus
Chronic kidney disease
Splenic dysfunction or splenectomy: Absence of spleen decrease clearance of opsonized
bacteria from blood and also causes a defect in antibody production

Morphology and Physiology:

Encapsulated gram-positive diplococci or short chains


The capsular antigen in virulent strains is made of
complex polysaccharide (there are more than 90
different types of capsule)
Produce alpha hemolysis when aerobically and beta
hemolysis when grown anaerobically
Anaerobic growth may show beta-hemolysis due to the
production of pneumolysin by bacteria which is an
oxygen labile pore-forming toxin
Fastidious growth requirements
o Enriched media with 5% defibrinated blood
o This media provides catalase for destruction of H2O2
S. pneumo ferment carbohydrates; major endproduct is lactic acid
Accumulation of hydrogen peroxide inhibits growth

Virulence Factors:
1. Adhesion proteins
allow bacteria to bind to
oropharyngeal
epithelium and enable
initial colonization
2. Secretory IgA
protease assists
spreading by
enzymatically disrupting
secretory IgA (sIgA)
clearance of bacteria.
3. Pneumolysin functions as multifactorial cytotoxin. It assists spreading by destroying ciliated
epithelial cells though host cell membrane cholesterol binding. It also stimulated complementmediated inflammation.
4. Capsule-mediated phagocytic survival allows bacteria to escape from neutrophils
234

MERP Medical Bacteriology

Pathogenesis:
Pneumonia
Pneumonia is characterized by severe shaking chills, fever, productive cough with blood-tainted
sputum and chest pain. Due to the lower oxygen tension preferred by bacteria, it is usually seen in
the lower lobes of the lungs. Tachycardia and tachypnea are often seen. Recovery is rapid in the
presence of antibiotics. Without antibiotics, the recovery is associated with high levels of opsonizing
antibody.
Facts about pneumonia:

When endogenous oral pneumococci are aspired into lung alveoli it causes aspiration
pneumonia.
Local inflammation causes
pulmonary capillaries to leak fluid
into the interstitium and bacteria
further multiply in nutrient-rich
edema.
Capillaries may get damaged
and erythrocytes leak from
capillaries causing bloody
sputum.
Phagocytic cells (neutrophils,
followed by alveolar
macrophages) migrate and phagocytose and destroy pneumococci.
Children and elderly may have diffuse bronchopneumonia.
Pneumococcal lesion can be observed by X-ray.
Radiological resolution (clean chest X-ray) within 2-3 weeks with antimicrobial therapy

Sinus infections (sinusitis) and otitis media


These infections usually follow a viral infection after which the inflammatory cells obstruct the sinuses
and ear canal, creating a moist, warm environment allowing bacteria to multiply.

Ear infections are usually in kids, sinus infections could occur in patients of all ages.

235

MERP Medical Bacteriology

These infection are often preceded by upper respiratory tract viral infection with PMN caused
obstruction of sinuses and ear canal, preventing air flow and clearing.

Meningitis:

Bacteria can then spread to CNS after a sinus or ear


infection. Alternatively it can enter CNS following
bacteremia that can accompany pneumonia.

This organism is now the leading cause of bacterial


meningitis in older children and adults.

Mortality and severe neurological complications are 420 times more likely than with meningitis caused by
other organisms

Bacteremia
Very common in people with pneumonia or meningitis.

Not common in people with ear or sinus infections.

Immune Response:

Naturally acquired active immunity: Type-specific anticapsular immunity during a natural


infection

Artificially acquired active immunity: Immunization via polyvalent vaccine as prophylaxis.


Streptococcus pneumoniae vaccine is given as either

236

MERP Medical Bacteriology

1) Prevnar 13; PCV13 is a 13-valent conjugated vaccine for kids under 2 years old. PCV13
was licensed in February 2010, supplanting PCV7. PCV13 is identical in formulation for the
previous vaccine and contains seven common serotypes in PCV7, with six additional antigens.

2) PPSV23 is a 23-valent polysaccharide vaccine for anyone over 65 years of age and adults
at high risk. For the 23-valent vaccine, purified capsular material from most common
serotypes (23 different polysaccharides covering 94% of clinically relevant serotypes) is
administered.
Diagnosis:
1. Gram-stain of the sputum
shows G+ diplococci.
Quellung reaction is seen
when anti-capsular
antibodies are added to the
bacteria and examined
under a microscope. It is defines as observation of bacteria swelling as a result of greater refraction
around bacteria in the presence of antibodies.
2. Culture of sputum, CSF or aspirate from sinus or middle ear can be inoculated into enriched
medium with blood and antibiotics that selectively inhibit the growth of other bacteria. Since Strep is
present in normal flora of nasopharynx and upper respiratory flora, the sputum results are hard to
interpret and other types of lower respiratory tract specimen must be obtained.
3. Biochemical test: The biochemical test often done on this organism is bile solubility. Because a
bacterial enzyme that causes autolysis is activated by bile, placing a drop of bile on the bacterial
colony would rapidly dissolve the colony.
4. Specific tests: A recent test (ELISA) for detection of bacterial antigens secreted in the urine is
also available. Usually ELISA is done by testing for excreted capsular antigen.

Practice questions:
1. Which of the following is the main mechanism behind rheumatic fever caused by Streptococcus
pyogenes?
a. IgE mediated mast-cell activation which leads to heart damage
b. CD8 T cell invasion into the myocardium with the subsequent lysis of the heart cells
c. IgA recruitment of the inflammatory molecules and complement
237

MERP Medical Bacteriology

d. IgG binding to the heart tissue causes subsequent inflammation


e. Group A streptococcus invades the heart and secretes super-antigen exotoxins
f. Immune complexes containing Streptococcal antigen and anti-Streptococcal antibody get
deposited in the heart valves causing damage
2. A 15 year old boy comes in to clinic with pain in his right ear. He is on his school swim team and
spends about 2 hours per day in the pool. Along with the pain in his ear, he has watery drainage
coming from the ear canal. On exam his vital signs are normal, and he appears healthy, but his
left external ear looks quite red and there is copious watery, slightly green drainage coming from
the ear. The organism mostly likely causing this is:
a.
b.
c.
d.
e.

Staph aureus
Adenovirus
Streptococcus pneumoniae
Streptococcus pyogenes
Rhinovirus

3. Which of the following statements about the 23-valent pneumococcal vaccine is not correct?
a. It is a protein-conjugated, polysaccharide vaccine
b. It is poorly immunogenic in young children and immunocompromised hosts
c. It is routinely recommended for immune competent adults and children >2 yrs. of age at risk for
serious pneumococcal disease
d. It protects against the major serotypes of pneumococci causing infection
e. An adult with asplenia would be a candidate for this vaccine
4. All of the following statements about the M-protein of Group A Streptococci are correct EXCEPT
a. There are over 80 distinct serotypes.
b. Antibodies to M proteins exhibit crossreactivity with heart tissue
c. Antibodies to M protein confer type-specific immunity.
d. M protein is the major virulence factor of Group A streptococci.
e. M protein is the major constituent of the capsule of Group A streptococci.
5. A 12 year old boy presents with acute onset of sore throat, fever to 38.9 degrees C and painful
anterior cervical lymphadenopathy. On exam the pharynx is red and swollen and the tonsils are
covered with yellow-white exudate. The child also has halitosis. Which of the following nonsuppurative complications are of concern?
a. Sinusitis
b. Acute rheumatic fever alone
c. Acute glomerulonephritis alone
d. Acute rheumatic fever and acute glomerulonephritis
e. Scarlet fever alone
6. Which of the following Streptococcal infections is commonly prevented by a vaccine?
a.
b.
c.
d.

Strep throat
Group B strep meningitis
Strep. pneumoneae meningitis
Flesh-eating bacteria disease
238

MERP Medical Bacteriology

e. Skin infections
7. A 2 day old infant born to a mother who has not received any prenatal care is brought to the
emergency room. The child has a fever, is febrile, and will not feed well. His CSF analysis shows
low glucose level, but elevated protein and white blood cell levels. Gram-positive cocci are
isolated from the CSF. Which of the following would have prevented this disease in the baby?
a.
b.
c.
d.
e.

Intravenous antibiotic administration to the mother during labor


Vaccination of the mother
Treatment of neonates eyes with the antibiotic erythromycin
Vaccination of the baby at birth
Avoidance of sushi and other raw products by the mother

8. Which of the following diseases caused by Staphylococcus aureus is not mediated by a toxin, but
rather by bacterial replication?
a.
b.
c.
d.
e.

Food poisoning
Toxic shock syndrome
Scalded skin syndrome
Impetigo
Scarlet fever

239

MERP Medical Bacteriology

LECTURES 22 AND 23: FOOD INFECTIONS AND FOOD POISONING


Learning Objectives:
1. Differentiate clinically between general symptoms of food poisonings (Staphylococcus aureus,
Bacillus cereus, Clostridium spp.) and food infections (E. coli, Salmonella, Shigella, Cholera,
Listeria).
Describe structurally and biochemically bacteria that cause food poisonings and food
infections
For E. coli, differentiate the strains that cause inflammatory versus non-inflammatory
diarrhea and know the functions of toxins for EHEC and ETEC
For Salmonella discuss the differences between strictly enteric infections and S. typhi
Describe the various forms of disease caused by Shigella and the site of the infection
Describe the function of Shigella toxin
Describe the function of cholera toxin, relate its function to disease and understand the
mechanism of action of oral rehydration
Describe the use of, and identify growth on of McConkey and Hektoen media
Describe the composition of the vaccine where one exists
For Bacillus, describe the clinical symptoms and virulence factors for anthrax
Be able to differentiate biochemically B. cereus from B. anthraces
For Clostridium, describe skin disease caused by C. perfringens and neurological
disease symptoms for C. botulinum, as well as disease mechanism for C. tetani
For E. coli describe UTI and neonatal meningitis as well as septicemia
Biochemically distinguish the three causes of neonatal meningitis: E coli, GBS and
Listeria
Bacilli Gram-positive rods, facultative anaerobic or strictly
aerobic, form spores, catalase positive
Bacillus anthracis
Background:

Spores of B. anthracis gain access to the soil from the


carcasses of large animals such as sheep, goats,
horses, and cattle that die of disseminated disease.
The organism can be harbored in the gastrointestinal
tract of animals that do not develop symptoms and also
in animals that die of disease.

General features:

The organism is a gram-positive, spore-forming nonmotile, rod with characteristic square-cut ends.

240

MERP Medical Bacteriology

They grow singly, in pairs, or in long chains and are encapsulated.

The spores of B. anthracis survive in the soil for decades and, like spores of the Clostridia, are
highly resistant to physical and chemical agents. The spores resist heating at 100oC (i.e.,
boiling water and steam) for up to 30 min; the spores may be killed after heating to 121C for
15 min or with steam-under-pressure in an autoclave.

The organisms are facultative anaerobes that grow best aerobically on blood agar at 37 oC and
are non-hemolytic.

Colonies appear rough with an irregular edge that gives a "Medusa head" appearance.

Virulence factors:
Exotoxin:

Diagram of the actions of the secreted anthrax toxins. LT (lethal


toxin) has a devastating effect on immunity, impairing the
immunogenic response by destroying macrophages, which
release the bacteria into lymph, and induce further spread of B.
anthracis and septicemia. Most importantly, the cellular effects of
LT on the heart result in decreased stroke volume and cardiac
output, which are further diminished by the effects of
hypovolumia, resulting in hypotension and death.

The extracellular product associated with the pathogenicity of the organism is an exotoxin.

The exotoxin is a heat-labile protein composed of 3 components referred to as "protective


antigen" (PA), "lethal or toxic factor" (LF), which is responsible for most of the toxicity, and
"edema factor" (EF).

Although the mechanism of action is not well understood, maximal biological activity occurs
only when all components are present together with the capsule.

Structure:

Capsule is composed of a polypeptide made up exclusively of D- glutamic acid.

The capsule exhibits anti-phagocytic activity and does not stimulate protective antibody.

241

MERP Medical Bacteriology

Transmission:
Transmission appears to be exclusively by the

Transmission of anthrax

exogenous route through contact with spores.


1. Cutaneous anthrax is the result of infection
with spores that gain access to humans
through small abrasions or scratches in the
skin while handling diseased animals or
animal products such as meats, skin, brushes
made from infected horse hair and drums
constructed of infected goat skins.
2. Inhalation anthrax or woolsorters' disease
usually results from inhalation of spores from infected animals or animal products.
Pathogenesis:
Example of cutaneous anthrax

1. Cutaneous anthrax
a. Cutaneous anthrax is both a toxemic and
invasive disease.
b. Spores are introduced into the skin via abrasions
or cuts and germinate.
c. The vegetative cells multiply locally and the host
responds with an acute inflammatory response
characterized by an influx of polymorphonuclear
leucocytes.
d. Phagocytosis by the polymorphonuclear
leucocytes is inhibited by the capsule and the
organisms continue to survive and multiply.

e. The organisms release exotoxin locally and begin to invade adjacent tissue rapidly producing
extensive damage.
f. Within 2 to 5 days, this process manifests as a malignant pustule - a necrotic, black lesion
surrounded by a ring of vesicles containing dark, bluish-black, serosanguinous fluid and exotoxin.
g. Rapid dissemination occurs via the lymphatics to the circulation resulting in septicemia and
tissue invasion, including the lungs.
h. Profound toxemia and necrosis may result in death despite therapy.
242

MERP Medical Bacteriology

2. Inhalation anthrax or Woolsorters' disease


a. This syndrome is also both toxemic and invasive.
b. It is acquired either by inhalation of vegetative cells in droplet nuclei from a patient with
respiratory disease or more commonly by the inhalation of spores from infected animals or
animal products.
c. When spores are introduced, they germinate to vegetative cells in the trachea, bronchi and/or
lungs.
d. Vegetative cells undergo multiplication in these organs with the host response, mechanism of
survival, and exotoxin release as described for the cutaneous form of the disease.
e. Within 24 hours, pulmonary necrosis, septicemia, and meningitis occur which is most often
and rapidly fatal despite therapy.
Inhalation anthrax

Diagnosis:
1. Rapid presumptive diagnosis based upon history and clinical manifestations is essential.
2. Specimens
a. Specimens obtained depend upon the disease process and
include material from lesions, blood for culture, spinal fluid, and
sputum.
b. Gram-stain of the specimen will reveal characteristic grampositive rods with square cut ends and often in pairs or long chains.
243

MERP Medical Bacteriology

c. Spores appear as an oval unstained space in the center of the


organism stained by Gram stain, and may be confirmed by the use
of a special spore stain called malachite green.
3. Primary isolation of cultures and biochemical or direct identification
a. This requires initial cultivation on blood agar.
b. Overnight incubation under aerobic conditions at 37oC is optimal
for isolation of the organism, which is identified as a non-hemolytic,
gram-positive, non-motile, spore forming rod growing singly, in pairs,
or in long chains. Colonies have medusa head appearance.
c. A direct immunofluorescence assay on the isolated organism is
available where antibody from the laboratory is directed against the
capsular polypeptide. PCR tests are also available.
Bacillus cereus
Morphology:
B. cereus is similar to B. anthracis, however it is:

beta-hemolytic on blood agar

non-encapsulated

does not express the edema causing exotoxin.

Virulence factors:

Some strains express a heat-stable emetic toxin (mechanism unknown) and others heatlabile diarrheal toxin which stimulates adenylate cyclase in intestinal epithelial cells
leading to diarrhea.

Pathogenesis:
Bacillus cereus exists in water and soil. The organism may cause a self-limiting type of food
poisoning usually resulting from the ingestion of contaminated rice or meat dishes containing
enterotoxins. The incubation period and clinical manifestations resemble those seen in
staphylococcal food poisoning.

Gastroenteritis:
Emetic food poisoning results from consumption of contaminated rice. The steps in the
development of gastroenteritis:
1. Bacteria in rice are killed from cooking the rice but heat-resistant spores survive if the rice is
inadequately boiled.
244

MERP Medical Bacteriology

2. If rice is not refrigerated, these spores can then germinate at R.T, releasing live bacteria which
multiply and secrete the toxin.
3. This toxin may not be destroyed during reheating of the rice.
4. After ingestion of the food with the toxin, a person gets ill with the symptoms that include:
vomit, nausea, and abdominal cramps.

Diarrheal form results from contaminated meats, sauces and veggies. This disease has a longer
incubation period and lasts longer.

Diagnosis:

For emetic gastroenteritis, suspected food should be cultured.

Stool can be used to identify the organism in patients with diarrheal form of disease; however,
fecal colonization of healthy individuals is common. Isolation of organism is usually done for
epidemiologically related studies.

B cereus

245

MERP Medical Bacteriology

Clostridia:
General features:
The clostridia are relatively large, gram-positive, rod-shaped bacteria.
All species form endospores and
have a strictly fermentative
(anaerobic) mode of metabolism.
Most clostridia will not grow under
aerobic conditions and vegetative
cells are killed by exposure to
atmospheric O2, but their spores are able to survive long periods of exposure to air.
Their fermentation of organic
compounds, such as sugars,
produces large amounts of CO2
and H2 as well as volatile organic
compounds like acetic and butyric
acid, acetone, and butanol.
Metabolism of substrates like
amino acids and fatty acids results
in foul-smelling degradation
products.
Clostridium tetani and Clostridium
botulinum produce the most
potent biological toxins known to affect humans.
o The virulence of C. tetani and C. botulinum is entirely due to their toxigenicity.
o C. botulinum is usually encountered in improperly sterilized (canned) foods in which
endospores have germinated.
o C. tetani is acquired through exposure to the spores of the bacterium Clostridium tetani
which are universally present in the soil.
Clostridium perfringens produces a huge array of invasins and exotoxins and causes wound and
surgical infections that lead to gas gangrene, in addition to severe uterine infections.
o Clostridial hemolysins and extracellular enzymes such as proteases, lipases, collagenase
and hyaluronidase all contribute to the invasive process.

Clostridium perfringens also produces an enterotoxin and is an important cause of food poisoning.
246

MERP Medical Bacteriology

C. perfringens
Virulence factors:

These bacteria can cause a range of disease from mild disease to very severe.

A single strain can produce up to 12 toxins, however the alpha toxin is the most important of these
toxins. This toxin is a lecithinase (phospholipase C) which breaks lecithin (also known as
phosphatidylcholine) in the cell membrane.

The effect of the toxin is to cause massive hemolysis, tissue destruction, myocardial destruction
and hepatic failure.

Beta-toxin makes necrotic lesions in the mucosa.

Other toxins increase vascular permeability and have necrotic activity.

Certain C. perfringens strains produce a superantigen-like entertoxin which is made during the
spore formation and is released together with the spore.

Pathogenesis:
Soft-tissue infection:

C. perfringens can cause cellulites, fasciitis and gas gangrene.


Wounds are colonized by the organism usually through spore
from the environment which germinate and produce vegetative
bacteria in the tissue. Bacteria will then make gas under
anaerobic conditions, and cause accumulation of pus and
muscle pain.

Gas gangrene is caused accumulatiob of hydrogen and CO2


gas.
247

MERP Medical Bacteriology

Alpha toxin splits lecithin into diacylglycerol and IP3 which in turn activate the protein kinase C
second messenger system.

The toxin also causes hemolysis and bleeding by increased vascular permeability and
destruction of platelets, hepatic toxicity, and myocardial dysfunction (bradycardia,
hypotension). Muscle necrosis, shock, renal failure, and death can occur within days.

Food poisoning:

Like other food poisonings this infection has a short incubation period (8-24 hours), and is
characterized by abdominal cramps, no fever and depending on the strain watery diarrhea with
or without vomiting.

It lasts about 2 days. The symptom are usually due to ingestion of spore-contaminated meat
products where the spores, that did not get killed during the original cooking process,
germinated in the food. If the food is not refrigerated, the bacteria divide and secrete
enterotoxin into the food.

Since the toxin is heat-labile, reheating the food will result in destruction of the toxin. However
eating the food without reheating can result in an intoxication.

C. perfringens can also cause a food infection (rather than food poisoning). The mechanism of
food infection is the germination of spores in the spore-contaminated food, rapid bacterial
proliferation followed by the ingestion of a large number of organisms. Once in the persons
gut, the bacteria will go through sporulation which also causes toxin production. When the
spore is released, the toxin is released with it and the person feels the symptoms of
gastroenteritis. This is classified as a food infection because the organism is ingested prior to
toxin production.

Reheating the food prior to ingestion will kill the organism and the toxin it makes.

248

MERP Medical Bacteriology

Necrotizing enteritis:

This is a rare, acute necrotizing process by


which the jejunum is destroyed.

It is characterized by vomit, bloody diarrhea,


abdominal pain, ulceration of small intestine,
and peritonitis (inflammation of a twolayered serous membrane covering both the
surfaces of the organs that lie in the
abdominal cavity and the inner surface of
the abdominal cavity itself) and results in
50% mortality.

Beta-toxin of C. perfringnens Type C strain


is responsible for this disease. The betatoxin can be inactivated by pepsin and
trypsin proteases which are found in people
on protein rich diet: These enzymes are low
in small children and malnutritioned people
so they are more susceptible.

Diagnosis:

Clinical diagnosis is of primary importance. For food poisoning, the recovery of 10 5 organisms
per gram of food or a quantifiable number of bacteria from feces is indicative of infection.

For soft-tissue disease, microscopic detection of these bacteria in specimen in the absence of
leukocytes (which differentiates it from other soft tissue infections) is enough to initiate

249

MERP Medical Bacteriology

immediate therapy. Since gas gangrene is a toxin-mediated disease, there is little leukocytes
infiltration.

Bacteria grows quickly anaerobically. On blood agar, it displays double layer of hemolysis:
inner beta-hemolysis and outer alpha-hemolysis. The beta-hemolysis is by theta toxin and
alpha-hemolysis is by alpha toxin (see figures below).

C. tetani

General features:

The organism is found in soil,


especially heavily-manured
soils, and in the intestinal
tracts and feces of various
animals.

Bacteria are motile, very


sensitive to O2 and extremely
difficult to grow.

They have a drum-stick appearance.

The spores are capable of surviving in oxygen.

250

MERP Medical Bacteriology

Virulence factors:
Exotoxin:

Tetanus is a highly fatal disease of humans.

The disease stems not from invasive


infection but from a potent neurotoxin
called tetanus toxin or tetanospasmin
produced when spores germinate and
vegetative cells grow after gaining
access to wounds. The toxin is also
made during sporulation.

The toxin is a neurotoxin that is heatlabile. It is destroyed at 56 degrees C in


5 minutes.

It blocks the release of inhibitory


neurotransmitters glycine and gamma-amino butyric acid across the synaptic cleft. These
neurotransmitters are required to control and inhibit the nervous impulse.

Presynaptic blockade is specific to the synapse between the inhibitory Renshaw cells and
alpha motor neurons. Only the Renshaw cells that handle the transmission of gamma aminobutyric and glycine are affected, not the Renshaw cells that handle acetylcholine
transmission.

If nervous impulses cannot be checked by normal inhibitory mechanisms, they produce the
generalized muscular spasms characteristic of tetanus.

Upon entering the blood stream the toxin is initially carried to and binds to peripheral nerve
terminals.

The toxin then migrates along neural paths from a local wound to sites of action in the central
nervous system.

It is transported within the axon and across synaptic junctions until it reaches the central
nervous system. There, it becomes rapidly fixed to gangliosides at the presynaptic
inhibitory motor nerve endings, and is taken up into the axon by endocytosis.

Tetanospasmin appears to act by selective cleavage of a protein component of synaptic


vesicles, synaptobrevin II. This prevents synaptobrevin II within the membrane of the vesicle

251

MERP Medical Bacteriology

from fusing with syntaxin within the plasma membrane. This blocks the release of
neurotransmitters by the cells.

Because of the widespread use of the tetanus toxoid for prophylactic immunization, fewer
than 150 cases occur annually in the U.S.

Pathogenesis:
Adult tetanus:

Tetanus is the only vaccine-preventable disease that is not communicable but acquired through
environmental exposure to the spores of Clostridium tetani.

Most cases of tetanus result from small puncture wounds or lacerations which become
contaminated with C. tetani spores which germinate and produce tetanospasmin.

The infection remains localized with only minimal inflammatory damage. The clinical pattern of
generalized tetanus consists of spastic paralysis, severe painful spasms and rigidity of the
voluntary muscles and irritability. The characteristic symptom of "lockjaw" involves spasms of
the masseter muscle accompanied by grotesque, grinning expression.

Spasms of the pharyngeal muscles cause difficulty in swallowing.

Death usually results from interference with the mechanics of respiration.

Neonatal tetanus:

Accounts for about half of the tetanus deaths in developing countries

Neonatal tetanus follows infection of the umbilical stump in infants born to non-immune mothers
(therefore, the infant has not acquired passive immunity). The main symptom of neonatal tetanus
is failure to thrive.

Unlike other diseases, such as diphtheria, recovery from the natural disease usually does not
confer immunity, since even a lethal dose of tetanospasmin is insufficient to provoke an immune
response.

Prevention:
Prophylactic immunization is accomplished with tetanus toxoid, as part of the DTaP vaccine or the
DT (Td) vaccine.
Three injections are given in the first year of life, and a booster is given about a year later, and
again on the entrance into elementary school.
Diagnosis:
Clinical presentation is most indicative. Neither tetanus toxin nor antibodies to the toxin are
detectable in the patient because it is rapidly absorbed by muscle motor neurons and internalized.
252

MERP Medical Bacteriology

Microscopic examination and culture can be done but are not useful because they are not sensitive
enough.
C. botulinum
C. botulinum is a large anaerobic bacillus with endospores. It is widely distributed in soil,
sediments of lakes and ponds, and decaying vegetation. Hence, the intestinal tracts of birds,
mammals and fish may occasionally contain the organism as a transient flora.

Virulence factors:
Exotoxin:

Botulin toxin

Botulinum neurotoxins
predominantly affect the
peripheral nervous system
with a preference of the toxin for
stimulatory alpha-motor
neurons at a neuromuscular
junction.

The toxins bind to the neuron, is


internalized and prevent the
release of acetylcholine
across the synaptic cleft.
o The heavy chain of the
toxin mediates binding to
presynaptic receptors. The A fragment of the toxin enters the cell by receptor mediated
endocytosis.
o The affected cells fail to release ACH, thus producing paralysis of the motor system.
o The recovery of function requires sprouting of a new presynaptic axon and the
subsequent formation of a new synapse.

Seven toxigenic types of the organism exist, each producing an immunologically distinct form
of botulinum toxin.

With a defect in acetylcholine release, the primary symptom is weakness or flaccid paralysis.

Botulinum toxins are very similar in structure and function to the tetanus toxin, but differ
dramatically in their clinical effects because they target different proteins in the nervous
system.

253

MERP Medical Bacteriology

Botulin toxins are designated A, B, C, D, E, F, and G. In the U.S. type A is the most significant
cause of botulism, involved in 62% of the cases.

Pathogenesis:
Food-borne Botulism

In food-borne botulism the botulinum toxin is ingested


with food. The entry of the toxin into GI tract and the
blood stream disseminated it in the body.

C. botulinum spores are relatively heat resistant and may


survive the sterilizing process of improper canning
procedures.

The steps during exposure to botulin toxin are as follows:


The spores from foods grown in soil contaminate a can during canning process
In the anaerobic conditions of the can, the spore germinates and bacteria begin to
divide.

The germination of spores occurs only in the right conditions, i.e. if the canned food is
too acidic, too sugary, or kept refrigerated, the spores will not germinate.
If spores do germinate, then bacteria divide and release botulin toxin.
The toxin is heat labile, so reheating the food before ingestion will inactivate it, however,
because it is a very potent neurotoxin, even small amounts will cause morbidity.
When the food is ingested, the toxin is absorbed by the upper part of the GI tract in the
duodenum and jejunum, and passes into the blood stream by which it reaches the
peripheral neuromuscular synapses.
The toxin then binds to the presynaptic stimulatory terminals and blocks the release of
the neurotransmitter acetylcholine which is required for a nerve to simulate the muscle.

Food-borne botulism is not an infection but an intoxication since it results from the ingestion
of foods that contain the preformed clostridial neurotoxin (an exotoxin with effect on the
nervous system).

Clinical symptoms of botulism begin 18-36 hours after toxin ingestion with weakness, dizziness
and dryness of the mouth. Other symptoms:
o Flaccid paralysis - descending weakness of skeletal muscles
o Nausea and vomiting may occur
o Neurologic features of blurred vision (diplopia), inability to swallow (dysphagia) and
difficulty in speech (dysphonia). These are often called the 3D symptoms.
o Pupil dilation
254

MERP Medical Bacteriology

o Lack of sweating
o Respiratory paralysis
o In botulism, CNS involvement is rare
Infant Botulism:

Infant botulism is due to infection caused by C. botulinum.

The disease occurs in infants 5 - 20 weeks of age who have been


exposed to solid foods, presumably the source of contamination with
spores.

Production of toxin by bacteria in the GI tract induces symptoms.

In adults, neither the organism nor the spores survive the low pH of the GI tract, whereas in the
infant (1-6 months) the high pH and the lack of normal flora allow spores to germinate and the
organism to flourish.

The symptoms in babies are usually failure to thrive. A child diagnosed with infant botulism is
unable to move or open his eyes. His cry is barely audible. His breathing becomes shallow and
this frequently leads to death.

C. botulinum organism as well as the toxin can be found in the feces of infected infants.

Immunity and Prevention:


The toxins that cause botulism are each specifically neutralized by an antitoxin.
As with tetanus, immunity to botulism does not develop, even with severe disease, because the
amount of toxin necessary to induce an immune response is deadly.
The most important aspect of botulism prevention is proper food handling and preparation.
Because the toxin is heat-labile boiling or intense heating of contaminated food will inactivate the
toxin.
A multivalent toxoid evokes good protective antibody response but its use is unjustified due to the
infrequency of the disease.
Diagnosis:
Electrodiagnostic by repetitive nerve stimulation
Testing of multiple muscles may be needed to see increments
Muscle biopsy: scattered angular, small muscle fibers
Analysis of serum, feces & implicated food
C. difficile

Pseudomembranous colitis in humans is caused by overgrowth of Clostridium difficile in the colon,


usually after the normal flora has been disturbed by antimicrobial chemotherapy.

C. difficile produces two toxins: Toxin A is referred to as an enterotoxin because it causes fluid
accumulation in the bowel. Toxin B is an extremely lethal (cytopathic) toxin.

255

MERP Medical Bacteriology

ENTEROBACTERIACEAE
General features:
Gram-negative rods, facultative anaerobes

Catalase positive, Oxidase negative

Ferment glucose to produce acid and gas

Can undergo anaerobic respiration and reduce nitrogen rather than oxygen

In the presence of oxygen, they reduce oxygen, but with an enzyme other than cytochrome C
oxidase which is the reason behind the oxidase negative test result

Selective/differential media is used to isolate different bacteria in this group

Main sites of infection

Media used for isolation:


MacConkey Agar:

Selective and differential

Selective: Contains bile salts and the dye crystal violet, which inhibit the growth of grampositive bacteria and selects for gram-negative bacteria

256

MERP Medical Bacteriology

Differential: Has lactose, which allows differentiation of gram-negative bacteria based on their
ability to ferment lactose
o Organisms which ferment lactose produce acid end-products which react with the
pH indicator neutral red, and produce a pink color. The medium itself is light
pink/yellow in color.

Hektoen Enteric agar: used to culture infected blood or stool

Designed for
differentiation
between Salmonella
and Shigella

Differential:
Differentiates
between bacteria
that ferment lactose
and those that don't
as well as between bacteria that can produce H2S and those that cant

Acid produced from fermenting lactose imparts a yellow-orange color to the medium
due to the presence of a pH indicator (ex. E. coli in top figure)

Non-lactose fermenters do not significantly change the color of the medium (ex. Shigella
in top figure)

Production of hydrogen sulfide gas, turns parts of the medium black

Sodium thiosulfate in the media provides a source of sulfur

257

MERP Medical Bacteriology

Ferric ammonium citrate in the media provides a source of


iron to allow visualization of black precipitate (figure right)

Ex. Salmonella does not ferment lactose and appears light


gray but it also produces H2S which is seen as a black zone
in the middle of colony

H2S-

Selective: Bile salts and the dyes bromthymol blue and acid fuchsin inhibit
the growth of most gram-positive organisms and serve as acid base indicators

Various routes of acquiring Enterobacteriaceae

E. coli
Pathogenesis of E. coli

258

H2S+

MERP Medical Bacteriology

Specific infections:
Septicemia: The E. coli infections that cause septicemia originate in the urinary tract or GI tract. They
cause the highest number of systemic diseases within the
Enterobacteriaceae group.

Urinary tract infections:

These infections are usually due to contamination of


urethra and the bladder by bacteria originating in colon.

Certain bacterial serotypes produce adhesive pili that


attach to the cells lining the bladder and the urinary tract
are prevent the bacteria from being eliminated in the
voided urine.

The bacteria causing UTI could also affect kidney and


prostate.

Neonatal meningitis:

Bacterial strains with K1 capsular antigens are commonly

Urinary tract infections

present in the GI tract of pregnant women and newborns.


Along with Group B Strep., E. coli K1 is a major causes of CNS infections in infants younger
than 1 month.

The capsule is a made of polysialic acid polysaccharide.

These E. coli strains invade the blood stream of infants from the nasopharynx where the
infants first come in contact with bacteria during delivery and the bacteria are then carried to
the meninges.

Gastroenteritis: Virulent strains causing GI infections differ from nonvirulent E coli only in
possessing genetic elements for virulence factors. For example, strains producing enterotoxins are
called enterotoxigenic E coli (ETEC) and can cause disease in humans. Strains that cause
gastroenteritis (such as ETEC) are not part of normal flora.

Depending on the virulence factors, virulent Escherichia coli strains can cause either
non-inflammatory diarrhea (watery diarrhea) or inflammatory diarrhea (stools containing
any or all of the three: blood, mucus, leukocytes).
259

MERP Medical Bacteriology

Pathogenesis:

Transmission is by the fecal-oral route.

Pathogenesis involves adhesins and/or exotoxins.

Pili (fimbriae) allow the bacteria to colonize and adhere to the ileal mucosa.

Cytotoxic enterotoxins (encoded on plasmid or bacteriophage DNA) induce watery diarrhea.

Epidemiology:
Infection is common where sanitation is poor; both infants and susceptible travelers to developing
countries are particularly at risk. The disease is most serious in infants.
Diagnosis:
The diagnosis is suggested by the clinical picture and confirmed by stool culture. Serotyping and tests
for virulence factors are occasionally performed for outbreaks.
Control:

Prevention depends on sanitary measures to prevent fecal-oral transmission, hand-washing


and proper preparation of food, chlorination of water supplies, sewage treatment and disposal.

Parenteral or oral fluid and electrolyte replacement is used to prevent dehydration.

Broad-spectrum antibiotics are used in chronic or life-threatening cases.

Summary of the characteristics of various gastroenteritis-causing E. coli strains:


EPEC (Enteropathogenic)
The most important virulence
factor is attachment
Bundle-forming pilus-mediated
attachment
Non-fimbrial (non-pilus)mediated adhesion via intimin

Moderately invasive (not as


invasive as Shigella or EIEC)

Does not produce exotoxin

Causes infantile diarrhea; watery


diarrhea similar to ETEC

Little/no inflammation, and no


blood or WBC in the stool

260

MERP Medical Bacteriology

No fever; symptoms probably result mainly from mild invasion of the cell and loss of absorption
rather than toxigenesis

ETEC (Enterotoxigenic)

The most important virulence factor is exotoxin


Bacteria is non-invasive to the cell and stays extracellular
Exotoxins can penetrate cells

Watery diarrhea in infants and travelers

No inflammation

No fever

EIEC

The most important virulence factors are adhesins and ability to move laterally within cells
Non-fimbrial adhesins
This strain is invasive, i.e., they penetrate and multiply within epithelial cells of the colon similar
to Shigella but EIEC do not produce Shiga toxin

Symptoms include: inflammatory, dysentery-like diarrhea with mucous, white blood cells and
erythrocytes in the stool

Fever

EHEC

Found in cattle GI track and fecal matter of cattle

Acquired by eating undercooked meat, unpasteurized milk or


contaminated apple cider, lettuce and other uncooked food

Moderately invasive to the cell

The most important virulence factors are adhesins and Stx-1 (shiga-like) toxin
o Adhesins are not well characterized, probably intimin
o Shiga-like toxin

Shiga-like (Stx-1) disrupts ribosome structure and


inhibits protein synthesis (Figure right)

Shiga-like toxin targets: colonic mucosa cells, RBCs


and endothelial cells of the capillaries and kidney
tubular cells
261

MERP Medical Bacteriology

Symptoms include: pediatric diarrhea, copious bloody discharge (hemorrhagic colitis), intense
inflammatory response that may be complicated by hemolytic uremic syndrome (HUS) and and
thrombotic thrombocytopenic purpura (TTP)

Common in developing countries

Fewer than 100 bacteria can cause disease

Main cause- strain O157:H7

Receptor for toxin is more common in kids

Invasive due to the toxin rather than bacteria, so leukocytes may be absent in stool

Various attachment means of E. coli:


EPEC (e.g. E. coli O127:H6) and EHEC
(e.g. E. coli O157:H7) have intimin for
attachment. It belongs to a class of
proteins called attaching and effacing
(A/E) proteins. They and other
virulence factors are responsible
for diarrhea.

Pilus/Fimbria

Intimin

ETEC have pilus which adheres to the


microvilli of small intestinal epithelial
cells and delivers enterotoxins

Salmonella
General features:

Motile by peritrichous flagella (H antigen). It is uniformly distributed over the body.

Ferment neither lactose nor sucrose and produce abundant H2S.

Genus contains 3 species identified on basis of biochemical activity, metabolic end product
difference, and antigenic difference.
a. S. typhi
b. S. cholerae-suis
c. S. enteritidis (1700 + serotypes)

Collectively, clinical syndromes include: gastroenteritis, septicemia, enteric fever,


asymptomatic carriage
262

MERP Medical Bacteriology

For Salmonella, animals are main reservoir of human disease except for bacteria responsible
for typhoid fever

At risk: anyone consuming foods contaminated with large numbers of Salmonella, particularly
children younger than 1 year old, elderly, patients with reduced gastric acids, and patients with
AIDS, for S. typhi foreign travelers or individuals exposed to carriers

Transmission is through the ingestion of contaminated food products (especially poultry, eggs,
dairy products) and direct human to human fecal-oral spread for typhus

Pathogenesis:
Gastroenteritis:
Many infections are due to ingestion of

Transmission of Salmonella

contaminated food, especially poultry (see figure


on the right)

Organism can grow while food is stored outside


the fridge

Can also be transmitted to humans from reptiles

Onset at about 48 hours after ingestion of


contaminated food

Characterized by vomiting, diarrhea, colicky abdominal cramps, headache and fever (see
figure on bottom right).

Bacteria invade the GI epithelium through M cells, invade the Peyer's patches and migrate to
lamina propria of the small intestine. An
acute inflammatory reaction is then
initiated.

Some strains appear to produce an


enterotoxin that stimulates adenylate
cyclase activity (cholera-like).

Usually of short duration

No septicemia

263

MERP Medical Bacteriology

Spread of Salmonella

264

MERP Medical Bacteriology

S. typhi
Enteric fever (typhoid fever):

Transmission of typhoid fever in the body

265

MERP Medical Bacteriology

Incubation period is about 10 days.

The pathogenesis of enteric fever is more complex than that of watery diarrhea or dysentery.

See figure below: bacteria invade mucosa of the distal small bowel through M-cells or Peyer's
patches of small intestine (1), then pass into mesenteric lymph nodes where they multiply (2)
and then enter the blood stream via the thoracic duct (3).

Subsequent spread outside the bowel to the liver, gall bladder, biliary tract, mesentery and
spleen may occur (4).

Multiplication in the gall bladder (5) and spill over into the biliary tract leads to seeding the
intestine with large numbers of bacteria (6).

Little or no diarrhea is present upon initial infection, but diarrhea may be present later in the
course of the illness

The most prominent features are fever and abdominal pain, which develop gradually over a
few days.

Bacteria multiply in

macrophages and
monocytes and in
the organs that
3

house them.

Characterized by

6
4

prolonged febrile
illness with
5

splenomegaly,
leukopenia,
abdominal pain,
and positive blood,
urine and stool
cultures.

Rose spots on the

Rose spots

abdomen reflex systemic involvement

Infection of intestinal lymphoid tissue may lead to necrosis


and ulceration.

266

MERP Medical Bacteriology

Bacteremia is common, occasionally causing a chronic infection in the gall bladder. This
infection may or may not be asymptomatic.

Although it is usually self-limiting, enteric fever carries a significant risk of serious disease and
significant mortality.

Shigella Bacillary dysentery


General features:

Non-motile - no H (flagella) antigens


Closely related to E. coli
Four species that are pathogenic for humans:
o S. dysenteriae- 10 Serotypes (Group A)
o S. flexneri- 6 Serotypes (Group B)
o S. boydii- 15 Serotypes (Group C)
o S. sonnei- 1 Serotype (Group D)
S. sonnei most common in U.S

Some strains produce a protein synthesisinhibiting toxin

Epidemiology:
Humans and higher primates are the only
reservoirs
Transmitted by oral-fecal route
Very low infectious dose: ~ 200 bacteria. So
sanitation can be fairly good and transmission will still occur.
High incidence of unapparent infection makes control difficult
General pathogenesis:

Invasion of the large bowel


via M cells

Infection of macrophages in Peyers


patches

Infection of basolateral side of the


epithelium

Cell-to-cell spread

Migration to adjacent cells occurs via


filopoidal-like protrusions

267

MERP Medical Bacteriology

Diseases
Mild acute diarrheal disease (ADD)
Fever
Abdominal pain
Vomiting
Watery stools may progress to dysentery (exudate with blood, pus, etc)
Last 1 to 7 days without treatment
No bacteremia
Dysentery - more severe diarrhea

Begins with the rapid onset of frequent intestinal evacuations, but the stools are of smaller
volume than in watery diarrhea and contain blood and pus (including WBCs).

High fever, abdominal pain, cramps, and tenesmus are frequent complaints.

Vomiting occurs less often.

In dysentery, the focus of pathology is the colon. Organisms can produce inflammatory and/or
destructive changes in the colonic mucosa either by direct invasion and/or by production of
cytotoxins.

Dysentery is usually due to S. dysenteriae


S. dysenteriae are invasive to the cell, but do not penetrate to the bloodstream, and
produce a potent toxin with the following properties:

It inhibits protein synthesis

Toxin cleaves off an adenine molecule from 28S ribosomal RNA causing a
structural alteration in 60S ribosomal subunit.

Usually toxin only effects intestinal epithelium, but can be disseminated in blood
and cause renal failure (HUS: hemolytic uremic syndrome) or neurological problems.

This damage produces the pus and blood seen in the stools, but does not result in substantial
fluid loss because the absorptive and secretory capacity of the colon is much less than that of
the small bowel.

Inflammation of colon mucosa >> necrosis >> ulceration

Necrosis is probably due to inflammatory reaction, endotoxin, toxic metabolic


products and cytotoxin

Dysenteric infections generally last longer than the common watery diarrheas, but most cases
still resolve spontaneously in 2 to 7 days.

Without treatment dysentery leads to dehydration-related shock and has a mortality rate of
20%.
268

MERP Medical Bacteriology

Diagnosis:
Direct IF of stool sample, culture stool on Hektoen media, biochemical tests on the colonies. Slide
agglutination tests using anti-O Ag antibodies can also be done on the stool isolates.
Cholera (not an Enterobacterieae)
General features:
Gram-negative, facultative anaerobic
Comma-shaped bacillus
Oxidase positive
Motile
Catalase positive
Optimal growth: 18-37 C; pH - 7.0, tolerates up to
pH 9
V. cholerae can grow with or without salt, but
other Vibrios prefer salt environment
There are 139 serogroups of V. cholerae based on
O-somatic antigen (O-polysaccharide side chain of lipopolysaccharide)
V. cholerae endemic in regions of Southern and Southeastern Asia and is responsible for
pandemic cholera
Vibrio cholera
Transmission:
Vibrio species survive in fecal-contaminated water and grow well in increased salt environment
Unlike other Vibrio species cholera do not require salt to grow.

Shellfish often carry this organism. So the spread is usually by ingestion of fecal- contaminated
food or water.

Since a very large number of bacteria are needed to cause disease (108), direct person to
person transmission is rare.

Found in area of poor sanitation

Virulence factors:
Pili helps attach to small
intestine

Cholera toxin - encoded by


a bacteriophage and
functions in activating
adenylate cyclase which, in
turn, converts ATP to cAMP

269

MERP Medical Bacteriology

increasing its levels. cAMP activates Protein Kinase A. This results in the opening of CFTR
chloride channel:

When the CFTR channel is forced to open, it is sends chloride ion to the lumen, sodium
will follow chloride and water will follow both electrolytes. Bicarbonate from pancreas
and potassium from diet are not reabsorbed. This results in secretory isotonic diarrhea.

Hypersecretion of water follows the concentration gradient which ultimately results in the
induced secretion and reduced absorption of electrolytes from the lumen.

Main sites of toxin function are


the crypt cells of
small intestine
which induces
hypersecretion.

Absorption is
effected less, and
thats the reason
why rehydration
therapy can work
in cholera.

Fluid loss can


approach 15 to 20
liters/day

Pathogenesis:
Gastroenteritis or Cholera:
Infections range from mild disease to severe fatal diarrhea, and depends on whether the strain
carries cholera toxin.

V. cholera can attach to the epithelial tract by toxin co-regulated pili and prevent the flushing
out of the organism during fluid loss.

Rapid onset of watery, non-bloody diarrhea, and colorless, odorless stool, speckled with
mucus

Vomiting

The name rice-water stools refer to cholera disease where mucus specks are found in the
stool.

Fever, and presence of leukocytes or blood in the stool are uncommon.


270

MERP Medical Bacteriology

Severe electrolyte loss due to CTFR channel disruption can lead to dehydration, metabolic
acidosis (HCO3 loss), hypovolemic shock, hypokalemia (potassium loss), and renal failure.

Kidney failure occurs when the kidneys shut down from increased constriction as a result of
sympathetic activity. They then lose their filtering ability leading to build up of excess amounts
of fluids and wastes in the body

Prompt replacement of electrolytes reduced the mortality rate from 60% to 1%.

In the absence of cholera toxin, a less violent disease similar to gastroenteritis is produced.

Cholera serotypes O1 and O139 are the most severe.

Cholera can resolve spontaneously.

Diagnosis:

Stain: Bacteria is very thin and curved and are difficult to visualize by gram stain.
o Darkfield microscopy is specialized illumination technique that capitalizes on oblique
illumination to enhance contrast in specimens that are not imaged. A bright image of the
specimen is superimposed onto a dark background

Culture: Vibrios grow on most media including blood agar and


MacConkey agar. Therefore a selective medium for cholera is
used.
o The most typical medium is called thiosulfate citrate bile salts
sucrose (TCBS) agar. It is selective and differential
Selective:

The high concentrations of thiosulfate and citrate and the


strong alkalinity of this medium largely inhibit the growth of Enterobacteriaceae.

Alkaline pH of 8.0-9.0 selects Vibrio and suppresses other bacteria

Ox bile and cholate (bile acid) suppress primarily enterococci.

Differential:

Sucrose is added with the pH indicator blue-bromothymol which changes its color to
yellow, when acid is formed (even in this strongly alkaline medium).

Fermenters such as Vibrio form distinctive yellow-orange colonies.

Cholera is a lactose non-fermenter and sucrose fermenter.

Biochemical tests: can then be done on the bacterial colonies to confirm Cholera.

Treatment, Prevention & Control:


Rehydration & support - Oral or IV
Tetracycline
271

MERP Medical Bacteriology

Sanitary and sewage treatment


Water purification
Vaccines are not yet totally efficacious

Comparison of E. coli, Shigella and Salmonella

Summary of lecture

272

MERP Medical Bacteriology

Practice questions:
1. Within a few hours of eating a delicious blinchik, a group of picnickers get violently ill with vomiting,
nausea and abdominal cramps. Symptoms last less than a day. A gram positive, aerobic bacillus
growing in chains is isolated from the food. What else is likely true about this organism?
a.
b.
c.
d.
e.

It forms spores
It produces LOS
It has a poly-D glutamic acid capsule
It encodes a potent endotoxin
It is salt tolerant

2. All Enterobacteriaceae share all of the following characteristics EXCEPT:


a. Ferment glucose
b. Reduce nitrates to nitrites
c. Gram negative
d. Oxidase positive
e. Rod-shaped (bacilli)
3. Which of the following statements regarding enterotoxigenic E. coli are CORRECT?
a. They are important causes of traveler's diarrhea.
b. Transmission occurs from ingestion of contaminated food and water.
c. Disease is caused by production of one or both of two types of enterotoxins.
d. None of the above are correct.
e. All of the above are correct.
4. Two sisters presented to the emergency department with abdominal cramping, fever, and frequent
blood-tinged stools. They indicated that hamburgers that they had eaten 3 days earlier at a
restaurant were pink and undercooked. A methylene blue-stained smear of stool noted very few
leukocytes. What was the most likely cause of their illness?
a. Heat-stable enterotoxin produced by enterotoxigenic (ETEC) E. coli
b. Shiga-like toxin produced by EHEC E. coli 0157:H7
c. Toxin A and toxin B produced by Clostridium difficile
d. Bundle forming pili produced by enteropathogenic (EPEC) E. coli
e. An AB toxin secreted by Vibrio cholera
5. Which of the following is the most likely source of Salmonella typhi infection?
a. Contact via infected person
b. Steamed rice
c. Undercooked chicken
d. Undercooked hamburger
e. Contaminated marine water

273

MERP Medical Bacteriology

6. Which of the following best describes the structure and growth of Cholera?

a.
b.
c.
d.
e.

Ability to ferment
lactose

Encodes a
neurotoxic exotoxin

yes
no
no
yes
no

no
no
yes
no
yes

Tolerates
high salt
medium
yes
yes
yes
no
no

Anaerobic

no
no
no
yes
yes

7. Which of the following best described the clinical symptoms of Cholera?

a.
b.
c.
d.
e.

Fever

Invasive to the cell

no
yes
no
yes
yes

yes
no
no
yes
yes

274

Penetrates the
mucosa
yes
yes
no
no
no

Blood in the
stool
no
no
no
yes
yes

MERP Medical Bacteriology

Campylobacter:
Characteristics:
Spiral, gram-negative rods
No fermentation or oxidation of carbohydrates
Microaerophilic growth,
Growth at 42oC (C. jejuni)
Motile, catalase +, oxidase +
The major carbon and nitrogen sources are amino
acids, and it possesses several enzymes for the
amino acid deamination. Serine catabolism is
especially significant.
Pathology:
Gastroenteritis:
Infectious dose - few hundred bacteria
However bacteria can be killed by gastric acid, so larger inoculation dose or conditions that
neutralize the gastric acid favor disease. Infections are zoonotic and it is usually the
consumption of contaminated milk, food or water that causes the infection.
No person-to-person spread
Diarrhea, fever, malaise, abdominal pain. More than 10-20 stools a day
Stools contain blood
Septicemia is uncommon but happens often with other species of Campylobacter such as C.
fetus.
Histological damage to mucosal surface of the jejunum, ilium and colon
See ulcerated, fluid filled mucosal surfaces, infiltration of neutrophils, eosonophils and
mononucler cells into lamina propria (a thin vascular layer of connective tissue beneath the
epithelium of an organ's mucous membrane)
The important factors for pathogenesis of this organism are adhesins and flagella
A very rare sequel to gastroenteritis has been associated with Guillain-Barre syndrome (a
neurological disorder in which auto-immune response reacts to peripheral nervous system
causing weakness), recovery may take months.
The mechanism of Guillain-Barre is due to cross-reactivity between antigens of C. jejuni and
glycosphingolipids in the surface of neural tissues. Arthritis has also been observed as a
complication for this disease.
Diagnosis:
Stain: Stool specimen can be analyzed by gram stain or dark-field microscopy (it is hard to see the
organism, because it is too thin and very small).
Culture: the organism grows in reduced oxygen (5-7%), and increased CO2 (5-10%). It grows better
at 42oC than 37oC. Selective media containing blood and antibiotics are usually used. Additives that
remove toxic oxygen radicals can also be added. A media used to grow C. jejuni is called Campy
agar contains 10% sheep blood, sodium bisulfite, and three antibiotics. The sodium bisulfite reduces
some of the oxygen in the medium which enhances recovery of Campylobacter. The antibiotics
prevent other gram-negative bacilli and yeast from growing.

275

MERP Medical Bacteriology

Listeria
General features:
All Listeria species are small, Gram-positive rods, which are
sometimes arranged in short chains.
In direct smears they may be coccoid, so they can be mistaken
for streptococci.
Flagella are produced at room temperature rather than at 37C.
Tumbling motility in fluid cultures is due to flagella being
produced at 250C (not at 370C)
Catalase-positive, beta-hemolytic
Can grow at 1oC
Hemolysin production is an important marker for L monocytogenes
Strains of Listeria are usually serotyped to characterize surface
antigens, such as O antigens (teichoic acids) and H antigens
(proteins)
Virulence factors:
An invasion factor secreted by the pathogenic bacteria enables
them to penetrate host cells of the epithelial lining. Bacteria enters
through the GI tract epithelial cells and lyses the acidic vesicles with
listeriolysin O. Listeriolsin O is a cytolytic, thiol-activated, pore-forming
toxin protein that is activated by reducing agents and inhibited by
oxidizing agents. It works best at pH of 5.5. After lysis of the vesicle,
bacteria multiplies in the cytoplasm of the cell, then propels itself
using bacterial protein ActA and host actin into the next cell, where
the process of vesicle lyses and multiplication begins again.
Pathogenesis:
Listeria monocytogenes is ingested with raw, contaminated food. Listeriosis has recently been
recognized as an important
public health problem in the
United States in neonates,
elderly, pregnant women and
patients with defective
cellular immunity. Postpasteurization contamination
is often the cause of an
outbreak. Normally, the
immune system eliminates
the infection before it
spreads. Indeed, most adults
who have no history of
listeriosis have T
lymphocytes primed
specifically by Listeria
antigens. If the immune system is compromised, however, systemic disease may develop. Listeria
monocytogenes multiplies not only extracellularly but also intracellularly within macrophages after
phagocytosis and even within parenchymal cells which are entered by induced phagocytosis. It
therefore belongs to the large group of facultatively intracellular pathogens.
276

MERP Medical Bacteriology

Listeriosis also may be transmitted congenitally across the placenta. The immunocompetent
mother suffers at worst a brief, flu-like febrile illness, but the fetus, whose defense system is still
immature, becomes seriously ill. Depending on the stage of gestation, the fetus is either stillborn or
born with signs of congenital infection. Typically, multiple pyogenic foci are found in several organs
(granulomatosis infantiseptica). The onset of listeriosis is delayed (i.e., a few days after birth) when
infection is acquired during labor by bacteria colonizing the genital tract of the mother.
Diagnosis:
Gram stain of CSF usually shows no organism (it is present below detection level) which is in
contrast to most other bacteria in the CNS. If it is isolated from CSF, it must be differentiated from a
similar-looking G(+) rod (Corynebacterium) which is often a contaminant of specimen from other parts
of the body.
Bacteria can be cultured from blood and CSF on most lab media, and selected by cold enrichment.
Beta-hemolysis is hard to detect on blood agar. Listeria also shows motility in semi-solid agar and
liquid culture.
Biochemical tests and very specific genetic markers for pathogenic Listeria strains are used for
definitive diagnosis.
Control:
Hygienic food processing and storage may reduce the risk of listeriosis. Individuals in high-risk
groups (i.e., immunocompromised individuals and pregnant women) should avoid uncooked food or
should at least marinate salads for a long time in a vinegar-based dressing to kill adherent bacteria.
Since a cell-mediated immune response (the most potent weapon against L monocytogenes) is
induced only by injection of living antigen, vaccination is difficult. Antimicrobial agents are the
mainstay of treatment.

277

MERP Medical Bacteriology

LECTURE 24: BACTERIAL MENINGITIS AND RESPIRATORY INFECTIONS


Learning objectives:
1. Describe the structure and clinical symptoms of respiratory pathogens: Corynebacterium
diphtheria, Bordetella pertussis, Haemophilus influenza

Identify respiratory versus systemic diphtheria symptoms


Describe the function of diphtheria toxin and its role in myocarditis
Explain the mechanism behind the Elek test and the media used for C. diphtheria
List the clinical and structural differences between H. influenzae type b and non-typable H.
influenzae
List the clinical features of epiglottitis, cellulitis and meningitis
List the growth requirements for H. influenza
Describe the clinical symptom of B. pertussis whooping cough
Describe the composition of the vaccines for these pathogens
List the basics about the media used to grow Bordetella

2. Describe clinical presentation of meningitis caused by Neisseria and the symptoms of


meningococcemia

Describe the composition of N. meningitides vaccine


Differentiate biochemically between N. meningitides and N. gonorrhoeae

3. Describe the clinical presentation of gonorrhea in men and women

List the features of gonococcemia and ophthalmia neonatorum


Describe ways to diagnose gonorrhea (media used, biochemical tests, gram-stain, etc.)

4. Describe and differentiate structurally/biochemically the common causes of non-neonatal


meningitis (Streptococcus pneumoniae, Neisseria meningitides, Haemophilus influenzae).
Describe the pathogens that cause neonatal versus non-neonatal meningitis.

278

MERP Medical Bacteriology

Corynebacterium diphtheriae
General features:
Anatomy and histology of upper respiratory tract
The nasal cavity, the upper part of the pharynx, the trachea, and the bronchi are lined with pseudostratified ciliated
columnar epithelium

Diphtheria are small, nonmotile, irregularly staining pleomorphic Gram-positive rods with clubshaped swelled ends

Arrangement of cells in short chains of "V" or "Y"


configurations or in clumps

May contain phosphate rich inclusion bodies, known


as metachromatic granules

Facultative anaerobes produce acid but not gas


from carbohydrates

Catalase positive

Disease causing strains carry a phage-encoded


exotoxin

Widely distributed in nature; worldwide in occurrence

Human is the only natural host

Carried asymptomatically in the oropharynx of immune individuals

279

MERP Medical Bacteriology

Virulence factors:

A bacteriophage-encoded toxin is
responsible for the pathogenicity of
this organism.

Diphtheria A-B exotoxin interrupts


peptide formation at the ribosomal
level

The B component has receptor-binding


region and a translocation region,
whereas the A subunit is the catalytic
subunit.

The A subunit terminates host cell protein elongation by adding ADP-ribose to the EF-2.

The B subunit of the toxin has a receptor on a variety of cells including heart and nerve cells.

The toxin is produced in the presence of limiting amounts of iron

The toxin is distributed to tissues distal of the respiratory tract by the circulation.

Another enzyme, called phospholipase D, increases vascular permeability, thus allowing C.


diphtheriae to spread through tissues of the naso-pharyngeal area

Pathogenesis:
Corynebacterium diptheriae: toxigenic strains with a phageencoded toxin cause diphtheria in humans
Diphtheria - respiratory disease:

Local multiplication in the in the pharynx and adjacent areas


causes initial damage

This is followed by sore throat, low grade fever, exudative


pharyngitis with a thick pseudomembrane (right) as a
result of fluid from the dying cells that coagulates

The pseudomembrane is composed of:


Bacteria
Necrotic epithelium
Macrophages
Fibrin

280

MERP Medical Bacteriology

The pseudomembrane covers tongue, uvula, and can be extended into the nasopharynx and
the larynx.

The membrane firmly adheres to tissue and is difficult to dislodge without causing bleeding.

Spreading of the membrane down the bronchial tree can occur, causing respiratory tract
obstruction and dyspnea.

If the patient recovers, the membrane eventually dislodges.

Toxin may contribute to the initial adhesion of bacteria.

Complications may occur as a result of systemic bacterial spread of the exotoxin to other
target organs in the body such as the heart

Bacteria itself is not invasive to the bloodstream

Most mortality results from systemic toxin-mediated heart failure

Cutaneous diphtheria (extra-respiratory disease)

Acquired by skin contact; organism enters through break in subcutaneous tissue

Chronic non-healing ulcer results

Most strains causing skin disease are non-toxigenic unless associated with an outbreak of
pharyngeal diphtheria.

Diagnosis:
Usually clinical diagnosis is used to give initial treatment
Direct examination: of clinical material is unreliable as several normal
flora inhabitants resemble this organism
Culture: of the nasopharynx or throat specimen is done on special
medium:
Cysteine-tellurite blood agar (right) is selective and differential:

Selective: Tellurite inhibits growth of most upper


respiratory tract bacteria and gram-negative rods, and is reduced by C. diphtheriae
to tellurium to produce a gray color.

Differential: H2S production from cysteine and its interaction with tellurite salt
produces a brown halo around the colonies.

Toxigenicity test called Elek test is done on bacterial cultures (see figure below).

281

MERP Medical Bacteriology

Anti-toxin is placed
vertically on an agar plate
growing C. diphtheriae
(perpendicular to the anti-toxin).
A visible precipitate representing
anti-toxin and toxin complex is
formed between the anti-toxin
soaked filter and the bacteria.
Control of infection:
Antitoxin is used for neutralizing exotoxin
o Effective in conjunction with antibiotic therapy
Antibiotics
Penicillin G
Erythromycin
Vaccine:
Diphtheria toxoid vaccine given as a part of DTaP (diphtheria toxoid, tetanus toxoid, acellular
pertussis) vaccine at ages 2, 4, 5, 15 months and 5-6 years
Bordetella pertussis
General features:
Small, gram-negative coccobacilli
Obligate aerobes
Highly communicable and infectious
Person-to-person spread via inhalation of infectious aerosols
Man is only natural host
Multiply among cilia of epithelial cells
Virulence factors (table right):
Bacterial attached and destroys
ciliated epithelial cells.
Hemagglutinin and special
adhesive pili are used for
attachment.
They promote binding to the sialic
acid on ciliated respiratory cells
Massive proliferation of bacteria.

282

MERP Medical Bacteriology

Production of localized tissue damage by several toxins.


Bacterial toxins stimulate histamine and causes systemic
toxicity.

Pathogenesis: Whooping cough (right table)


This pathogen causes a strictly human disease.
Disease is divided in stages:
1st stage- catarrhal stage- common cold-like.
Most bacteria is produced at this stage and
therefore a person at this stage is very
contagious.
2nd stage- paroxysmal stage ciliated epithelial
cells are extruded from the respiratory tract
and the clearance of mucus is impaired. At
this stage whooping cough develops (series
of repetitive coughs followed by a deep noise inspiration taking a breath). Mucus production can
restrict airways. Often vomiting and exhaustion follows these coughs. In adults the infection can
cause a chronic cough which can be contagious to children.
3rd stage: convalescent stage- paroxysms diminish in number and severity. However secondary
complications may occur. In vaccinated people, organism can colonize but not cause disease a bad cold is often due to B. pertussis outgrowth in immunized people (possibly due to the less
efficient nature of the acellular vaccine). Unvaccinated infants have a very high mortality rate.

283

MERP Medical Bacteriology

Components of the mucociliary system (below): the


cilia (hair type projections of the epithelia cells), the
periciliary fluid layer and the mucus. The mucus is
secreted from the goblet cells and the submucosal
glands. Mucociliary clearance is the primary mechanism
of clearance of mucus.

Diagnosis:

Organism is very sensitive to drying. Therefore the nasopharyngeal aspirate must be collected
on special non-cotton fiber swabs that are low in fatty acids.

Specimen should either be directly cultured or transported in a special medium such as


Regan-Lowe (moist, charcoal-horse blood medium).

Aspirated specimen can be directly observed with fluorescin-labeled anti-pertussis Ab.

Culture:
Regan-Lowe media supplemented with charcoal and starch (to absorb toxic substances in lab
media), glycerol, peptones. Media should also be supplemented with NAD (Factor V) an incubated at
35oC in humidified incubator. Bacteria are slow growing.

Specific assays:
Specific latex agglutination assay (using anti-pertussis specific Ab) and IF tests can be done directly
on the specimen (saves time, b/c culture can take up to 7 days). These tests are used to definitively
characterize the organism.

284

MERP Medical Bacteriology

Prevention:

Vaccine: Acellular B. pertussis and diphtheria and tetanus toxoids are prepared in formalin for
inclusion in DTaP vaccine. The acellular pertussis contains pertussis toxoid, hemagglutinin, and
fimbriae.

Left: Cases of pertussis


declined rapidly in the
1940s. The all-time low
was in 1976, with only 75
cases reported in the
United States. Since the
early 1980s, there has
been an increase in
reported cases of
whooping cough.
Haemophilus influenzae
General features:
Gram-negative rods

Non-motile, facultative anaerobe

Require hemin (factor X) and NAD (factor V) for


growth

Catalase positive, oxidase positive

Non encapsulated organisms from sputum are


pleomorphic and often exhibit long threads and filaments

Virulence Factors:
Encapsulated strains:

Polysaccharide capsule
((polyribitol phopshate (PRP)
found in type b strain and was

285

MERP Medical Bacteriology

the main cause of invasive bacteria before the vaccine.


o Encapsulated H. influenza are not often found normally in the upper respiratory tract but
cause several diseases in unvaccinated kids.
o Encapsulated bacteria can cross epithelial and endothelial cells and enter blood causing
high grade bacteremia.
o After the vaccine for Hib, encapsulated H. influenzae type c and f are responsible for
most diseases.

Non-encapsulated strains:
H. influenzae colonize upper respiratory tract early in life, and can spread locally to cause otitis
media, sinusitis, and bronchitis, but disseminated disease is relatively uncommon.

For both encapsulated and non-encapsulated strains, pili and adhesins impair the function of
the ciliated cells in the oropharynx leading to damage of the respiratory epithelia.

Pathogenesis:

The pathogenesis of H. influenzae infections is not completely understood, although the presence
of the type b polysaccharide capsule is known to be the major factor in virulence.

Encapsulated organisms can penetrate the epithelium of the nasopharynx and invade the blood
capillaries directly. Their capsule allows them to resist phagocytosis and complement-mediated
lysis in the non immune host.

Non-typable (non-encapsulated) strains are less invasive, but they are apparently able to induce
an inflammatory response that causes disease.

Vaccination with type b polysaccharide (in the form of Hib conjugate vaccines) is effective in
preventing infection, and several vaccines are now available for routine use.

Naturally-acquired disease caused by H. influenzae seems to occur in humans only.

In infants and young children (under 5 years of age), H. influenzae type b causes bacteremia and
acute bacterial meningitis.

Nontypable H. influenzae causes ear infections (otitis media) and sinusitis in children, and is
associated with respiratory tract infections (pneumonia) in infants, children and adults.

Disease caused by H. influenzae usually begins in the upper respiratory tract as nasopharyngitis.

H. influenzae does not produce any demonstrable exotoxins

286

MERP Medical Bacteriology

The direct role of endotoxin in meningitis or bacteremia is unclear, although the Gram-negative
bacterium's outer membrane lipooligosaccharide is thought to play a role in inflammation
associated with otitis media.

All virulent strains produce neuraminidase and an IgA protease.

Fimbriae increase the adherence of bacteria to human mucosal cells in vitro, and they are
required for successful colonization of the nasopharynx.

The capsule material is antiphagocytic, and it is ineffective in inducing the alternative complement
pathway.

Infections are mainly pediatric in nature and most of the time infections are caused by patients
bacterial flora (endogenous).
Meningitis:

Between 46 and 60% of all serious Hib-related diseases present as meningitis.

Infuenzae type b was the most common cause of pediatric meningitis in places with no vaccine.

Person-to-person spread is common.

Initial manifestations of meningitis, seen in most patients with Hib meningitis are

Altered cry
Lethargy
Nausea or vomiting
Fever
Headache
Photophobia
Irritability
Anorexia
Seizures

An altered cry in small children is an important feature for this type of meningitis, and appears as
high pitch cry, lethargic moaning or an attempt to cry without a noise.
Lethargy is more common in bacterial meningitis as opposed to viral meningitis and at least half of
the patients experience lethargy with bacterial meningitis. Convulsive seizures are also common
Vomiting is reported as an early manifestation in nearly 50% of Hib meningitis cases. If vomiting
occurs, it generally does so within hours to days after the onset of fever.
This meningitis form has a high level of morbidity although not as high as pneumococcal or
meningococcal disease. There is an up to 10% mortality rate in patients that receive treatment.

287

MERP Medical Bacteriology

Epiglottitis:
Anatomy and histology of epiglottis

Epiglottis is a flap of tissue that sits at the base of the tongue that keeps food from going into
the trachea, or windpipe, during swallowing.

When it gets infected and inflamed, it can obstruct, or close off, the windpipe, which may be
fatal unless promptly treated.

The epiglottis swells, turns bright red and protrudes into the airway.

Without a functional
epiglottis, food could
enter the airways,
and you would cough
or choke after
swallowing.

Patients with
epiglottitis usually
have an upper
respiratory infection followed by fever and stridor (high pitched sound heard on inspiration that
is indicative of airway obstruction).

Patients cant swallow saliva- and present with drooling. Since the development of a vaccine,
there are fewer cases since epiglottitis was mainly caused by Hib.

288

MERP Medical Bacteriology

Cellulitis (right image):

Cellulitis is an inflammation of the connective tissue


underlying the skin, usually caused by a bacterial infection.

Hib cellulitis usually involves the face, head, or neck. Most


cases occur in children aged 2 years or younger.

In Hib cellulitis, patients often present with reddish-blue


patches on cheeks and periorbital (of the tissues
surrounding the eye) areas.
o Hib used to be the most common cause of
periorbital cellulitis but post vaccine,
streptococcal organisms are now the most common cause.

In other areas affected by cellulitis the local inflammatory reaction stems from granulocytic
infiltrations, hyperemia, and capillary leakage.
o This is the basis for the skin disruption inherent in cellulitis. The patient may guard the
tender area. If the cellulitic area overlies a mobile area such as a joint, the patient may
display resistance or anxiety with limb movement, either passive or active.
Additional images of cellulitis

Otitis media, sinusitis, lower respiratory tract disease:

Acute otitis media usually follows a cold after which the ear becomes involved and can cause
severe pain.

The pain will usually settle within a day or two, but can last over a week. Sometimes the ear
will discharge pus, but this is rare.

The tissues surrounding the Eustachian tube swell due to inflammation within. It remains
blocked for a period of time.

289

MERP Medical Bacteriology

The air present in the middle ear is slowly absorbed into the surrounding tissues creating
negative pressure and pain. This pain can be severe because the sensory nerve endings in
the eardrum respond to increased pressure with pain.

Diagnosis:

Stain:

For meningitis- take blood or CSF specimen, for localized infection


take specimen by needle aspiration from affected areas.
o Blood specimen can also be collected for cellulatitis and
epiglottitis.
o Detecting of H. influenza (gram-negative rods that range
in size) is useful in preliminary diagnosis.

Culture:

Chocolate agar (heat blood agar to release V and X factors).

Bacteria can also be detected as small satellite colonies in regular blood culture inoculated with S.
auerus, because Staph. can lyse red blood cells allowing the release of factor X and V. However,
because the blood is not previously inactivated, there are inhibitors of factor V present, and
therefore the colonies of H. influenza are very small (satellite).

Antigen detection latex agglutination test:

PRP Antigen can be detected in CSF and urine (where some of the antigen is eliminated
intact). This test only works for Type b which is rare in US. PRP is detected with particle
290

MERP Medical Bacteriology

agglutination which can detect less than 1 ng/ml of PRP in a clinical specime

Control of infection:

Treatment:

Virtually all patients treated early in the course


of H. influenzae meningitis are cured.

Ampicillin has been the drug of choice, but


presently over 20 percent of all strains of H.
influenzae are resistant to ampicillin because
of plasmid-mediated -lactamase production.

The recommended treatment for H. influenzae


meningitis is ampicillin for strains of the
bacterium that do not make -lactamase, and
a third-generation cephalosporin or chloramphenicol for strains that do.

Vaccine:

The use of protein-conjugated PRP, has vastly reduced the frequency of infection due to type
b H. influenzae.
o The PRP vaccine consists of the type b capsular polysaccharide and either a diphtheria
toxoid or tetanospasmin (also called tetanus toxoid).

291

MERP Medical Bacteriology

Neisseria
General features:

Cells are Gram-negative cocci, usually seen in pairs with the


adjacent sides flattened.
Oxidase positive, catalase positive
Not motile
Both Neisseria gonorrhoeae and meningitides are
considered to be an obligate aerobe; they have been shown,
however, to grow in the absence of oxygen by anaerobic
respiration by using nitrite as a terminal electron acceptor.
Can make acid byproducts from glucose and/or maltose during glucose oxidation rather than
fermentation.
Have lipooligosaccharide (LOS) cmposed of Lipid A + core polysaccharide in addition to the
typical LPS (Lipid A + core polysaccharide + O antigen). Some feature that differentiate
Neisseria gonorrhoeae from Neisseria meningitides are listed in the table below.

Neisseria meningitidis
Virulence factors: (Table bottom right)

The antiphagocytic polysaccharide capsule is a prominent feature.

N. meningitidis is grouped, on the basis of capsular


polysaccharides,
into 12 serogroups,
some of which are
subdivided
according to the
presence of outer
membrane protein and lipopolysaccharide antigens.

292

MERP Medical Bacteriology

Pathogenesis:

Asymptomatic carriage of meningococci in the nasopharynx in 5 - 20% of population provides a


reservoir for infection but also enhances host immunity. Attack rates peak in infants 3 months to 1
year old.

Meningococcal meningitis occurs both sporadically (mainly groups B and C meningococci) and in
epidemics (mainly group A meningococci), with the highest incidence during late winter and early
spring.

Infection is by aspiration of infective particles, which attach to epithelial cells of the


nasopharyngeal and oropharyngeal mucosa, cross the mucosal barrier, and enter the
bloodstream.

If not cleared, blood-borne bacteria may enter the central nervous system and cause meningitis.

Meningococci establish systemic infections only in individuals who lack serum bacterial antibodies
directed against the capsular or noncapsular antigens of the invading strain, or in patients
deficient in the late-acting complement components.

Diseases:
Meningitis:
Purulent inflammation of the meninges is mainly due to types A, B (sialic acid capsule), C, W-135.
Headache, fever, meningeal signs are common
Meningeal signs include:
o Kernigs Sign if positive then a supine patient experiences pain when his/her hips and
knees are flexed at 90% and a physician extends his/her knees further beyond 135
degrees.
o Brudzinskis Sign If positive then flexing the patients neck causes flexion of the
patients hips and knees.
Arthritis and hearing loss can occur
People with complement deficiency are more predisposed for this disease.
This is the most common cause of meningitis in infants (6-24 months old) and young adults
(age18-24).
Sequel of the infection include arthritis and hearing loss. This has a close to 100% mortality rate if
untreated, and 10% in treated patients.
Although the bacteria can be carried in the pharynx for reasons not fully known, it can sometimes
overwhelm the bodys defenses allowing the infection to spread through the bloodstream and to the
brain.
293

MERP Medical Bacteriology

Meningococcemia:

This is a life-threatening disseminated disease.

Early recognition is critical to implement prompt antibiotic


therapy and supportive care.

Treatment must be instituted rapidly because irreversible


shock and death may occur within hours of the onset of
symptoms.

Meningococcal bacteria produce 100 to 1,000 times greater amount of endotoxin than other gram
negative bacteria. As the bacteria multiply and move through the bloodstream, it sheds
concentrated amounts of toxin.

The endotoxin directly affects the heart, reducing its ability to circulate blood.

It also causes hemorrhaging. As some blood vessels start to hemorrhage, major organs like the
lungs and kidneys are damaged.Thrombosis of small blood vessels, petechial rash (pinpoint,
nonraised, perfectly round, purplish red spots
caused by intradermal or submucous
hemorrhaging) and skin lesions can form large
hemorrhagic lesions.

Disease may culminate in intravascular


coagulation, shock and bilateral destruction of the
adrenal glands (endocrine glands above the
kidney).

Of the cases of meningococcal disease, 30-50%


present with meningitis alone, 40% have meningitis with septicemia, and 7-10% have only
septicemic features.

The clinical difference between septicemia and meningitis is important because patients who
present with shock are treated differently than patients who present primarily with increased
intracranial pressure.

Treated with antibiotics

N. meningitides has a more frequent bloodstream spread than gonorrhea.

294

MERP Medical Bacteriology

Diagnosis:

Gram stain: of CSF or blood


Culture:
Specimen from sputum, CSF or blood is grown on
enriched blood agar or Thayer-Martin (TM) media.
Colonies are oxidase positive.

Control:

Control:
Penicillin is the drug of choice. Household contacts
require chemoprophylaxis with rifampin.
Menactra vaccine - a quadrivalent (protects against
serogroups A, C, W, and Y) meningococcal conjugate vaccine given at age 11 12 yr.
Bexsero vaccine as of 2014, this recombinant vaccine is available for Meningococcal
Serogroup B.
Neisseria gonorrhoeae:
Virulence factors: (see table on the right)
Note on endotoxin:
In N. gonorrhoeae the endotoxin is not only
released when bacteria dies but also when it
grows (unlike other gram-negative in which
endotoxin is released during bacterial death).
Levels of endotoxins are much larger in
Neisseria. Lipid A in the bloodstream binds to
LPS binding protein and then is carried to
TLR-4 receptor on macrophages.
Macrophages are activated and secrete IL-1,
IL-6, IL-8.

295

MERP Medical Bacteriology

Pathogenesis:
Gonorrhea is a sexually transmitted disease of worldwide importance.

The highest attack rate in both men and women occurs between 15 and 29 years of age. Hostrelated factors such as the number of sexual partners, contraceptive practices, sexual preference,
and population mobility contribute to the incidence of gonorrhea.

Gonorrhea is usually acquired by sexual contact. However, virtually any mucous membrane can
be infected by this microorganism.

Gonorrhea details:

Gonococci adhere to mucus secreting

PathogenesisPurulent
of gonorrhea
urethral discharge

columnar epithelial cells, penetrate them, and


multiply in the basement membrane. Bacteria
can also reach sub-epithelial space by
traveling between cells. In sub-epithelial layer
bacteria can adhere to polymorphonuclear
leukocytes (PMNs) and get phagocytized by
them. Neutrophils kill some bacteria,
however some bacteria survive and multiply
inside PMNs (see figure on the right).

Adherence is facilitated through pili and opa


proteins. Gonococcal lipopolysaccharide
stimulates the production of tumor necrosis
factor, which causes cell damage.

A typical host response is characterized by invasion of neutrophils, followed by epithelial


sloughing, formation of submucosal microabscesses, and purulent discharge.
o Women are at 50 % risk of infection after exposure to an infected man
In women cervix
is primary site of infection, specifically the infection of endocervical columnar endothelial
cells is most common
50% of infected women may be asymptomatic. Symptomatic women experience vaginal
discharge, dysuria, and ascending infections such as inflammation of fallopian tubes
(salpingitis) and ovarian abscesses. In 15% of women pelvic inflammatory disease (PID=
inflammation of the upper reproductive tract) is observed. This infection can result in infertility
due to tubal scarring and/or ectopic pregnancy.
296

MERP Medical Bacteriology

Repeated infections in sexually active people with multiple partners are very common.
o Men are at 20% risk of infection after exposure to infected woman.
Most men are initially symptomatic.
Urethra is the primary site of infection. Purulent urethral
discharge, and dysuria (pain when urinating) are common when
symptoms are seen. Also common are unilateral testicular pain
and swelling.
Bacteria can attach to sperm and carry it to upper reproductive tract of women

Although serum antibodies can be detected after the infection, it is not known whether these
antibodies are protective against future infection with gonorrhea.

Gonococcemia (Disseminated gonococcal infection DGI):

Gonococci may disseminate via the bloodstream. Strains that cause


disseminated infections are usually resistant to serum and
complement. Individuals with genetic defects in late-acting
complement components are at an increased risk for disseminated
infections. Systemic infection stimulates systemic inflammation.
Protection, if it exists, may be strain specific.

1-3% of infected women experience disseminated infection due to untreated asymptomatic


infections.

Manifestations include fever, arthritis in knees, ankles, wrists, and pustular rash. The rash
presents as small elevated area on the skin filled with pus and erythematous base. The rash is
usually seen over extremities but not head or trunk.

Three fourths of the cases of DGI occur in women; susceptibility is increased if the primary
mucosal infection occurs during menstruation or pregnancy

Ophthalmia neonatorum:

Purulent conjunctivitis in neonates infected during vaginal delivery.

Untreated scarring of cornea, blindness

The most common cause of this disease is Chlamydia


trachomatis.

Whenever you treat a pregnant woman for gonorrhea you also


treat for Chlamydia not vice versa.
297

MERP Medical Bacteriology

Diagnosis: Culture and non-culture methods


Gonorrhea cannot be diagnosed solely on clinical grounds.
Gram-stain: - Good for symptomatic urethral infections
o For men, a smear of urethral exudate showing intracellular Gram-negative diplococci is
diagnostic.
o For women, culturing on selective medium is often required.
Culture: Thayer-Martin is a complex, moist, selective media with heated blood (chocolate agar) as a
base and added cysteine, extra amino acids, purines, pyrimidines and starch to neutralize
fatty acids. It is enhanced (10%) CO2 and several antibiotics to inhibit normal flora.
The specimen cultured on Thayer-Martin is usually obtained from male urethra or female
cervix. In case of arthritis it is obtained from the fluid of joints affected by arthritis.
Biochemical test: catalase and oxidase tests can be quickly performed on the cultures.
New tests: Not FDA-cleared for pharyngeal or rectal specimens but approved for urethral and cervical
specimen
o In many labs, culture has been replaced with sensitive nucleic acid tests done directly on
collected specimen.
o These tests do not require viable organisms and are useful where maintenance of viability
during specimen transport is a problem; however, it is not as sensitive as culture.
Example of such tests include:
1) Nucleic acid amplification tests (NAAT). NAATs detect and make many copies of the genetic
material (DNA) of gonorrhea bacteria. NAATs include polymerase chain reactions (PCRs) and
transcription mediated amplification (TMA). These tests are very accurate and can be done either on
a urine specimen or a sample of body fluid from the potentially infected area.
2) Nucleic acid hybridization test (DNA probe test, molecular probe test). Molecular probe
testing detects genetic material (DNA) of gonorrhea bacteria. This test is done on the body fluid
collected from the potentially infected area, most often the cervix or urethra. Often the molecular
probe test for gonorrhea also tests for chlamydia, another STI with symptoms similar to those caused
by gonorrhea. Serologic tests are not recommended for uncomplicated infections.
Control:
Recommended treatment for uncomplicated infections is a third-generation cephalosporin or a
fluoroquinolone plus an antibiotic (e.g., doxycycline) and is effective against possible coinfection with
298

MERP Medical Bacteriology

Chlamydia trachomatis. Sex partner(s) should be referred and treated. No effective vaccine yet
exists. Condoms are effective in preventing gonorrhea.
Practice questions:
1. Which characteristic pertains to meningitis- causing oxidase negative bacteria that have a welldefined capsule serotype and the infection with which is preventable by conjugating this capsule
to a protein?
a. Bile soluble
b. Expresses high levels of LOS
c. Requires hemin and NAD for growth
d. Requires Thayer-Martin media to grow
e. Appears as gram-negative diplococcic
2. After performing a latex agglutination test on a respiratory specimen of a patient symptoms
epiglottitis, you notice that the beads form a clump on the test slide. Which of the following is the
best conclusion?
a. The patients respiratory secretion contain antibodies to Haemophilus influenzeae
b. The patients serum contains antibodies to Haemophilus influenzae
c. The patients serum is negative for Haemophilus influenzae antigen
d. The patients respiratory secretion is positive for Haemophilus influenzae
e. This test is not specific for Haemophilus influenzae and needs to be repeated
3. A 3-month-old infant is brought to the pediatric ER in severe respiratory distress. The child
appears dehydrated, and there is prominent peripheral lymphocytosis. The chest radiograph
reveals perihilar infiltrates. The childs grandmother who watched the infant now that the mother
returned to work has had a dry hacking cough. The most likely causative agent is:
a. Haemophilus influenzae type b
b. Bodetella pertussis
c. Streptococcus agalacatiae
d. Corynebacterium diphtheria
e. Streptococcus pyogenes
4. What is the best way to isolate Neisseria gonorrhea?
a.
b.
c.
d.
e.

To culture a blood sample on Thayer-Martin agar


To culture blood sample on blood agar
To culture genital specimen on Thayer-Martin agar
To culture a genital specimen on blood agar
To culture a genital specimen on chocolate agar

5. Which of the following statements about Neisseria meningitides is true?


a.
b.
c.
d.
e.

A polysaccharide vaccine conjugates can be administered


People that lack complement components are as susceptible as normal patients
It is the most common cause of neonatal meningitis
It can only grow on Thayer Martin medium
It only leads to a local inflammatory disease
299

MERP Medical Bacteriology

6. Which of the following describes the morphology of the pathogen that causes gonorrhea?
a. Gram negative cocci in chains
b. Gram positive cocci in chains
c. Gram positive rods in pairs
d. Gram positive rods in chains
e. Gram negative cocci in pairs

300

MERP Medical Bacteriology

LECTURE 25: MYCOBACTERIA


Learning Objectives:
Describe the structure and pathogenesis of Mycobacteria tuberculosis, and detail how these bacteria
are structurally different from all the others discussed.

Describe acidfast stain


Describe reasons for the environmental resistance of TB
Describe the various presentations and stages of TB (Asymptomatic TB, Acute/Primary TB,
Latent/Reactivated TB, Systemic/Military TB)
Describe the histology and function of granulomas
Describe what happens in caseating necrosis
Detail the mechanism behind PPD test
List the populations at risk

Mycobacterium tuberculosis
General features:

Obligate aerobes growing most successfully in tissues


that have the highest partial pressure of oxygen, such
as lung apices

Facultative intracellular pathogens preferentially utilizing


mononuclear phagocytes as their habitats and only
rarely, if at all, inhabiting nonprofessional phagocytes

Slow-growing with a generation time of 12 to 18 hours

Lesions typically evolve over a chronic

Structure of M. tb envelope

course

High lipid content cell wall (see Figure above). Because the cells are hydrophobic and tend to
clump together, they are impermeable to the usual stains, ie, Gram's stain.

Mycobacteria are classified as acid-fast bacilli (AFB) because they retain the carbol-fuchsin
red dye after washing with acid, alcohol, or both.

Due to a high lipid content - extremely resistant to physical stress

60% of cell wall is made of mycolic acid which are long fatty acids

Mycobacterium tends to be quite resistant to most antibiotics such as Penicillin, but there are
some exceptions such as Streptomycin and Rifampin.

No exotoxin produced

301

MERP Medical Bacteriology

The Acid fast stain:


1. Add carbol fuchsin to slide
2. Heat slide
3. Wash slide with acid-alcohol
4. Counter stain with Methylene Blue
Virulence factors:

Cleared or latent TB

Symptoms of active or reactivated TB


302

MERP Medical Bacteriology

Pathogenesis:
Overview
Once organisms have made their way into the lung, they have four potential fates:
1. The initial host response to primary infection can be completely effective and kill all
bacilli, such that the patient has no chance of developing tuberculosis at any time in the
future.
2. The organisms can begin to multiply and grow immediately after infection, causing
clinical disease known as pulmonary tuberculosis or progressive primary TB.
3. Bacilli may become dormant and never cause disease at all, such that the patient has
what is referred to as latent infection and will manifest only by a positive tuberculin skin
test.
4. The latent organisms can eventually begin to grow, with resultant clinical disease,
known as reactivation (secondary) tuberculosis.
In most people infections are controlled.
Primary lesions that are controlled go through fibrosis and calcification. However, M.
tuberculosis can remain viable for a long time.
Some primary infections progress to reactivation or disseminated tuberculosis.
Multiplication in the non-activated macrophage can be followed by dissemination to lymph
nodes and bloodstream.
Details of progression:
Stage 1
The larger aerosol particles are trapped in the mucosal
surfaces and are removed by ciliated cells in the respiratory
tract.
Smaller particles with as little as 1-3 bacilli can reach alveolar
spaces and establish infection in susceptible host (inside a
macrophage).
Infection correlates to the number of organisms in the sputum
of a person with active disease.
Alveolar macrophages uptake bacteria.
Stage 2
2a. Organism prevents fusion of phagosome and lysosome and evade intravesicular killing by
destroying any oxidants that
are formed.
2b. Bacterial replication
eventually causes
macrophage to burst
2c. Surrounding blood DCs
phagocytose M. tb and bring it
to regional lymph nodes.

303

MERP Medical Bacteriology

Stage 3

Th1 cells can activate macrophages which can subsequently control the bacterial growth. If the
infectious load is small, granulomas /tubercles (localized collection of activated macrophages,
lymphocytes and fused Langerhans cells) prevent bacterial spread. In patients with a higher
bacterial load the center of the tubercle is characterized by "caseation necrosis" meaning semisolid or "cheesy" consistency. These eventually calcify and wall-off bacteria. M. tb cannot
multiply within these tubercles because of the low pH and anoxic environment.
90% of people infected with TB reach this stage but feel no symptoms. Another 5% of
people develop symptoms because the Th1 cells have a hard time fighting bacteria and become
over-activated (see stage 4/5). Yet in additional 5% of people without the initial symptoms, M. tb.
can persist within the tubercles for extended periods and be reactivated later in in life when the
immune response wanes. Immune response weakening is the reason why in many patients
disease does not develop until later in life.

Stage 4/5
The disease can also develop due to low Th1 cell numbers. However the pathology in TB can
also be due to an overactive Th1 response that for some reason cant contain the bacteria and
release a high level of cytokines that cause
systemic symptoms. Without containment,
the caseating centers of tubercle liquefy,
the pathogen starts growing and can
spread. From the bronchi TB can be
disseminated to various other lung
regions which then causes pulmonary
symptoms to develop. This damages the
lung severely. In primary and reactivated
TB, symptoms develop in about 10% of
the people initially infected and are
attributed to overreaction or under-reaction
of the immune response.
The tubercle can also invade an artery. The hematogenous spread may result in
extrapulmonary TB otherwise known as miliary tuberculosis.

304

MERP Medical Bacteriology

Predisposition to tuberculosis:
HIV infection is the #1 predisposing factor for M. TB. infection. 10 percent of all HIV-positive
individuals harbor M. TB. This is 400-times the rate associated with the general public. Having HIV
increases risk of TB reactivation 300 times. Other factors include:
Close contact with large populations of people, i.e., schools, day cares, nursing homes,
dormitories, prisons, etc.
Poor nutrition
Homeless
IV drug use
Alcoholism
Difference between TB Infection and TB disease:
TB infection means that M. tb is in the body but the immune system is keeping the bacteria
under control. The immune system
does this by producing macrophages
that surround the tubercle bacilli. The
cells form a hard shell that keeps the
bacilli contained and under control.
Most people with TB infection have a
positive reaction to the tuberculin
skin test. People who have TB
infection but not TB disease are NOT
infectious, i.e., they cannot spread
the infection to other people. These
people usually have a normal chest
x-ray.
Summary:
Primary TB infection (usually asymptomatic) - 75-90% of people activate good cellular
response and in 3-6 weeks bacteria ceases to replicate.
Active disease (Primary progressive):
~5% of patients, pulmonary macrophages are unable to contain
the bacilli and are overwhelmed, leading to a clinically apparent
infection. Symptoms may include lower respiratory react infection
with night sweats, cough, weight loss, and bloody sputum. Usually
both lung lobes are affected.
Reactivated disease (Secondary progressive, latent
progressive) - In 5-20% of patients who cannot completely
eradicate bacteria during primary stage, disease develops later in
life due to a reactivation of a primary lesion that may have been
quiescent for years. Large necrotic areas in the lungs, the bronchus and small blood vessels can be
305

MERP Medical Bacteriology

noted. Untreated, this will lead to disseminated disease and death. Untreated tuberculosis has a
mortality rate of 50%.

Diagnosis:

Stain:
Clinical specimen can be directly analyzed by acid-fast stain, and
then particular mycobacterial specie is confirmed by PCR.

Culture:
M. tb grows very slowly, so collected specimen is decontaminated with
2% NaOH to kill other contaminating, rapidly dividing bacteria and are
then plated on a Lowenstein-Jensen medium which is an egg based
medium. Increased CO2 (5-10%) also helps bacterial growth. Bacteria
will appear in 2 weeks. These colonies can be tested for niacin
production which is unique for M. tb.

306

Culture

MERP Medical Bacteriology

TB test:
A positive tuberculin skin test usually develops 3-4 wks. after initial immune conversion and remains
for life. A positive result should be followed by a chest Xray.
PPD test

Chest X-ray:
In active pulmonary TB, infiltrates or consolidations and/or cavities are often seen in the upper
lung. However, lesions may appear anywhere in the lungs. Old
healed tuberculosis usually presents as pulmonary nodules in
the hilar area or upper lobes, with or without fibrotic scars and
volume loss.
Pleural scarring may be present. Nodules and fibrotic scars
may contain slowly multiplying tubercle bacilli with the potential
for future progression to active tuberculosis. Persons with these
findings should be considered high-priority candidates for

Chest X-ray

treatment of latent infection. Conversely, calcified nodular


lesions pose a very low risk for future progression to active tuberculosis. Abnormalities on chest
radiographs may be suggestive of, but are never diagnostic of, TB.

Quantiferon Gold test:


Interferon Gamma Release Assays measure the cell mediated response in infected individuals
through the levels of interferon gamma released. During an infection, T cells from the individual will
be sensitized (via MHC II proteins) to the antigens presented by macrophages of the infected
organism. Memory Th1 cells will then be releasing interferon-gamma upon recognizing a macrophage
presenting tuberculin antigen. Interferon gamma release assays take advantage of this natural
process in infected immunocompetent individuals. When tuberculin purified protein derivatives are
applied to whole-blood samples from infected individuals, Th1 cells previously sensitized to the
307

MERP Medical Bacteriology

antigens will be present in the blood, and will bind to the MHCII/antigen complex. The T cells will then
release Interferon-Gamma which can be detected by an ELISA type assay; the presence of
sensitized T-cells in infected individuals will result in far higher levels of IFN-G release than among
uninfected individuals.
Practice questions:

1. Which of the following statements about the purified protein derivative (PPD) and tuberculin test is
most correct?
a. It is strongly recommended that the medical and other health science students have PPD test
every year.
b. Persons immunized with BCG rarely if ever convert to positive PPD skin test.
c. A positive tuberculin test result indicates that an individual has been infected with M. tuberculosis
in the past.
d. A positive PPD skin test results imply that a person is immune to active tuberculosis.
e. The first intradermal PPD test may result in the sensitization of a persons immune system and
cause that individual to develop anti-tuberculin response.
2. A 10-year-old child has a primary pulmonary tuberculosis disease. Which of the following features
of tuberculosis is most correct?
a. In primary tuberculosis an active exudative lesion develops and spreads to the to the lymphatics
and regional lymph nodes
b. The exudative lesion of primary tuberculosis rarely heals spontaneously
c. If tuberculosis develops later, it usually is a result of exposure to a different strain of tuberculosis
d. A PPD test, which tests for the antibody titer again tuberculosis, should test positive in the child
e. A PPD test will be positive in the child only after secondary exposure to tuberculosis
3. Which of the following statements regarding interferon-gamma release assay is correct?
a.
b.
c.
d.
e.

It is useful for evaluating immunocompromised patients for active tuberculosis


It detects antigens common to all Mycobacteria species
It is used to detect antibodies in a patient exposed to tuberculosis
It uses a molecular probe to detect the DNA of the organism
It is used as an alternative to the tuberculin skin test to evaluate for latent TB

308

MERP Medical Bacteriology

309

Anda mungkin juga menyukai